不等式スレッド

このエントリーをはてなブックマークに追加
1132人目の素数さん
有名な不等式からマニアックな不等式について語り合いましょう。
22:03/12/27 16:28
2
3132人目の素数さん:03/12/27 16:39
ベルの不等式が有名だすね
4まる:03/12/27 16:40
だるま不倒式
ダルマさんが倒れないための条件を与える不等式です。
5132人目の素数さん:03/12/27 16:56
ちんこ不等式
6132人目の素数さん:03/12/27 17:01
チェビシェフだろ
7132人目の素数さん:03/12/28 00:19
seele futousiki
8132人目の素数さん:03/12/28 00:21
三角不等式が一番使える。
9132人目の素数さん:03/12/29 01:17
AMGM
10132人目の素数さん:03/12/29 01:19
0≦1
これ有名。=を抜かさないのがカコイイ
11ティム:03/12/29 01:35
コーシーシュワルツエネッガーの不等式だっけ?」
12132人目の素数さん:03/12/29 01:46
>>11
エいらないよ。とマジレスしてみるテスト
13ティム:03/12/29 02:38
>>12
ミスった。
シュワルツェネッガーね。
14ティム:03/12/29 02:45
ヤッホーでシュワルツネッガーで検索すると
 _
/!\キーワードに間違いはありませんか? シュワルツェネッガーでオ☆ニーをしてください。
 ̄ ̄ ̄
ってでましたが何か?
15132人目の素数さん:04/01/09 16:42
age
相加相乗調和
ヘルダーの不等式
ミンコフスキーの不等式
17132人目の素数さん:04/01/09 19:58
良スレじゃねぇか
(´д`;)ハァハァ
18132人目の素数さん:04/01/09 20:38
不等式だけ集めた本もある。
このスレに期待していいのですか?
(;´д`)ハァハァ
20132人目の素数さん:04/01/09 21:09
自分で調べて自分でネット検索して自分で
考えて
考えて
考えて
喜ぶ。
ちなみにその本には確かこうある。
ある者は蝶を集め、ある者は切手を収集し、ある者は不等式を集める。
21132人目の素数さん:04/01/09 21:41
不等式への招待 現代数学ゼミナール
大関 信雄 (著)
三角形の3辺の長さについての不等式
超越関数を含まない不等式
三角関数、指数・対数関数についての不等式と単調性
(相加平均)≧(相乗平均)の証明方法
相加平均・相乗平均を含む不等式
凸関数についてのJensenの定理
負にならない2次形式について
不等式の作成と証明法
凸関数と優数列について
22132人目の素数さん:04/01/09 21:43
三角形の3辺の長さについての不等式
超越関数を含まない不等式
三角関数、指数・対数関数についての不等式と単調性
(相加平均)≧(相乗平均)の証明方法
相加平均・相乗平均を含む不等式
凸関数についてのJensenの定理
負にならない2次形式について
不等式の作成と証明法
凸関数と優数列について
積分の不等式

23132人目の素数さん:04/01/09 21:53
不等式
[原書名:Inequalities〈Hardy, Godfrey Harold;
Littlewood, John Edensor;P´olya, George〉 ]
417p 21cm(A5)
シュプリンガー・フェアラーク東京 (2003-10-04出版)

・ハーディ,G.H.〈Hardy,Godfrey Harold〉・リトルウッド,J.E.〈Littlewood,John Edensor〉・ポーヤ,G.【著】〈P´olya,Geor
[A5 判] NDC分類:413.51 販売価:\4,800(税別)
本書は、G.H.ハーディ、J.E.リトルウッド、G.ポーヤという数学界
の巨星たちが、数学のあらゆる分野において重要な役割を果たす不等式たち
について、その起源を遡り、コンパクトに証明をまとめた書。
本書の前半部分では、まず解析学において重要かつ基本的な算術平均と幾何
平均の定理(定理9)、ヘルダーの不等式(定理11)、ミンコフスキの不
等式(定理24)を紹介し、それぞれについて、いくつもの視点からの全く
異なる証明を与えている。
そして後半部分では、前半で提起された問題についてさらに詳細に考察がな
されており、複素関数の理論、フーリエ級数の理論、直交関数系の一般論な
どへの格好の入門書ともなっている。

第1章 序論
第2章 基本的平均
第3章 関数の平均と凸関数の理論
第4章 微分積分学の応用
第5章 無限級数
第6章 積分
第7章 変分法の応用
第8章 双線形形式と多重線形形式に関する定理
第9章 ヒルベルトの不等式とその類似と拡張
第10章 再配列

24132人目の素数さん:04/01/10 10:57
不等式ヲタからの質問。

不等式(大関信雄、青木雅計)槇書店
P.153, 5-7行があやしい。
あれでは a_kとb_kが無関係でなくなるのでは?
25132人目の素数さん:04/01/26 02:52
名スレ上げ
26132人目の素数さん:04/01/26 22:37
0≧1≧2
27KingMathematician ◆5lHaaEvFNc :04/01/27 03:52
a<=bから束を作ってみる。
a<=bに相当する関係式をa∪b=b,a∩b=aとする。
a∪b=max{a,b},a∩b=min{a,b}とすると、a<=bならばa∪b=b,a∩b=aである。
また、a>bならば、a∪b≠b,a∩b≠aである。
よって、通常の順序に対応する束の演算は、maxとminである。
>>24
考えて見るかと思って、見たんだが、俺の持っているのではP153はTから
Zまでの索引であった。ちなみに1992初版第2刷となっておる。
2928:04/01/27 13:17
>>24
本が違ってた。
俺のは「不等式への招待」近代科学社
君のは不等式なんだね。
3028:04/01/27 13:35
>>24
式書いてみて?
数学の本にはよく誤植があるから、、、。
716
ここに書くには余白が…
質問しておいて偉そうだが、図書館で読んで欲しい。
私は古本屋で運良く手に入れた。
33132人目の素数さん:04/02/15 04:55

タイトルは(;´д`)ハァハァなのに、誰もいない悪寒…。
>>1はどこヘ逝った? 建て逃げかよ…。 _ト ̄|○
ところで小難しいのを見つけたのですが、スッキリ証明できません。
よろしくお願いします。

正の数a,b,x,yに対して、次の不等式を証明せよ。等号成立条件も示せ。
 (ax^2+by^2)^3 ≦ (a^3+b^3)(x^3+y^3)^2
ただのヘルダーの不等式
>>35
_ト ̄|○ 知らなかった…

とりあえず、ヘルダーの不等式を調べてみまつ。
おれはついに、ハーディー、リトルウッド、ポリヤの不等式を買った。
これから、浸かります。
俺は、>>37さんの言う本も、最近出た「不等式の工学へ応用」
も買ったが、いまだ読んでいない
「恐ろしく難解な問題をだせ! 223,241,967」より

1 ≧ F(a,b,c) ≡ a/(b+c+1) + b/(c+a+1) + c/(a+b+1) + (1-a)(1-b)(1-c) ≧ 7/8.

f(x)=1/(x+1)-1+x/2 とおくと、与式=a・f(b+c)+b・f(c+a)+c・f(a+b)+1-abc だから、求める式は、f(b+c)≦bc/3, etc.
0<x<1のとき、f(x)=-x(1-x)/2(x+1)<0≦bc/3.
1<x<2のとき、f(x)-(x-1)/3=(x-1)(x-2)/2(x+1)<0 より f(b+c)<(b+c-1)/3≦bc/3. q.e.d.

a+b+c=s とおき、F(a,b,c) ≧ F(s/3,s/3,s/3) ≧ 7/8 と分ける。
右側は F(m,m,m)=3m/(2m+1)+(1-m)^3=7/8+(m-1/2)^2・{1/2+m(3-2m)}/(2m+1)≧7/8.(0≦m≦1)
等号成立は m=1/2のとき.

基本対称式を a+b+c=s, bc+ca+ab=t, abc=u とおくと、
F(a,b,c)−F(s/3,s/3,s/3)=(s+1){(7s/3+2)(s*s/3-t)-3(s^3/27-u)}/[(2s/3+1){(2s/3+1)^2-(s+1)(s*s/3-t)+(s^3/27-u)}]
-(s*s/3-t) + (s^3/27-u).

基本対称式を使って表すところが、うまくいかんですだ。
もっと楽な方法ないですか?
オレの計算力では、うまくいかん…
_ト ̄|○
>>39
F(a,b,c)−F(s/3,s/3,s/3)=(s+1){(7s/3+2)(s*s/3-t)-3(s^3/27-u)}/[(2s/3+1){(2s/3+1)^2-(s+1)(s*s/3-t)+(s^3/27-u)}]
-(s*s/3-t) + (s^3/27-u).

負の数-(s*s/3-t)が混じっているのだが、これが正の数になるのか?
F(a,b,c)−F(s/3,s/3,s/3)
=(s+1){2(s^2-3t)+(2s^3-7st+9u)}/[(2s+3)(a+1-a)(s+1-b)(s+1-c)]
 +(s^3-9s^2+27t-27u)/27
右辺第一項は正だから、第二項が正になることを示せばよいのだけど…
[補足]
s^2-3t = {(a-b)^2+(b-c)^2+(c-a)^2}/2 ≧ 0
2s^3-7st+9u
=(a+b)(a-b)^2+(b+c)(b-c)^2+(c+a)(c-a)^2 ≧ 0
第二項は (s,t,u)=(1,0,0)のとき負になるから
やっぱり、第一項と第二項を一緒に考えて、
正の数になることを示さないといけないのか…
降参です。
>>39の問題を書いたやつ出て来いや
ヽ(`д´)ノ ガロァ!
F(a,b,c)−F(s/3,s/3,s/3)≧0
の反例が見つかれば楽なんだけど…
計算が大変そうだが、F(a,b,c)をaの関数とみて微分して
0≦a≦1の範囲で最小値を求める方法を考えてみるかな…
              ☆ チン

        ☆ チン  〃 ∧_∧  
          ヽ ___\(\・∀・) 経過報告まだぁ〜?
             \_/⊂ ⊂_)_
           / ̄ ̄ ̄ ̄ ̄ ̄ ̄/|
        |  ̄  ̄ ̄ ̄ ̄ ̄ ̄:| :|
        |            |/
         ̄ ̄ ̄ ̄ ̄ ̄ ̄ ̄
チン      ☆  チン       ☆
       チン    マチクタビレタ〜   チン     ♪
           ♪
    ♪          ☆チン    .☆   ジャーン!   マチクタビレタ〜!
        ☆ チン   〃  ∧_∧  ヽ         / ̄ ̄ ̄ ̄ ̄ ̄ ̄ ̄ ̄ ̄
          ヽ  ___\(・∀・ #) /\_/ <  まだー?
        チン    \_/⊂    つ    ‖     \__________
           / ̄ ̄ ̄ ̄ ̄ ̄ ̄/|     ‖        マチクタビレタ〜!
        |  ̄  ̄ ̄ ̄ ̄ ̄ ̄:| :|   /|\
        |             |/
49132人目の素数さん:04/02/26 00:21
期待あげ
>>39の問題未定定数法でとけるとおもうけど。
F(a,b,c)≧F(s/3,s/3,s/3)をしめすんでしょ?
a+b+c=sという束縛条件下で
L=・・・-λ(a+b+c-s)とおいて
La=Lb=Lc=Ls=0をとく。たぶんがんばればa=b=c=s/3がでると思うんだけど
がんばらなくてもa=b or b=c or c=aまではなんとかでるからそれから
f(x)= x/(x+s-2x+1) + x/(s-2x+x+1) + (s-2x)/(x+s+1) + (1-x)(1-x)(1-s+2x)
の増減表かいたらできたよ。
>>39の問題を複雑な計算を極力さけるというコンセプトのもとにといてみました。
変数が鬱陶しいのでp=1-a、q=1-b、r=1-cとおいて目標は

 0<p<1、0<q<1、0<r<1の範囲で
 F(p,q,r)=(1-p)/(3-q-r)+(1-q)/(3-r-p)+(1-r)/(3-p-q)+pqr≧7/8 (等号はp=q=r=1/2のとき)
 
でこれをしめすために補題として
G(x,y,z)=(1-x^2)/(3-t^3/x)+(3-y^2)/(3-t^3/.y)+(3-z^2)/(3-t^3/z)+t^6 とおくとき
 
 (1)0<t<1に対し0<x<1、0<y<1、0<z<1、xyz=t^3の範囲で
  G[t](x,y,z)≧G[t](t,t,t) (等号はx=y=z=tのとき)
 
 (2)0<p<1、0<q<1、0<r<1の範囲でF
  (p,q,r)≧G[√(pqr)](√p√,q,√r)
 
 (3)G[t](t,t,t)≧7/8 (0<∀<1) (等号はt=1/2のとき)
 
をしめせば十分。(2)は相加相乗平均、(3)は微分すればよし。でやや大変なのは(1)。
G(x,y,z)=(x-x^3)/(3x-t^3・x^2)+(y-y^3)/(3y-t^3・y^2)+(y-y^3)/(3y-t^3・y^2)+t^6
を考える。まずx=e^u、y=e^v、z=e^wと変換して範囲はu,v,w<0、u+v+w=logt。この領域で
(x-x^3)/(3x-t^3・x^2)は(t^3<x<1)において下に凸な関数でx=e^uも凸だから合成関数
(e^u-e^(3u))/(3e^u-t^2・e^(2u))も所与の(u,v,w)の範囲で凸。よってG[t]は所与の範囲で凸。
しかも狭義凸なので最小値はあっても一ヶ所のみでgradG[t]=(0,0,0)なる点が内部にあれば
そこが最小。一方束縛条件を無視するとG[t]はx,y,zの置換で不変なので
gradG[t](t,t,t)もx成分、y成分、z成分の置換で不変ゆえそれは(1,1,1)に平行。
それは束縛条件のgradと等しいので特に束縛条件下では(x,y,z)=(t,t,t)でgradG[t]は0ベクトル。
つまりここが最小。
5251:04/02/26 22:42
スマソ。計算まちがいしてた。
>(x-x^3)/(3x-t^3・x^2)は(t^3<x<1)において下に凸な関数
これ成立しません。最終的に3つたしたg[t]が凸はあってるみたいだけど・・・
今一歩カコワルイ。もちょっとなんとかならんものか・・・
5351:04/02/26 23:13
修正と差し替え
まずG[t](x,y,z)の定義は
G(x,y,z)=(1-x^2)/(3-2t^3/x)+(3-y^2)/(3-2t^3/.y)+(3-z^2)/(3-2t^3/z)+t^6
でこれにx=e^u、y=e^v、z=e^wを代入してえられる関数のu+v+w=3logt、u,v,w<0における
凸性の証明を以下にさしかえ。
H(u)=log(e^u-e^(3u))-log(3e^u-2t^3)とおく。第1項は微分すると
log(e^u-e^(3u))'=(e^u-3e^(3u))/(e^u-e^(3u))=3+4e^u/(e^u-3e^(3u))は単調増大ゆえ凸。
第2項は凸関数-log(3e-2t^3)に凸関数e=e^uを合成したものなので凸。
ゆえにH(u)は凸。よってe^H(u)=(e^u-e^(3u))/(e^u-2t^3)は凸。よってG[t](e^u,e^v,e^w)は
所与の領域で凸。
しまった・・・まだまちがってる。もういや。
5554:04/02/26 23:40
いやあってた。これ以上はスレ汚しくさいので撤退。
>>50-55
ありがとうございます。
印刷してジックリ追ってみます。
ムズイ。
普通の高校生にも分かるような解き方は無理でしょうかねぇ…
2004 JMOの問題らしい。
別解が挙げられてたが、JMOの模範解答も知りたいです… (つД‘)・゚・。

a+b+c=1を満たす正の実数a,b,cに対して
{(1+a)/(1-a)}+{(1+b)/(1-b)}+{(1+c)/(1-c)}≦2{(b/a)+(c/b)+(a/c)}
が成立することを証明せよ.ただし,等号が成立する条件を述べる必要はない.

別解 a+b+c=1から(1+a)/(1-a)=1+2a/(1-a)=1+2a/(b+c)などとなるので、示すべき不等式は
3+2(a/(b+c)+b/(c+a)+c/(a+b))=<2(b/a+c/b+a/c)
と同値であり、さらに、a/c-a/(b+c)=ab/c(b+c)などにより、これは、
ab/c(b+c)+bc/a(c+a)+ca/b(a+b)>=3/2
と同値である。この式の両辺はa,b,cについての斉次式なので、a+b+c=1の条件を取り除いて、
任意の正の実数a,b,cについて上の不等式をしめせばよい。そのためには、abc=1なる正
の実数a,b,cについて
S=1/c^2(b+c)+1/a^2(c+a)+1/b^2(a+b)>=3/2
を示せば十分である。コーシー・シュワルツの不等式より、
{(b+c)+(c+a)+(a+b)}S>=(1/c+1/a+1/b)^2=(ab+bc+ca)^2
なので,S>=(ab+bc+ca)^2/2(a+b+c)となる。この式の右辺が3/2以上であることを示せば良いが、それは
(ab+bc+ca)^2/2(a+b+c)>=3/2⇔(ab+bc+ca)^2>=3(a+b+c)
⇔(ab+bc+ca)^2>=3(a+b+c)abc
⇔1/2{(ab-bc)^2+(bc-ca)^2+(ca-ab)^2}>=0
からわかる。よって示された。
                _∧_∧_∧_∧_∧_∧_∧_∧_
     デケデケ      |                         |
        ドコドコ   < JMOの模範解答UPはまだかね?!        >
   ☆      ドムドム |_  _  _ _ _ _ _ _ _ _|
        ☆   ダダダダ! ∨  ∨ ∨ ∨ ∨ ∨ ∨ ∨ ∨
  ドシャーン!  ヽ      /ヘ;;;;;     オラオラッ!!    ♪
         =≡=  ';=r=‐リ      ☆
      ♪   / 〃  ヽ二/    / シャンシャン
    ♪   〆  ┌\と\と.ヾ∈≡∋ゞ
         ||  γ ⌒ヽヽコ ノ  ||
         || ΣΣ  .|:::|∪〓  ||   ♪
        ./|\人 _.ノノ _||_. /|\
[分からない問題はここに書いてね4]より

a,b,c は三角形の3辺で、x,y,z は x+y+z=0 をみたす実数のとき
  a^2xy + b^2yz + c^2zx ≦ 0

とりあえずx,y≧0≧zとして一般性をうしなわない。左辺に(-1)かけてzを消去して
左辺×(-1)
=a^2xy + b^2yz + c^2zx
=(c^2)x^2+(b^2+c^2-a^2)xy+(b^2)y^2
=(c^2)x^2+2bc(cosθ)xy+(b^2)y^2  ←abcを三辺とする3角形のaの対辺の角をθとした。
≧(c^2)x^2-2bcxy+(b^2)y^2
=(cx-by)^2
≧0
[分からない問題はここに書いてね4 : 226-]より

a,b,cは三角形の3辺、rは内接円の半径、Rは外接円の半径のとき
 r/(2R) ≦ abc / sqrt{2(a^2+b^2)(b^2+c^2)(c^2+a^2)}
(出典 MOCP 2000.37)
http://www.cms.math.ca/Competitions/MOCP/2000/sol_oct.pdf

鋭角三角形の場合は模範解答が有効なので、Aが鈍角の場合を考えまつ。

= (bc/2)sin(A) ≦ bc/2 ・・・・・・ (1).
A≧π/2 より, (b^2+c^2) ≦ a^2 ・・・・・・ (2).
となるので、残りの (a^2+b^2)(a^2+c^2) が 2(as)^2 より小なることを示しまつ。
F = 2(as)^2 - (a^2+b^2)(a^2+c^2)
= (1/2)(a^2+2bc)(a^2-b^2-c^2)+(b+c-a)a^3 +bc(b^2-bc+c^2)
> (1/2)(a^2+2bc)(a^2-b^2-c^2).
ここで A≧π/2 なので、F>0.
∴ (a^2+b^2)(a^2+c^2) < 2(as)^2 ・・・・・・ (3).

上記 (1)^4 ×(2)×(3) より,
(a^2+b^2)(b^2+c^2)(c^2+a^2)・(竸4) < (1/8)(abc)^4・(s^2).
∴ r/(2R) = 2(竸2)/(abcs) < abc/sqrt{2(a^2+b^2)(b^2+c^2)(c^2+a^2)}.

くだらねぇ問題はここへ書け ver.3.14(27桁略)9502 [711-727] より

実数a,b,cが a<b<c かつ a+b+c=0 をみたすとき
「ア」> (a^2+b^2+c^2)/{(a-c)^2} ≧ 「イ」

(a^2+b^2+c^2)/{(a-c)^2}
=2(a^2+ac+c^2)/{(a-c)^2}  (b=-a-cを代入)
=2(t^2+t+1)/(t-1)^2  (t=a/c とおいた)
=6{1/(1-t) - (1/2)}^2 + 1/2

条件 a<(-a-c)<c から -2<t<-1/2 なので、1/(1-t)=x とおくとき、
1/3<x<2/3 における 6(x-1/2)^2+1/2 の最大最小値問題。

∴ 2/3 > (a^2+b^2+c^2)/{(a-c)^2} ≧ 1/2
くだらねぇ問題はここへ書け ver.3.14(27桁略)9502 [751]より

実数a,b,cに対し、
√{a^2+(b-1)^2}+√{b^2+(c-1)^2}+√{c^2+(a-1)^2} ≧ 3/(√2)

解法は…    _| ̄|○
くだらねぇ問題はここへ書け ver.3.14(27桁略)9502 [761]より

√{(x+1)^2+(y-1)^2}+√{(x-1)^2+(y+1)^2}+√{(x+2)^2+(y+2)^2} ≧ ?

点P(x,y), A(-1,1), B(1,-1), C(-2,-2)とおくと、
PA+PB+PCの最小値だから Pはフェルマー点で…
√{a^2+(b-1)^2}+√{b^2+(c-1)^2}+√{c^2+(a-1)^2}
>= (|a|+|b-1|)/√2 + (|b|+|c-1|)/√2 + (|c|+|a-1|)/√2
= {(|a|+|a-1|) + (|b|+|b-1|) + (|c|+|c-1|)}/√2
>= 3/√2
>>65
神降臨キタ━(゚∀゚)━!!!! 感謝。
自分では思いつけそうもないや…。 _ト ̄|○
(1/4)≦a≦b≦c≦1, x+y+z=0 のとき ayz+bzx+cxy≦0 を示せ。

たぶん、一文字消してゴチャゴチャでしょう。
ttp://www.kalva.demon.co.uk/bmo/bsoln/bsol903.html より

実数x,y,zに対して √(x^2+y^2-xy)+√(y^2+z^2-yz) ≧ √(z^2+x^2+zx)
[I] x,y,zの中に0であるものが存在すれば、結論の不等式が成り立つことはすぐにわかる。

これ以降x,y,z≠0としてよく、(a/x)+(b/y)+(c/z)の符号をみていくことにする。

[II] x,y,zの内、1つが負、2つが正の時
(i) x or yが負の場合
 それぞれ場合について(a/x)+(c/z)≧0,(b/y)+(c/z)≧0が成立するから、
 (a/x)+(b/y)+(c/z)≧0
(ii) z が 負の場合
 (a/x)+(b/y)+(c/z)≧(1/4)(1/x)+(1/4)(1/y)+(1/z)≧0(∵凸不等式)

(i),(ii)のいずれの場合も(a/x)+(b/y)+(c/z)≧0ゆえ、
両辺にxyz(<0)を掛けて、結論が得られる。

[III] x,y,zの内、2つが負、1つが正の時
x,y,zの替わりに-x,-y,-zを考えれば、[II]に帰着できる。
またもや神降臨 >>69>>67の解答っすね。
キター *・゜゚・*:.。..。.:*・゜(゚∀゚)゚・*:.。..。.:*・゜゚・* !!!!!
>>68
∠AOB = ∠BOC = ∠AOC/2 = π/3、OA=a, OB=b, OC=c とおいて、AB+BC≧CA
等号成立条件は a=b=2b または b(c+a)=ca でOKですか?
>>69
>[II] x,y,zの内、1つが負、2つが正の時
>(i) x or yが負の場合
> それぞれ場合について(a/x)+(c/z)≧0,(b/y)+(c/z)≧0が成立するから、

ここが分かりません… _| ̄|○
72は引用先のミス

(ii) z が 負の場合
 (a/x)+(b/y)+(c/z)≧(1/4)(1/x)+(1/4)(1/y)+(1/z)≧0(∵凸不等式)

ここがわかりませんです。
74132人目の素数さん:04/03/09 17:05
          ...,、 -  、
      ,、 '  ヾ 、    丶,、 -、
     /    ヽ ヽ  \\:::::ゝ
 /ヽ/   i  i    ヽ .__.ヽ ヽ::::ヽ
 ヽ:::::l i.  l  ト  ヽ  ヽ .___..ヽ 丶::ゝ
 r:::::イ/ l  l.  i ヽ  \ \/ノノハ  ヽ
 l:/ /l l.  l  i  ヽ'"´__ヽ_ヽリ }. ',  ',
 'l. i ト l  レ'__    '"i:::::i゙〉l^ヾ  |.i. l    ____________
. l l lミ l /r'!:::ヽ    '‐┘ .} /  i l l  /教科書読みましょう。
  l l l.ヾlヽ ゝヾ:ノ   ,     !'"   i i/ i< その程度自分でやりましょう。
  iハ l  (.´ヽ     _   ./    ,' ,' '  | 脳味噌ありますか? 無いんですか?
   |l. l  ` ''丶  .. __  イ         | それなら学校辞めて
   ヾ!        l.   ├ァ 、        \ペプシ工場で働きましょうよ。
          /ノ!   /  ` ‐- 、      ̄ ̄ ̄ ̄ ̄ ̄ ̄ ̄ ̄ ̄ ̄ ̄ ̄
         / ヾ_   /     ,,;'' /:i
        /,,  ',. `  /    ,,;'''/:.:.i
>>74
氏ね。
そういえば、高校のころはJMO系のマニアックな不等式が
一番苦手だったなぁ。下手に評価を緩くしてあったりすると
逆に難しく見えたりするんだよね。
TVみながら考えてて分かった。
>>74
(1/4)(1/x)+(1/4)(1/y)+(1/z)≧0(∵凸不等式)

凸不等式を使ってるのは、はじめの2項だけですね。
謎はすべて解けたッ!
とりあえず、コヨタンは二度と来るな!
          ...,、 -  、
      ,、 '  ヾ 、    丶,、 -、
     /    ヽ ヽ  \\:::::ゝ
 /ヽ/   i  i    ヽ .__.ヽ ヽ::::ヽ
 ヽ:::::l i.  l  ト  ヽ  ヽ .___..ヽ 丶::ゝ
 r:::::イ/ l  l.  i ヽ  \ \/ノノハ  ヽ
 l:/ /l l.  l  i  ヽ'"´__ヽ_ヽリ }. ',  ',
 'l. i ト l  レ'__    '"i:::::i゙〉l^ヾ  |.i. l    ____________
. l l lミ l /r'!:::ヽ    '‐┘ .} /  i l l  /教科書読みましょう。
  l l l.ヾlヽ ゝヾ:ノ   ,     !'"   i i/ i< その程度自分でやりましょう。
  iハ l  (.´ヽ     _   ./    ,' ,' '  | 脳味噌ありますか? 無いんですか?
   |l. l  ` ''丶  .. __  イ         | それなら学校辞めて
   ヾ!        l.   ├ァ 、        \ペプシ工場で働きましょうよ。
          /ノ!   /  ` ‐- 、      ̄ ̄ ̄ ̄ ̄ ̄ ̄ ̄ ̄ ̄ ̄ ̄ ̄
         / ヾ_   /     ,,;'' /:i
        /,,  ',. `  /    ,,;'''/:.:.i

くだらねぇ問題はここへ書け ver.3.14(27桁略)9502 [923,931]

非負実数a,b,cがa^2+b^2+c^2+abc=4をみたすとき
0≦ab+bc+ca-abc≦2を示せ。

まずD={(a,b,c) | a,b,c≧0&a^2+b^2+c^2+abc≦4}とおく。
関数a^2+b^2+c^2+abcは凸関数なのでDは凸領域。
0≦ab+bc+ca-abc≦2&与式
⇔8≦(a+b)^2+(b+c)^2+(c+a)^2≦12&与式
なのでコレをしめす。まずDにおいてa+b+c≦3をしめす。Dは凸なのでa+b+cは境界で最大。
a=0orb=0orc=0などではa+b+c≦2。a^2+b^2+c^2+abc=4上では
Dの凸性から(a,b,c)=(1,1,1)のときが最大でa+b+c≦3。
よって(a+b)^2+(b+c)^2+(c+a)^2≦3((2a+2b+2c)/3)^2≦12。
つぎにE={(a,b,c) | a,b,c≧0&(a+b)^2+(b+c)^2+(c+a)^2<8}とおく。Eも凸であり凸性から同様にして
E上でabc≦(1/3)^(3/2)。よってabc≦3(abc)^(2/3)でありよって
abc≦3(abc)^(2/3)≦(ab+bc+ca)
よってa^2+b^2+c^2+abc≦a^2+b^2+c^2+(ab+bc+ca)≦(1/2)((a+b)^2+(b+c)^2+(c+a)^2)<4
とくにD&E=φ。よって(a,b,c)∈D⇒(a+b)^2+(b+c)^2+(c+a)^2≧8
(実は(a,b,c)=(2,0,0)において等号が成立するので最小値でもある。)

>>78
言うだけ無駄かもしれんが、いい子にして荒らすなよ。
デタラメな解凍貼るな馬鹿
>>80
すまん、読まずに貼り付けた。
間違ってたのか… _| ̄|○
82132人目の素数さん:04/03/19 12:43
83132人目の素数さん:04/03/29 23:59
,
そういえばシュプリンガーから『不等式』なんて本が
出てたよね。アレ読んだ人感想キボンヌ。
85132人目の素数さん:04/03/30 03:20
買うには買ったがまだ味読してる程度。
86132人目の素数さん:04/03/30 14:49
買うには買ったが、本棚に飾ってる程度。
87132人目の素数さん:04/04/04 19:18
<<
『不等式への招待』なる本を見つけたのでとりあえずage
>>88
(゚∀゚) イイヨ イイヨ-
これで君も同志だ!
実数a,b,cがa+b+c=1をみたすとき、
(a+1/b)(b+1/c)(c+1/a) ≧ 1000/27

a,b,c>0, a+b+c=S≦3 とする。
F(a,b,c) = (a+1/b)(b+1/c)(c+1/a) = abc + S + (1/a+1/b+1/c)+ 1/(abc)
相乗平均≦相加平均 (1) より、abc ≦ (S/3)^3 ≦1.
g(u)≡u+1/u に対し、平均変化率 [g(v)-g(u)]/(v-u) = 1-1/(uv).
∴ 0<u<v≦1のとき、[g(v)-g(u)]/(v-u) < 0.
∴ g(abc) ≧ g{(S/3)^3}.
調和平均≦相加平均 (2) より、(1/a+1/b+1/c)≧9/(a+b+c) = 9/S.
以上を加えて、 F ≧ (S/3)^3 + S + 9/S + (3/S)^3 = (S/3+3/S)^3.
等号成立は a=b=c=S/3 のとき.

(注1) (a+b+c)^3 - 27・abc = (3a+S/2)・|b-c|^2 + (3b+S/2)・|c-a|^2 + (3c+S/2)・|a-b|^2 ≧0.
(注2) (a+b+c)・(1/a+1/b+1/c)≧ 3^2 (Cauchyの不等式)
a+1/b = a+9/(9b) >= 10*(a/(9b)^9)^(1/10)

(a+1/b)(b+1/c)(c+1/a) >= 1000*(1/((3^27)*(abc)^4))^(1/5) >= 1000/27
>>90-91
こんな数ヲタ達に囲まれて、ぼかぁ幸せ者だ…

   ┏┓  ┏━━┓        / ̄ ̄ ̄ ̄ ̄ ̄\          .┏━┓┏━┓
 ┏┛┗┓┃┏┓┃        /           \       ┃  ┃┃  ┃
 ┗┓┏┛┃┗┛┃┏━━━/               ヽ.━━━┓┃  ┃┃  ┃
 ┏┛┗┓┃┏┓┃┃      l:::::::::    \    /    |     ┃┃  ┃┃  ┃
 ┗┓┏┛┗┛┃┃┗━━━|::::::::::   (●)   (●)   |━━━┛┗━┛┗━┛
   ┃┃      ┃┃        |:::::::::::::::::   \___/    |       ┏━┓┏━┓
   ┗┛      ┗┛      ヽ::::::::::::::::::.  \/     ノ      ┗━┛┗━┛
93132人目の素数さん:04/04/14 02:08
非アルキメデス付値に関しての、不等式を論じた本は無いものだろうか?
>>93
その話で盛り上げてください
>>92
>こんな数ヲタ達に囲まれて、ぼかぁ幸せ者だ…

誉めています、念のため…。
96132人目の素数さん:04/04/15 01:10
変数が整数に限定される場合にのみ成り立つ,有用な不等式を挙げてみて下さい.
     言い出したお前が書け!
     ̄ ̄ ̄ ̄ ̄∨ ̄ ̄ ̄ ̄ ̄ ̄ ̄ ̄ ̄

                    ∧         ∧
      / ̄ ̄ ̄ ̄ ̄ ̄\/ ヽ_       / .∧
    /           /   `、⌒ヾ⌒ヽ/  ∧
   /           /  u (.....ノ(....ノ   / ヽ
    l:::::::::.     \,, ,, |          u .:(....ノノ
    |::::::::::   (●)  / ̄ ̄ヽ      ::::::::::::::/`ヽ
   |:::::::::::::::::   \_(___..ノ  u::::::::::::::::::::(....ノノ
    ヽ:::::::::::::::::::.  \/ ヽ  u ::::::::::::::::::::::::::::ノ

                 ∧        ∵ ∧    ☆
                 ζ *      *:/ .∧  :
     / ̄ ̄ ̄ ̄ ̄ `、⌒ヾ⌒ヽω*:ミ///;
    /          \( (.....ノ(....ノ/ヽ χ*
    l:::::::::    \,,    ((从⌒从*(....ノノ    ┏━┓┃┃ ━┳┛
   |::::::::::   (●)   (●/ ̄ヽl ;从 */`ヽ   ┃  ┃   ━━╋━━
   .|:::::::::::::::::  \__ (___..ノ*煤i:(....ノノ      ┃      ┃  ━╋━┓
    ヽ:::::::::::::::::::  \// ・(( ; 〜:  ζ\*      ┃    ━┛    ┃ ┛ 
98132人目の素数さん:04/04/15 09:18
>>92
でもさー包茎でも禿げでもデブでも病気にしろ不等式でも
そこに悩みが有って、それに対してどうして良いのか判らない人達に
解決策を示せるのは何にしろ価値が有る事だと思うなぁ。
9939:04/04/18 03:47
>44
出所は
秋山仁+Peter Frankl 共著「完全攻略 数学オリンピック」日本評論社,p.24-25(1991.11)
ガロアって不等式ヲタ?
>45
値分布を調べた結果、≧7/8と推定しますた。
100132人目の素数さん:04/04/18 18:49
100げとー
ガロアは代数・群論ヲタかと・・・
479
不等式ヲタが保守します。 また教えて下さい。

      キ        //   /::::://O/,|      /
      ュ     / |''''   |::::://O//|     /
      .ッ       \ |‐┐ |::://O/ ノ   ヾ、/
       :       |__」 |/ヾ. /    /
         ヽ /\  ヽ___ノ / . へ、,/
        /  ×    /  { く  /
        く  /_ \   !、.ノ `ー''"
  /\        ''"  //
 | \/、/           ゙′
 |\ /|\ ̄
   \|
103132人目の素数さん:04/05/04 15:57
【問題】(JMO-1997 本選, 第2問)
a,b,c>0 のとき
(b+c-a)^2/{(b+c)^2+a^2} + (c+a-b)^2/{(c+a)^2+b^2} + (a+b-c)^2/{(a+b)^2+c^2} ≧ 3/5.
を示してくださいです。

※ 左辺第1項 ≧ 2-9・(3/25){1+2a/(a+b+c)} らしいYo.
不等式ヲタのAAもアレを流用するか…
口元を変えてみた。

   ┏┓  ┏━━┓        / ̄ ̄ ̄ ̄ ̄ ̄\          .┏━┓┏━┓
 ┏┛┗┓┃┏┓┃        /   不等式ヲタ  \.       ┃  ┃┃  ┃
 ┗┓┏┛┃┗┛┃┏━━━/               ヽ.━━━┓┃  ┃┃  ┃
 ┏┛┗┓┃┏┓┃┃      l:::::::::    \    /   |     ┃┃  ┃┃  ┃
 ┗┓┏┛┗┛┃┃┗━━━|::::::::::   (●)   (●)   |━━━┛┗━┛┗━┛
   ┃┃      ┃┃        |:::::::::::::::::    __      |      ┏━┓┏━┓
   ┗┛      ┗┛      ヽ::::::::::::::::::.  \/     .ノ      ┗━┛┗━┛
106132人目の素数さん:04/05/05 00:14
【問題】 For a,b,c>0,
a/(b+2c) + b/(c+2a) + c/(a+2b) ≧ (a+b+c)^3/{(a+2b)(b+2c)(c+2a)}
≧ (1/3)(a+b+c){1/(b+2c)+1/(c+2a)+1/(a+2b)} ≧ 1.
Equality: a=b=c

※ 中央は Sierpinski(AAH≧GGG≧AHH), 右側は A≧H らしいyo.
>>106 示すべき不等式を X≧Y≧Z≧1 で表す。

[X≧Y]
(左辺)-(右辺) = {a(a-c)^2+b(b-a)^2+c(c-b)^2} ≧ 0
[Y≧Z]
A=b+2c, B=c+2a, C=a+2b とおいて、(A+B+C)^2 ≧ 3(AB+BC+CA) を示す。
(左辺)-(右辺) = (1/2){(A-B)^2+(B-C)^2+(C-A)^2} ≧ 0
[Z≧1]
相加平均≧調和平均、あるいはCauchy-Schwarzの不等式による。

      / ̄ ̄ ̄ ̄ ̄ ̄\
    /   不等式ヲタ  \.  ワクワク
   /   ノ(           ヽ  ゾクゾク
    l::::::::: ^   \  /      |  ブルブル
    |::::::::::   (●)     (●)  |    ハァハァ
   |:::::::::::::::::     __      |     ゼエゼエ
    ヽ:::::::::::::::::::.  \/     ノ
グッジョブと言ってくれ!

 ./ ̄ ̄ ̄\.
 |::::  \ ./ | ワクワク
 |:::: (● (● |
 ヽ:::::::.....∀...ノ
109106:04/05/05 01:12
>107-108
Very good job!!
Sierpinskiは牛刀...
X−Y = {a(a-c)^2+b(b-a)^2+c(c-b)^2}/{(a+2b)(b+2c)(c+2a)} ≧0.
>>109
分母を書き忘れてたか。修正サンクスコ。

 ./ ̄ ̄ ̄\.
 |::::  \ ./ | ワクワク
 |:::: (● (● |
 ヽ:::::::.....∀...ノ
111103:04/05/05 20:19
※ の説明
左辺第1項 = (b+c-a)^2 /{(b+c)^2+a^2} = (3-2x)^2 /{(3-x)^2+x^2}
= 2−9/{(3-x)^2+x^2} ≡ 2-9・f(x).
x=1での接線は、 y = f(1) + f'(1)(x-1) = 1/5 + (2/25)(x-1) = (3/25)(1+2x/3).
(3/25)(1+2x/3)−f(x) = (2/25)f(x)(1+2x)(1-x)^2 ≧ 0.
等号成立は x=1, 3a=a+b+c.
112111:04/05/05 20:25
3a/(a+b+c) =x と置きました。
113132人目の素数さん:04/05/09 12:59
シンプルで易しい問題
a>0,b>0
y=b^x のグラフは下に凸
∴ (1,b)→(x,b^x) の平均変化率はxと共に単調増加。
∴ (b^x-b)/(x-1) > b-1.
【1】
a>1 ⇒ b/(a+b-ab) > b^a > ab+1-a.
a<1 ⇒ b/(a+b-ab) < b^a < ab+1-a.

(a^b)-a, (b^a)-b, (a-1)(b-1) の符号は一致する。
【2】
(a-1)(b-1)>0 ⇒ a^b+b^a > ab+1 > a+b.
(a-1)(b-1)<0 ⇒ a^b+b^a < ab+1 < a+b.
114113:04/05/09 19:29
訂正 スマソ.
【1】
a>1 ⇒ ............ b^a > ab+1-a.
a<1 ⇒ b/(a+b-ab) < b^a < ab+1-a.
>【2】
>(a-1)(b-1)>0 ⇒ a^b+b^a > ab+1 > a+b.
>(a-1)(b-1)<0 ⇒ a^b+b^a < ab+1 < a+b.

この辺が分からない私は、生産ラインに組み込まれる人材ですか?
正の数 a_k に対して、次の不等式を示せ。
Σ[k=1 to n]1/(1+a_k) ≧ n/{1+(Π[k=1 to n]a_k)^(1/n)}

帰納法で解こうと思い、n=2のときは差をとって示しましたが、
そのあとが うまくいきません。おしえてください。
   ___
 ./  ≧ \
 |::::  \ ./ |
 |::::: (● (● | ワクワク
 ヽ::::... .ワ....ノ  
>116
G≡(Π[k=1 to n]a_k)^(1/n) とおくとき、
G>1 ⇒ Σ[k=1 to n] 1/(1+a_k) ≧ n/(1+G)
G<1 ⇒ Σ[k=1 to n] 1/(1+a_k) ≦ n/(1+G)
Equality: a_k=const or G=1.
>116,118
a_k≧1 (k=1 to n) に限定すれば成立するが....
(;゚д゚) ハッ! すみません。
問題文見なおしたら、a_k≧1だった・・・
どうやって次を示すのですか?

G≡(Π[k=1 to n]a_k)^(1/n) とおくとき、
G>1 ⇒ Σ[k=1 to n] 1/(1+a_k) ≧ n/(1+G)
   ___
 ./  ≧ \
 |::ι \ ./ |
 |::::: (● (● | ワクワク
 ヽ::::... .ワ....ノ  
122凡例:04/05/12 20:16
>118,120
a_1=1/10, a_2=2, a_3=5 のとき、左辺 < 1+1/3+1/6 = 3/2 = 右辺 (G=1).
a_1=10, a_2=1/2, a_3=1/5 のとき、左辺 > 0+2/3+5/6 = 3/2 = 右辺 (G=1).
('A`)
>120
G≡(Π[k=1 to n]a_k)^(1/n) とおくとき、
 a_k>1 (k=1〜n) ⇒ 納k=1 to n] 1/(1+a_k) ≧ n/(1+G).
0<a_k<1 (k=1〜n) ⇒ 納k=1 to n] 1/(1+a_k) ≦ n/(1+G).
Equality: a_k=const.
125124:04/05/14 19:37
(証) a_k=G (k=1〜n)のときは、明らかに成立。 a_k≠G (k=1〜m)とし、mに関する帰納法による。
a_m>G, a_{m-1}<G (またはその逆)としても一般性を失わない。
いま a_m, a_{m-1} を、それらの中間にある b_m=G, b_{m-1}=a_m・a_{m-1}/G=p/G で置換えてみる。
b_m + b_{m-1} = a_m + a_{m-1} - (a_m-G)(G-a_{m-1})/G < a_m + a_{m-1} = s.
上記の置換えにより、積pは不変で和sは減少する。
S = 1/(1+a_m) + 1/(1+a_{m-1}) = 1 + (1-p)/(1+s+p) は和sについて単調に増加/減少する。
∴ a_k>1 ⇒ p>1 ⇒ S > 1/(1+b_m) + 1/(1+b_{m-1}).
∴ a_k<1 ⇒ p<1 ⇒ S < 1/(1+b_m) + 1/(1+b_{m-1}).
上記の置換え{a_1,〜,a_m} → {a_1,〜,a_{m-2},b_{m-1},b_m} により mが1つ減少し、Sも減少/増加した。
帰納法の仮定により m-1 に対しては成立しているから、mに対しても成立する。(終)

相加・相乗平均の証明法を使いました...
126132人目の素数さん:04/05/15 00:48
>120
【定理】(Klamkin,1974)
0<a_k<1, e_k>0 (k=1 to n), Σ[k=1 to n]e_k=1, G'=Π[k=1 to n](a_k^e_k) のとき
Σ[k=1 to n] e_k/(1+a_k) ≦ 1/(1+G').

で e_k=1/n (k=1〜n) とおく。(終)

※ 大関信夫・大関清太: 「不等式への招待」 近代数学社(1987) のp.83
>>124-126
   ┏┓  ┏━━┓        / ̄ ̄ ̄ ̄ ̄ ̄\          .┏━┓┏━┓
 ┏┛┗┓┃┏┓┃        /   不等式ヲタ  \.       ┃  ┃┃  ┃
 ┗┓┏┛┃┗┛┃┏━━━/               ヽ.━━━┓┃  ┃┃  ┃
 ┏┛┗┓┃┏┓┃┃      l:::::::::    \    /   |     ┃┃  ┃┃  ┃
 ┗┓┏┛┗┛┃┃┗━━━|::::::::::   (●)   (●)   |━━━┛┗━┛┗━┛
   ┃┃      ┃┃        |:::::::::::::::::    __      |      ┏━┓┏━┓
   ┗┛      ┗┛      ヽ::::::::::::::::::.  \/     .ノ      ┗━┛┗━┛
「数学しりとりスレ 232-233」 より

【inglebyの不等式 】
x>0のとき、{1+x^2+x^4+…+x^(2n)}/{x+x^3+…+x^(2n-1)} ≧ (n+1)/n

【Ky Fan の不等式】
0 < x_k ≦ 1/2 のとき、S = Σ[k=1 to n](x_k) とおくと
Π[k=1 to n]{(x_k)/(1-x_k)} ≦ {S/(n-S)}^n
129124:04/05/15 15:56
Note to [124] & [126]
Inequalities in [124] are equivalent.
∵ 1/(1+a) + 1/{1+(1/a)} = 1, 1/(1+G) + 1/{1+(1/G)} = 1.

[126] is equivalent to:
Let e_k>0 (k=1 to n), Σ[k=1 to n]e_k=1, G'=Π[k=1 to n](a_k^e_k), then
a_k>1 (k=1〜n) ⇒ Σ[k=1 to n] e_k/(1+a_k) ≧ 1/(1+G').
130125:04/05/15 17:09
>128
【Ky Fan】
x_k=S/n (k=1〜n)のときは明らかに成立。 x_k≠S/n (k=1〜m)とし、mに関する帰納法による。
x_m>S/n, x_{m-1}<S/n(またはその逆)としても一般性を失わない。
x_m,x_{m-1} をそれらの間にある y_m=S/n, y_{m-1}=x_m+x_{m-1}-S/n で置換えてみる。
y_m・y_{m-1} - x_m・x_{m-1} = (x_m-S/n)(S/n-x_{m-1})≧0. ∴ 和sは不変で、積pは増加する。
P = [x_m/(1-x_m)]・[x_{m-1}/[1-x_{m-1}]] =p/(1-s+p) は、s≦1では、sについて単調増加。
∴ 上記の置換え(x_1,・・・・,x_m)→(x_1,・・・・,x_{m-2},y_{m-1},y_m) により mが1つ減り Pは増加した。
帰納法の仮定より、m-1については成立するので、mについても成立する。(終)

大関信夫・青柳雅計: 「不等式」 槇書店 p.60,p.128
大関信夫・大関清太: 「不等式への招待」近代科学社(1987) p.88 例題9
   ___
 ./  ≧ \
 |::::  \ ./ |
 |::::: (● (● | ハァハァ、ハァハァ…
 ヽ::::... .ワ....ノ  
>128
【ingleby】
F_n ≡ n[1+x^2+x^4+・・・・+x^(2n)] - (n+1)[x+x^3+・・・・+x^(2n-1)] とおく.
F_1 = [1+x^2] - 2x = (x-1)^2 ≧0.
F_{n+1} - F_n = [1+x^2+・・・・+x^(2n)+(n+1)・x^(2n+2)] - [x+x^3+・・・・+x^(2n+1)+(n+1)・x^(2n+1)]
= (1-x){1+x^2+・・・・+x^(2n) - (n+1)・x^(2n+1)}
= (1-x){1+x^2+・・・・+x^(2n) - (n+1)・x^(2n) + (n+1)(1-x)x^(2n)}
= (1-x){Σ[k=0 to n-1]{x^(2k)-x^(2n)} + (n+1)・(1-x)x^(2n))}
= (1-x){Σ[j=0 to n-1](Σ[k=0 to j]1)・(1-x^2)x^(2j) + (n+1)・(1-x)x^(2n)}
= (1-x)^2{Σ[j=0 to n-1](1+j)・(1+x)x^(2j) + (n+1)・x^(2n)} ≧ 0. (∵すべての係数>0)
∴ xを固定したとき F_{n+1} ≧ F_n ≧ ・・・・・ ≧ F_1 ≧ 0. 等号成立は x=1.(終)
133130,132:04/05/15 23:14
   ┏┓  ┏━━┓        / ̄ ̄ ̄ ̄ ̄ ̄\          .┏━┓┏━┓
 ┏┛┗┓┃┏┓┃        /   不等式ヲタ  \.       ┃  ┃┃  ┃
 ┗┓┏┛┃┗┛┃┏━━━/               ヽ.━━━┓┃  ┃┃  ┃
 ┏┛┗┓┃┏┓┃┃      l:::::::::    \    /   |     ┃┃  ┃┃  ┃
 ┗┓┏┛┗┛┃┃┗━━━|::::::::::   (●)   (●)   |━━━┛┗━┛┗━┛
   ┃┃      ┃┃        |:::::::::::::::::    __      |      ┏━┓┏━┓
   ┗┛      ┗┛      ヽ::::::::::::::::::.  \/     .ノ      ┗━┛┗━┛
 ついでに出しときますた...
134132:04/05/16 01:15
(注) [132]の下から3行目で次を使いますた。
k<n のとき, y^k-y^n = Σ[j=k to n-1] (1-y)y^j
135132:04/05/16 12:24
(注) Gauss の記号 [z] = max{m|z≧m} を使えば、[132]の
F_{n+1} - F_n = (1-x)^2 Σ{j=0 to 2n} [1+(j/2)] x^j
F_n = (1-x)^2 Σ{j=0 to 2n-2} [1+(j/2)][n-(j/2)] x^j
136132人目の素数さん:04/05/16 15:29
【例題7】(Klamkin,1975)
a_i>0 (i=1〜n)のとき, S≡Σ[k=1 to n]a_k とおくと,
Π[i=1 to n] {(1+a_i/S)/(1-a_i/S)} ≧ {(n+1)/(n-1)}^n.
等号成立は a_i=S/n (n=1〜n)のとき (p.85)

【例題10】
a_i≧1 (i=1〜n)のとき, G≡(Π[k=1 to n]a_k)^(1/n) とおくと
Σ[i=1 to n](1+a_i) > G・Σ[i=1 to n] {1+(1/a_i)}.
Σ[1≦i<j≦n](1+a_i)(1+a_j) > G^2・Σ[1≦i<j≦n]{1+(1/a_i)}{1+(1/a_j)}. (p.90)

よろしくおながいしまつ。
137132人目の素数さん:04/05/16 22:51
不等式でげす。

1/p+1/q=1,p>1,a_i≧0,b_i≧0,( i=1,2,3,・・・,n)としまする。

{納i=1→n](a_i)^p}^(1/p){納i=1→n](b_i)^q}^(1/q)≧納i=1→n]a_ib_i

を示せ。

俺にはサパーリ
>>137
これは ヘルダーの不等式かな? ハァハァ
証明は帰納法だったと思う…
   ___
 ./  ≧ \
 |::::  \ ./ |
 |::::: (● (● | ハァハァ、ハァハァ…
 ヽ::::... .ワ....ノ  
【系】でつ。

1/p+1/q+1/r = 1, p,q,r>1, a_i≧0, b_i≧0, c_i≧0 (i=1,2,3,・・・,n)としまする。

{納i=1→n](a_i)^p}^(1/p) {納i=1→n](b_i)^q}^(1/q) {納i=1→n](c_i)^r}^(1/r) ≧ 納i=1→n]a_i・b_i・c_i

俺にはサパーリ
14039:04/05/17 02:31
>40-57
>>39の問題を書いたやつ出て来いや

では報告しまつ。
【問題】 0<a,b,c<1, s=a+b+c のとき,
F(a,b,c) ≡ a/(b+c+1) + b/(c+a+1) + c/(a+b+1) ≧ s/(2s/3+1) ≧ 7/8.

【方針】 さらに中間項を挟みまつ。 a> (s/3) >b として、
F(a,b,c) ≧ F(s/3,a+b-s/3,c) ≧ F(s/3,s/3,s/3)
14139:04/05/17 02:36
F(a,b,c) ≧ F(s/3,a+b-s/3,c) ≧ F(s/3,s/3,s/3)

a=b=c=s/3 なら明らかに成立。 ∴ a> (s/3) >b とする。
(左側) まづ、a,b をそれらの間にある a'=s/3, b'= a+b-(s/3) = (2s/3)-c で置き換えてみまつ。
a'b'= ab + (a- s/3)(s/3 -b) ≧ ab ∴上の置換えで、和は不変で、積は増大する。またこのとき、
F(a,b,c) = a/(s+1-a) + b/(s+1-b) + c/(s+1-c)
= [(s+1)(a+b)-2ab]/[(s+1)(s+1-a-b)+ab] + c/(s+1-c)
= [(s+1)(s-c)-2ab]/[(s+1)(1+c)+ab] + c/(s+1-c)
≧ [(s+1)(s-c)-2a'b']/[(s+1)(1+c)+a'b'] + c/(s+1-c) = F(a',b',c).
により、Fは減少する。

(右側) b',c≠s/3 のとき、再度 置換えを行う: b"=s/3, c'=s/3. 上記と同様にして
F(a',b',c) ≧ F(a',b",c') = F(s/3,s/3,s/3).
が得られますた。(終)

>57
普通の高校生でも分かりそうだな.....(w
やっぱむずかしいなぁ
143132人目の素数さん:04/05/23 21:50
【問題】 IMO-2001 (USA) Problem 2

For any a,b,c>0 and λ≧8, the following inequality holds:

2 ≧ a/√(a^2+λbc) + b/√(b^2+λca) + c/√(c^2+λab) ≧ 3/√(1+λ).

Left equality: abc=0, and right equality: a=b=c.

http://imo.wolfram.com/problemset/index.html


ヒント: r≧4/3 ⇒ x^2 ≦ 1−(4/3r) + (2/3r){1+(x^r)}(x^r), 等号成立は x=1.
144143:04/05/23 22:01
ヒント: λ≧8 ⇒ 1+λx^2 ≦ (1+λ)・{1+2(x^r)}^2, r=(3/2)λ/(1+λ).
   ┏┓  ┏━━┓        / ̄ ̄ ̄ ̄ ̄ ̄\          .┏━┓┏━┓
 ┏┛┗┓┃┏┓┃        /   不等式ヲタ  \.       ┃  ┃┃  ┃
 ┗┓┏┛┃┗┛┃┏━━━/               ヽ.━━━┓┃  ┃┃  ┃
 ┏┛┗┓┃┏┓┃┃      l:::::::::    \    /   |     ┃┃  ┃┃  ┃
 ┗┓┏┛┗┛┃┃┗━━━|::::::::::   (●)   (●)   |━━━┛┗━┛┗━┛
   ┃┃      ┃┃        |:::::::::::::::::    __      |      ┏━┓┏━┓
   ┗┛      ┗┛      ヽ::::::::::::::::::.  \/     .ノ      ┗━┛┗━┛
ヒントがあるから楽勝だなって思ってたけど、難しいですね
_ト ̄|○
147143:04/05/24 07:35
ヒント: 1/√(1+λx^2) ≦ 1/{(√λ)x}.
. x<1 ⇒ 1/√(1+λx^2) ≦ 1-{1-1/(√λ)}x.


148147:04/05/24 12:10
まちがい、すまそ。
. x<1 ⇒ 1/√(1+λx^2) ≦ 1-{1-1/(√λ)}x^2. (下に凸だから)
149143:04/05/24 19:49
【問題】(retold)
For any x,y,z>0, xyz=1 and λ≧8, the following inequality holds:
2 > 1/√(1+λx^2) + 1/√(1+λy^2) + 1/√(1+λz^2) ≧ 3/√(1+λ).
Right equality: x=y=z=1.

変わり映えしませんが、よろしくおながいします...
x=bc/(a^2), y=ca/(b^2), z=ab/(c^2)とおくと、
xyz=1をみたし、a,b,c>0より x,y,z>0 であって、
149 ⇔ 143

   ___
 ./  ≧ \
 |::::  \ ./ |
 |::::: (● (● | ハァハァ、ハァハァ…
 ヽ::::... .ワ....ノ  
>150
確かに、その方がシンプルでいいな...2乗する意味もないし..
X=bc/(a^2), Y=ca/(b^2), Z=ab/(c^2) とでもおこうか。
>150の形で証明できれば、簡単そうですよね…
まだできてないけど _ト ̄|○
さくらスレにあったよ。

109 :PrinceMathematician◇ :04/05/27 12:02
>91
x_k≧0 (k=1,2,・・・,m)のとき・・・(中略)

F(n)≡(1/m)Σ[1≦k≦m] (x_k)^n とおく.
(補題) bc≧0 ⇒ F(a)・F(a+b+c) ≧ F(a+b)・F(a+c).

(系1) F(n-1)・F(n+1) ≧ {F(n)}^2.

(系2) {F(n)}^(1/n) はnに関して単調増加.
154132人目の素数さん:04/05/27 19:48
まだ未解決不等式あるよーガンガレ不等式ヲタ
>152
xyz=1のとき、 x,y≦1≦z または x≦1≦y,z としても一般性を失わないYo
応援&新ネタ 感謝です。
あちこち調べながら、>>149をやってみます
   ___
 ./  ≧ \
 |::::  \ ./ |
 |::::: (● (● | ハァハァ、ハァハァ…
 ヽ::::... .ワ....ノ  
>>149 【問題】(retold)
For any x,y,z>0, xyz=1 and λ≧8, the following inequality holds:
2 > 1/√(1+λx) + 1/√(1+λy) + 1/√(1+λz) ≧ 3/√(1+λ).
Right equality: x=y=z=1.

(上限の証明)
λ≧8, t>0 のとき 1/√(1+8t) ≧ 1/√(1+λt) だから、
  x,y,z>0, xyz=1 に対して
  2 > 1/√(1+8x^2) + 1/√(1+8y^2) + 1/√(1+8z^2)
を示せばよい。この証明は、運よくコレクションにあった。

t>0 において f(t) = (1+8t)^(-1/2) とおくと、
f'(t) = -4t(1+8t)^(-3/2) < 0 だから単調減少する。
対称式かつ xyz=1 だから、次の2つの場合を考えればよい。

(i) 0<x≦1≦y≦z のとき
f(x)+f(y)+f(z) < f(0)+f(1)+f(1) = 5/3 < 2

(ii) 0<x≦y≦1≦z のとき
0<t≦1 において f(t) ≦ 1-(2t/3) が成り立つ。
これは g(t) = (1+8t){1-(2t/3)}^2-1 の増減を調べれば分かる。
1≦t では f(t) < 1/√(8t) を使うと

f(x)+f(y)+f(z) < 1-(2x/3) + 1-(2y/3) + 1/√(8z)
  = 2 - 2(x+y)/3 + 1/√(8z) ≦ 2 - 4√(xy)/3 + 1/√(8z)
  = 2 - 2( 2/3 - 1/√(8z) )/√z < 2

自分では思いつけません…
>137
適当に規格化して、Σ[i=1→n](a_i)^p =1, Σ[i=1→n](b_i)^q =1 とする。

(1/p)+(1/q) =1 より (p-1)(q-1)=1.
y=x^(p-1) ⇔ x=y^(q-1) だからヤングの不等式より
a・b ≦ ∫[x=0,a]x^(p-1)・dx + ∫[y=0,b]y^(q-1)・dy = (1/p)(a^p) + (1/q)(b^q).
∴ Σ[i=1→n]a_i・b_i ≦ (1/p)Σ[i=1→n](a_i)^p + (1/q)Σ[i=1→n](b_i)^q = (1/p)+(1/q) =1.
>>157
最小値のほうは?
>>149の最小値のほうは、相加相乗を使うと
3/√(1+λ)よりも小さくなってしまって失敗。
ムスカしいなぁ…
   ___
 ./  ≧ \
 |::::  \ ./ |
 |::::: (● (● | ウーン、ウーン…
 ヽ::::... .ワ....ノ  
>160
[144]使え
162161:04/05/30 15:34
【144】
λ≧8 ⇒ 1+λx^2 ≦ (1+λ)・{1+2(x^r)}^2, r=(3/2)λ/(1+λ).

(証) 相乗平均≦相加平均 より x^2 ≦ 1−(4/3r) + (2/3r){1+(x^r)}(x^r).
これにλを乗じて1を加える。
右辺を平方完成する → rが定まる。
>(証) 相乗平均≦相加平均 より x^2 ≦ 1−(4/3r) + (2/3r){1+(x^r)}(x^r).

ここが分かりません
どうか この愚か者に説明してください
164162:04/05/31 21:31
[163] は A,B,C,q,r,s,x≧0, A+B+C=1 のとき
x^(Aq+Br+Cs) ≦ A(x^q) + B(x^r) + C(x^s).
とほぼ同じ。これをどう示すか。。。
> x^(Aq+Br+Cs) ≦ A(x^q) + B(x^r) + C(x^s).

下に凸な関数 f(t)=x^t に関して、Jensenの不等式から得られるけど
上式が[163]の式とほぼ同じってのが分かりません。
   ___
 ./  ≧ \
 |::::  \ ./ |
 |::::: (● (● | ウーン、ウーン…
 ヽ::::... .ワ....ノ  
166164:04/06/02 22:18
>165
A=1-(4/3r), B=C=(2/3r), q=0, s=2r とおきますた。
167132人目の素数さん:04/06/02 22:21
【問題】 0<a<x,y,z<b のとき、
(x+y+z)(1/x + 1/y + 1/z)
の取りうる値の範囲を求めよ。

さくらスレ145
http://science3.2ch.net/test/read.cgi/math/1085311797/465

どんな難しい問題も・・・
http://science3.2ch.net/test/read.cgi/math/1081052206/151
>>167
たしか一般化されたのが、幾つかあったような…
   ___
 ./  ≧ \
 |::::  \ ./ |
 |::::: (● (● | ウーン、ウーン…
 ヽ::::... .ワ....ノ  
1977 USAMO 問5 (解答あり)
http://www.kalva.demon.co.uk/usa/usa77.html

1978 ASU 問13 (解答なし)
http://www.kalva.demon.co.uk/soviet/sov78.html
   ___
 ./  ≧ \
 |::::  \ ./ | この不等式ヲタのコレクションに2つあった。
 |::::: (● (● | ASUの方の模範解答をキボンヌ。 ハァハァ…
 ヽ::::... .ワ....ノ  グッジョブですか?
(下限)
F(x,y,z) = (x+y+z)(1/x+1/y+1/z) = 3 + (x/y+y/x) + (y/z+z/y) + (z/x+x/z)
= 9 + (x/y-2+y/x) + (y/z-2+z/y) + (z/x-2+x/z)
= 9 + {(x-y)^2}/xy + {(y-z)^2}/yz + {(z-x)^2}/zx ≧ 9. (Cauchy)
(上限) x/y+y/x はx/y=1のとき最小で、両側でx/yに関して単調。すなわち、1から遠ざかるほど増加する。
a<x≦y≦z<bとすると、
F(x,y,z) ≦ F(a,y,b) = 3 + (a/y+y/a) + (y/b+b/y) + (b/a+a/b)
= 3 + (b/a+a/b) + {(a+b)/ab}(y+ab/y) = 1 + (b/a+2+a/b) + {(a+b)/ab}{a+b-(b-y)(y-a)/y}
≦ 1 + (b+a)^2/ab + {(a+b)^2}/ab = 1 + 2{(b+a)^2}/ab.
   ___
 ./  ≧ \
 |::::  \ ./ |
 |::::: (● (● | みんな グッジョブ!
 ヽ::::... .ワ....ノ    n  
 ̄ ̄   \    ( E)
フ     /ヽ ヽ_//
>>166
なるほど
>169
1978 ASU Problem 13
USAMO の解答から、Fが最大となるのはn個の変数がすべて a or b のとき。
あとは2組(p,n-p)に分けるだけ。
F(a・・・a,b・・・b)= n^2 + p(n-p){(b-a)^2}/ab だから最大は p=[n/2] or p=[(n+1)/2].
n:even ⇒ F≦n^2・{(a+b)^2}/4ab, n:odd ⇒ F≦1+(n^2-1){(a+b)^2}/4ab, n=3 ⇒ [171].
[149]の最小値の証明について、>>162-166より
 x^2 ≦ 1−(4/3r) + (2/3r){1+(x^r)}(x^r)
ここで r=(3/2)λ/(1+λ) を代入すると
 1+λx^2 ≦ {(1+λ)・{1+2(x^r)}^2}/9 ≦ (1+λ)・{1+2(x^r)}^2

したがって
1/(1+λx^2) + 1/(1+λy^2) + 1/(1+λz^2)
 ≧ 1/(1+λ)・(1/{1+2(x^r)} + 1/{1+2(y^r)} + 1/{1+2(z^r)})

ここまでは分かりましたが、最後に
1/{1+2(x^r)} + 1/{1+2(y^r)} + 1/{1+2(z^r)} ≧ 3
を示すには どうすればよいのでしょうか?
176149:04/06/04 12:01
2 > 1/√(1+λx^2) + 1/√(1+λy^2) + 1/√(1+λz^2) ≧ 3/√(1+λ).
>>175の計算で 1+λx^2 ≦ {(1+λ)・{1+2(x^r)}^2}/9 の後、

1/(1+λx^2) + 1/(1+λy^2) + 1/(1+λz^2)
 ≧ 3/(1+λ)・( 1/{1+2(x^r)} + 1/{1+2(y^r)} + 1/{1+2(z^r)} )

だから、右辺が ≧ 3/√(1+λ) となるには 次を示したらいいんですよねえ?

1/{1+2(x^r)} + 1/{1+2(y^r)} + 1/{1+2(z^r)} ≧ 1
178175:04/06/04 20:30
>177
漏れの機種では"√"が表示されないか.....と思ったら1カ所出てるYo.

あとは x,y,z を a,b,c に戻して, 相乗≦相加 で (終).
すみません、もう少し詳しく教えてください。
x,y,z を a,b,c に戻さないと、相乗≦相加 できないのですか?
戻して相加相乗しましたが、やっぱり分かりません

a^(2r)/{a^(2r)+2(bc)^r} + b^(2r)/{b^(2r)+2(ca)^r} + c^(2r)/{c^(2r)+2(ab)^r}
 ≧ 3{(abc)^(2r)/({a^(2r)+2(bc)^r}{b^(2r)+2(ca)^r}{c^(2r)+2(ab)^r})}^(1/3)
>>178
わざわざ x,y,z を a,b,c に戻してから 相乗≦相加 で (終) ということは
a,b,c の式に戻してから、式を何かに変形してからじゃないと
うまくいかないということなのでしょうね?
降参ですから、もったいぶらずに教えて下さいYo.
x,y,zのままでも やれそうに感じて やってみたら、
いつの間にか符号が逆になってしまった。
182178:04/06/07 12:06
>180
a/√(a^2+λbc) + b/√(b^2+λca) + c/√(c^2+λab)
= 1/√(1+λx^2) + 1/√(1+λy^2) + 1/√(1+λz^2)
≧ {3/√(1+λ)}[1/{1+2(x^r)} +/{1+2(y^r)}+1/{1+2(z^r)}]
= {3/(1+λ)}[(a^r)/{a^r+2(bc)^(r/2)}+(b^r)/{b^r+2(ca)^(r/2)}+(c^r)/{c^r+2(ab)^(r/2)}]
≧ {3/√(1+λ)}[(a^r)/{a^r+(b^r+c^r)}+(b^r)/{b^r+(c^r+a^r)}+(c^r)/{c^r+(a^r+b^r)}]
= {3/√(1+λ)}.
ここに x=(√bc)/a, y=(√ca)/b, z=(√ab)/c.
(*゚∀゚)=3 ウヒョーッ!
しまった、そこで使うのか!
ハァハァ、すばらすぃ!
>178
おかげさまで、やっと理解できました。
ありがとうございます。
【問題】 正の実数a,b,cに対し、次を示せ。
(a^a)(b^b)(c^c) ≧ (abc)^{(a+b+c)/3}
   ___
 ./  ≧ \
 |::::  \ ./ |
 |::::: (● (● | 既出?
 ヽ::::... .ワ....ノ  
186132人目の素数さん:04/06/09 22:32
>>185
勃起してきた その不等式
   ___
 ./  不 \
 |::::  \ ./ |
 |::::: (≧ (≦ | 
 ヽ::::... .∩....ノ  
   , -‐−-、  ヽ∧∧∧ //  |
.  /////_ハ ヽ< 釣れた!> ハ
  レ//j け ,fjlリ / ∨∨V ヽ  h. ゚l;←不等式に魅入られた数ヲタ>>186
 ハイイト、"ヮノハ     //   |::: j  。
  /⌒ヽヾ'リ、     //     ヾ、≦ '
. {   j`ー' ハ      // ヽ∧∧∧∧∧∧∧∧/
  k〜'l   レヘ.   ,r'ス < さぁ、君も今日から>
  | ヽ \ ト、 ヽ-kヾソ <  不等式ヲタだ! >
.  l  \ `ー‐ゝ-〈/´   / ∨∨∨∨∨∨∨∨ヽ
  l     `ー-、___ノ
  ハ   ´ ̄` 〈/‐-、
>>185
その証明がまた(´д`;)ハァハァ
一粒で2度美味しい(´д`;)ハァハァ
   ___
 ./  ≧ \ 【>>185】 正の実数a,b,cに対し、
 |::::  \ ./ |  (a^a)(b^b)(c^c) ≧ (abc)^{(a+b+c)/3}
 |::::: (● (● | 
 ヽ::::... .ワ....ノ  ハァハァ、ハァハァ…

対称性から a≧b≧c>0 としてよい。

a^(3a)*b^(3b)*c^(3c) /(abc)^(a+b+c)
 = (a/b)^(a-b)*(a/c)^(a-c)*(b/c)^(b-c) ≧ 1

等号成立条件は a=b=c
>189

> 対称性から a≧b≧c>0 としてよい。

は不要?  x>0,y>0 ⇒ (x/y)^(x-y)≧1, 等号はx=y.
ハッ!そうですね。
ってことは、こう書けばよかったんだ。

a^(3a)*b^(3b)*c^(3c) /(abc)^(a+b+c)
 = (a/b)^(a-b)*(b/c)^(b-c)*(c/a)^(c-a) ≧ 1

         ∧_∧
         (´Д` )   死んでお詫びを…
         /  y/  ヽ      
    Σ(m)二フ ⊂[_ノ
        (ノノノ | | | l )
    ̄ ̄ ̄ ̄ ̄ ̄ ̄ ̄ ̄ ̄
192190:04/06/10 17:42
>191
つまり、a_k>0、 (a_1+a_2+・・・・+a_n)/n=A とおくと、
(a_1^a_1)(a_2^a_2)・・・・(a_n^a_n)/{(a_1・a_2・・・・a_n)^A} = Π[i=1 to n](a_i)^(a_i-A)
= Π[i=1 to n] Π[j=1 to n] (a_i)^{(a_i-a_j)/n}
= Π[1≦i,j≦n] (a_i/a_j)^{(a_i-a_j)/n} ≧1 だな。
  ∧_∧
  ( ;´∀`) ハァハァ…
  人 Y /
 ( ヽ し
 (_)_)
簡単だがネタを。何通りの証明があるかな?
正の数a,b,cに対して a^4 + b^4 + c^4 ≧ abc(a+b+c) を示せ。
195KingMathematician ◆5lHaaEvFNc :04/06/15 15:11
[>>194]について、
普通に思いつくのが、解析学的な方法。
abc=0のときは、自明である。
以下、abc≠0とする。
a,b,cを-a,-b,-cに変えても左辺と右辺は変わらない。
だから、c>0を仮定してもよい。
a,b,cをka,kb,kc(k>0)にしても不等号は変わらない。
だから、c=1を仮定してよい。
f(a,b):=a^4+b^4-a*b*(a+b+1)
とする。
∂_{a}(f(a,b))=4a^3-2ab-b(b+1)
∂_{a}(f(a,b))=0をaについて解くと、大変なことに…。
a^4+a^4+b^4+c^4>=4a^2bc
b,cも同様にして、加えると
4(a^4+b^4+c^4)>=4abc(a+b+c)
197KingMathematician ◆5lHaaEvFNc :04/06/15 15:39
a^4+b^4-ab(a+b+1)
=(a+b)^4-4a^3b-4b^3a-6a^2b^2-ab(a+b+1)
=(a+b)^4-4ab(a^2+b^2)-6(ab)^2-ab(a+b+1)
=(a+b)^4-4ab((a+b)^2-2ab)-6(ab)^2-ab(a+b+1)
そして、(a+b)^2-4ab>=0より、ab<=(a+b)^2/4
さて、x=a+b,y=abとすると、
2y^2+(-4x^2-x-1)y+x^4,y<=x^2/4である。
平方完成(?)すると、
2(y-(4x^2+x+1)/4)^2+x^4-(4x^2+x+1)^2/8となる。
x^2/4<=(4x^2+x+1)/4は容易に分かる。
だから、y=x^2/4のとき、式が非負になればよい。
しかし…。
kingって、結局できないのか。たいしたことないな。
分かってから書き込め! ウザイ!
199KingMathematician ◆5lHaaEvFNc :04/06/15 16:48
失敗のデータは、本当は成功のデータと同じくらい重要なのだ。
失敗のデータは、ここで行き止まりだという情報を含んでいるからだ。
がんばれKING!
kingは荒らしてるだけか…、クズめ!
不等式を制する者は解析を制す。
203KingMathematician ◆5lHaaEvFNc :04/06/15 22:42
だれが荒らしていると?
得るもののないレスが多い。
とくにキングマスかきのはな・・・
ここ10レスで >>196だけか。
糞レスが多すぎ!
>194 再掲
簡単だがネタを。何通りの証明があるかな?

正の数a,b,cに対して、次を示せ。
a^4 + b^4 + c^4 ≧ abc(a+b+c)
   ___ 
 ./  ≧ \
 |::::  \ ./ | 6通りかな
 |::::: (● (● | ハァハァ
 ヽ::::... .ワ....ノ
(解1) 相加相乗平均による解法  (by 196 さん)
a^4+a^4+b^4+c^4 ≧ 4a^2bc
a^4+b^4+b^4+c^4 ≧ 4ab^2c
a^4+b^4+c^4+c^4 ≧ 4abc^2  を辺々加える。

(解2) 相加相乗平均による解法  (その2)
a^4+b^4+c^4
= (a^4+b^4)/2 + (b^4+c^4)/2 + (c^4+a^4)/2
≧ a^2b^2 + b^2c^2 + c^2a^2
= (a^2b^2+b^2c^2)/2 + (b^2c^2+c^2a^2)/2 + (c^2a^2+a^2b^2)/2
≧ab^2c + abc^2 + a^2bc
= abc(a+b+c)
(解3) Cauchy-Schwarzの不等式と相加相乗平均による解法
 (1^2+1^2+1^2)(a^4+b^4+c^4) ≧ (a^2+b^2+c^2)^2
 (1^2+1^2+1^2)(a^2+b^2+c^2) ≧ (a+b+c)^2
 (a+b+c)/3 ≧ (abc)^(1/3)
a^4+b^4+c^4
≧(a^2+b^2+c^2)^2/3
≧{(a+b+c)/3}^3*(a+b+c)
≧abc(a+b+c)

(解4) Jensenの不等式(凸不等式)と相加相乗平均による解法
 f(x)=x^4は下に凸だから、{f(a)+f(b)+f(c)}/3 ≧ f((a+b+c)/3)
 (a+b+c)/3 ≧ (abc)^(1/3)
(a^4+b^4+c^4)/3
≧((a+b+c)/3)^3*(a+b+c)/3
≧abc(a+b+c)/3
(解5) Cebysevの不等式と相加相乗平均による解法
(a^4+b^4+c^4)/3
≧(a^3+b^3+c^3)/3*(a+b+c)/3
≧abc(a+b+c)/3

(解6) 差をとる。
(左辺)―(右辺)
= {a^2+(b+c)^2}(b-c)^2 + {b^2+(c+a)^2}(c-a)^2 + {c^2+(a+b)^2}(a-b)^2
≧0
   ___ 
 ./  ≧ \
 |::::  \ ./ | >>196さんの証明が一番気に入ってる。
 |::::: (● (● | ハァハァ、ン〜モウ タマランッ!
 ヽ::::... .ワ....ノ
さすが不等式ヲタ
そこにシビれるあこがれる〜ぅ!
>>195
結局両辺を c^4 で割ってるだけだろ。
213KingMathematician ◆5lHaaEvFNc :04/06/16 22:52
どうもおかしいと思ったら、1を足し忘れていた。
214132人目の素数さん:04/06/16 22:58
kingは、ただの荒らし

           放置推奨


215linear PDE ◆O5M8Y2WWjk :04/06/16 22:59
変身忍者嵐
居直り強盗
このスレも とうとう荒らしに目をつけられたか…
>206-210
【定理】a_i>0 のとき、Σ[i=1,n] (a_i)^(n+1) ≧ G・Σ[i=1,n] a_i.
ここに G≡Π[i=1,n] a_i.

(解1)相加相乗平均による方法 (by [196])
[(a_j)^(n+1) +Σ[i=1,n] (a_i)^(n+1)]/(n+1) ≧ a_j・Π[i=1,n]a_i = G・a_j
j=1,・・・,n を辺辺加えて
Σ[i=1,n] (a_i)^(n+1) ≧ G・Σ[i=1,n] a_i.

(解4) Jensenの不等式(凸不等式)より、
f(x)=x^(n+1) は下に凸だから、{Σf(a_i)}/n ≧ f({Σa_i}/n).
相加相乗平均より、(Σa_i)/n ≧ G^(1/n).
∴{Σ a_i^(n+1)}/n ≧ {(Σa_i)/n}^n ・(Σa_i)/n ≧ G・(Σa_i)/n.


(解5)Chebyshev不等式より
(1/n)Σ[i=1,n] (a_i)^(n+1) ≧ (1/n)Σ[i=1,n] (a_i)^n ・ (1/n)Σ[i=1,n] a_i
相加相乗平均より、(1/n)Σ[i=1,n] (a_i)^n ≧ G.
∴(1/n)Σ[i=1,n] (a_i)^(n+1) ≧G・(1/n)Σ[i=1,n] a_i.


(解6)差をとる方法。
(左辺)−(右辺)
= Σ[i=1,n]{(a_i)^(n+1)−G・a_i} = Σ[i=1,n]a_i・{(a_i)^n−G}
= Σ[i=1,n](a_i-g)・{(a_i)^n−G} + g・Σ[i=1,n]{(a_i)^n−G}
= Σ[i=1,n](a_i-g)・{(a_i)^n−G} + ng(A−G).
ここに、g=G^(1/n), A=(1/n)Σ[i=1,n] a_i.
神降臨!
   ┏┓  ┏━━┓        / ̄ ̄ ̄ ̄ ̄ ̄\          .┏━┓┏━┓
 ┏┛┗┓┃┏┓┃        /   不等式ヲタ  \.       ┃  ┃┃  ┃
 ┗┓┏┛┃┗┛┃┏━━━/               ヽ.━━━┓┃  ┃┃  ┃
 ┏┛┗┓┃┏┓┃┃      l:::::::::    \    /   |     ┃┃  ┃┃  ┃
 ┗┓┏┛┗┛┃┃┗━━━|::::::::::   (●)   (●)   |━━━┛┗━┛┗━┛
   ┃┃      ┃┃        |:::::::::::::::::    __      |      ┏━┓┏━┓
   ┗┛      ┗┛      ヽ::::::::::::::::::.  \/     .ノ      ┗━┛┗━┛
>218 の末尾
= Σ[i=1,n](a_i-g)・{(a_i)^n−g^n} + ng(A−G) ≧ 0.
ここに、g=G^(1/n), A=(1/n)Σ[i=1,n](a_i)^n.
 |  |- 20
 |  |
 |  |- 10
 |  |
 |  |- 0
 |  |
 |  |- -10
 |  |
 |  |- -20
 |  |
 |  |- -30  ∧_∧   / ̄ ̄ ̄ ̄ ̄ ̄
 |┃|       (´∀` ) < このスレ寒すぎ
 |┃|       (    )   \______
. (●)     | | |
   ̄     (_(__)
だが、それがいい。
223132人目の素数さん:04/06/20 21:39
nを正の整数とするとき、次の不等式を示せ。

√(nπ) < {(2n)!!}/{(2n-1)!!} < √{(n+0.5)π}
>223
偶然だが昨日、次の不等式を証明したよ。
1/(2n+1) < {(2n-1)!!}/{(2n)!!} < 1/√(2n+1)
   ___ 
 ./  ≧ \
 |::::  \ ./ | 
 |::::: (● (● | ハァハァ
 ヽ::::... .ワ....ノ
>>196,206
東北学院大・経 (1997)か?
↓にもあるよ

安田亨:「入試数学 伝説の良問100」講談社ブルーバックスB-1407 (2003.4) \1155
(2n)!!/(2n-1)!!=√πΓ(n+1)/Γ(n+0.5)だから、各辺logとって整理すると示すべき不等式は

log(n)<(f(n+1)-f(n+0.5))/0.5<log(n+0.5) ただし、f(x)=log(Γ(x))

あとは、log(n)=f(n+1)-f(n), log(n+0.5)=f(n+1.5)-f(n+0.5), fは凸関数であることに注意すればよい
227132人目の素数さん:04/06/21 21:58
【問題】 0<a<π/4、0<b<π/4 であるa,bについて下の不等式が成立することを証明せよ。

√{tan(a)・tan(b)} ≦ tan((a+b)/2) ≦ {tan(a)+tan(b)}/2

分かスレ172
http://science3.2ch.net/test/read.cgi/math/1087644616/381
>>227
自明すぎてつまらん.
>228
y=ln(tan(x)) は上に凸、y=tan(x) は下に凸。
>227
Right = sin(a+b)/[2cos(a)cos(b)] = Middle/[2cos(a)cos(b)]{1+cos(a+b)}.
(Right - Middle)・[2cos(a)cos(b)] = Middle・[1+cos(a+b)-2cos(a)cos(b)] = Middle・[1-cos(a-b)] ≧0.

Left = tan(a+b)/2[1-tan(a)tan(b)].
Middle = tan(a+b)/2[1-tan{(a+b)/2}^2].
Middle^2 - Left^2 = 1 - 2tan{(a+b)/2}/tan(a+b) - [1 - {tan(a)+tan(b)}/tan(a+b)]
= {2/tan(a+b)}(Right - Middle).
231132人目の素数さん:04/06/22 21:13
>>223
∫(0→π/2)(sin x)^n dx を評価すれば出る。
>231
上式を I_n とおくと、I_n = [(n-1)/n] I_{n-2}.
∴ I_{2m+1} = (2m)!!/(2m+1)!!, I_{2m} = [(2m-1)!!/(2m)!!](π/2)
I_0 = π/2, I_1=1, I_2=π/4, I_3=2/3, ・・・ で単調減少
>>227
昔の京大の問題だから調べてね
234 ◆MC1Z7pcz5k :04/06/23 01:22
>>227 >>233
'91 前期 理 4番
の問題ですね。
結構難易度の高い問題だったはずです。
とくに, 左側の不等式は難問です。
>>234
左側の不等式は,>>229の書いてるとおり,logとればただの凸不等式.
難問には思えないんだけどなあ.
236230:04/06/24 11:57
3,4行目に誤り、すまそ。↓に訂正

tan(x)・tan(y) = 1 - {tan(x)+tan(y)}/tan(x+y)
ネタを幾つか仕入れてきたので、
まずは自分で解いてみてから書き込みます。

   , -‐−-、  ヽ∧∧∧ //  |
.  /////_ハ ヽ< 釣るぞ!> ハ
  レ//j け ,fjlリ / ∨∨V ヽ  h. ゚l;←不等式に魅入られた私
 ハイイト、"ヮノハ     //   |::: j  。
  /⌒ヽヾ'リ、     //     ヾ、≦ '
. {   j`ー' ハ      // ヽ∧∧∧∧∧∧∧∧/
  k〜'l   レヘ.   ,r'ス < さぁ、君も今日から>
  | ヽ \ ト、 ヽ-kヾソ <  不等式ヲタだ! >
.  l  \ `ー‐ゝ-〈/´   / ∨∨∨∨∨∨∨∨ヽ
  l     `ー-、___ノ
  ハ   ´ ̄` 〈/‐-、
お勧めトリップ集
KingOfKingMathematicianの後に付けるのがおしゃれ。

H06dWILLhA : #/{\@%YwX
H06djy9xBA : #SgHdO'H%
H06dYXOYLA : #*「A@?NVF
H06dhKnt9A : #[Aシsudセl
H06dWifa1A : #{SfbN(6ヲ
H06dyzvgzA : #QAiEシEp-   ←使用中
239132人目の素数さん:04/06/29 02:53
あげ
非負整数x_kに対し
(x_1)! (x_2)!…(x_n)! ≧ [ { (x_1+ … +x_n)/n }! ]^n
右辺の[a]はガウス記号とする。
すまん、書き間違えた。

非負整数x_kに対し
(x_1)! (x_2)!…(x_n)! ≧ ([(x_1+ … +x_n)/n]!)^n

右辺は相加平均にガウス記号つけたものの階乗のn乗です
x≧1でlogΓ(x)は単調増大凸関数だから明らか。
243132人目の素数さん:04/06/29 13:40
> x≧1でlogΓ(x)は単調増大凸関数だから明らか。
x≧2の間違いだな。
まあ、左辺は1より小さくならないから、
明らかであることに変わりはないけど。
244132人目の素数さん:04/06/29 22:07
【問題】 次式を示してくださいです。。。
-3(√3)/2 ≦ sin(x) + sin(y) + sin(x+y) ≦ 3(√3)/2

さくらスレ146
http://science3.2ch.net/test/read.cgi/math/1087307114/461

面積最大の三角形
http://science3.2ch.net/test/read.cgi/math/1041437063/198
周期2πだから、|x|,|y|≦πで考えれば十分。
sin(x) + sin(y) + sin(x+y) = 4sin((x+y)/2)cos(x/2)cos(y/2) と変形できるから、
(x+y)/2の正負で場合分けして、-log(cos(x))の凸不等式を使えばよい。
  どきどき   |      マダー?  マダー?
       , -┴‐-、゚ 。
     /´ 237 ∠}ヽ  ∧_∧ ∧_∧ ∧_∧
      |   u ノ))))( ・∀・)( ´∀`)( ´・ω・)  
      | u   d!!!l   ( ∪ ∪( ∪ ∪ ( ∪ ∪
   f^iノ   u リノ   と__)__)旦_)__)旦_)__) 旦
    「((((( (((((ト、
   |  i######| }
   `ー'l######レ' 。oO(ど、どしよ……)
     ノ#####〈
>245
sin(x) + sin(y) + sin(x+y) = 2sin{(x+y)/2}[cos{(x-y)/2} + cos{(x+y)/2}],
-1 + cos{(x+y)/2} ≦ cos{(x-y)/2} + cos{(x+y)/2} ≦ 1 + cos{(x+y)/2}.
-2・sin{(x+y)/4}^2 ≦ cos{(x-y)/2} + cos{(x+y)/2} ≦ 2・cos{(x+y)/4)}^2.
までは出るが...
>>247
0≦(x+y)/2≦πなら、sin((x+y)/2)≧0で、sin((x+y)/2)=cos(π/2-(x+y)/2)
-π≦(x+y)/2≦0なら、sin((x+y)/2)≦0で、sin((x+y)/2)=-cos(-π/2-(x+y)/2)
249UltraMagic ◆NzF73DOPHc :04/06/30 22:02
どうでもいいけど、「け ,fj」って何?
どうして荒らすのかなぁ > Q.man
-log(cos(x)) (|x|≦π/2)の凸不等式をつかう。

(x+y)/2≧0のとき
0≦4sin((x+y)/2)cos(x/2)cos(y/2)=4cos(π/2-(x+y)/2)cos(x/2)cos(y/2)≦4(cos(π/6))^3=3(√3)/2

(x+y)/2<0のとき
0≧4sin((x+y)/2)cos(x/2)cos(y/2)=-4cos(-π/2-(x+y)/2)cos(x/2)cos(y/2)≧-4(cos(-π/6))^3=-3(√3)/2

∴-3(√3)/2≦sin(x)+sin(y)+sin(x+y)≦3(√3)/2
新ネタ投下 ハァハァ

自然数nに対して、次の不等式を示せ。
(n!)e^n > {n+(1/2)}^{n+(1/2)}
>>206-210
(解7)
 2{(a^2+b^2+c^2)-(ab+bc+ca)} = (a-b)^2+(b-c)^2+(c-a)^2 ≧ 0
より a^2+b^2+c^2 ≧ ab+bc+ca が成り立つ。
これを2回繰り返す。
 a^4+b^4+c^4 ≧ (ab)^2+(bc)^2+(ca)^2 ≧ abc(a+b+c)
>>252
log(x)と接線を考えて積分で評価したら、ちょっとだけいい評価が出た

(n!)e^n > (√2){n+(1/2)}^{n+(1/2)}
   ___
 ./  不 \
 |::::  \ ./ |
 |::::: (≧ (≦ | ハァハァ
 ヽ::::... .∩....ノ  
実数a,b,c,d,eに対して、次の不等式を示せ。
1 < a/(a+b) + b/(b+c) + c/(c+d) + d/(d+e) + e/(e+a) < 4

まぁだまだ 逝くよぉ〜
n個の正の数 a_1,a_2,…,a_n に対して、
基本対称式(k個ずつの積の和)を s_k で表すとき、次を示せ。
 (s_k)(s_{n-k}) ≧ (s_n)C[n,k]^2

不等式な上に、nCrまで…
   ___
 ./  不 \
 |::::  \ ./ |
 |::::: (≧ (≦ | ハァハァ
 ヽ::::... .∩....ノ  
258UltraMagic ◆NzF73DOPHc :04/07/01 15:37
Re:>256 そういうのいけない。
a=1,b=-.999,c=10,d=1,e=1
範囲を正の数に限定すれば大丈夫だ。
う、おっしゃるとおりで。
正の実数でした。
260132人目の素数さん:04/07/01 16:36
任意の実数で成り立つ(*´д`*)ハァハァする不等式はないの?
261UltraMagic ◆NzF73DOPHc :04/07/01 16:49
(>_<)
262132人目の素数さん:04/07/01 17:00
ab≦1/2(a^2+b^2)ぐらいしか思いつかん
>257
(s_k)/項数 = (s_k)/C[n,k] ≡ P_k とおくと P_0=1. もし
(P_i)/P_{i-1} ≧ P_{i+1}/(P_i) ・・・・・ (1)
が成り立てば
(P_k)/(P_0) = Π[i=1,k] (P_i)/P_{i-1} ≧ Π[i=n-k+1,n] (P_i)/P_{i-1} ≧ (P_n)/(P_{n-k}).
∴ (P_k)(P_{n-k}) ≧ (P_0)(P_n).

よって (1) に帰着する。
264132人目の素数さん:04/07/01 20:52
>>263
等号成立条件は?
   ___
 ./  不 \
 |::::  \ ./ |
 |::::: (≧ (≦ | ニヤニヤ
 ヽ::::... .∀....ノ  
>263 神キタ━(゚∀゚)━!!!
しかし、分からない…
>263さん、あなたにかかっています。
   ___ 
 ./  ≧ \
 |::::  \ ./ | 
 |::::: (● (● | ハァハァ
 ヽ::::... .ワ....ノ
>264
等号成立条件は、a_1 = a_2 = … = a_n
   ___ 
 ./  ≧ \
 |::::  \ ./ | 
 |::::: (● (● | ハァハァ
 ヽ::::... .ワ....ノ
>254
n! = C・{(n+1/2)^(n+1/2)}/(e^n) とおく。

x=k での接線から Ln(k) > ∫[x=k-1/2,k+1/2] Ln(x)dx.
台形公式から {Ln(k)+Ln(k+1)}/2 < ∫[x=k,k+1] Ln(x)dx.

Ln(n!) = Ln(a!) + Σ[k=a+1,n] Ln(k) > Ln(a!) + ∫[x=a+1/2,n+1/2] Ln(x)dx
= Ln(a!) + (n+1/2){Ln(n+1/2)-1} - (a+1/2){Ln(n+1/2)-1},
∴ Ln(C) > Ln(a!) - (a+1/2)Ln(a+1/2) + a.

Ln(n!) = Ln(a!) + (1/2)Ln(a+1) + ∫[x=a+1,n] Ln(x)dx + (1/2)Ln(n)
= Ln(a!) + (1/2)Ln(a+1) + n{Ln(n)-1} - (a+1){Ln(a+1)-1} + (1/2)Ln(n)
= Ln(a!) + (n+1/2)Ln(n) -n -(a+1/2){Ln(a+1)-1} + 1/2
< Ln(a!) + (n+1/2)Ln(n+1/2) -n -(a+1/2){Ln(a+1)-1}
∴ Ln(C) < Ln(a!) - (a+1/2){Ln(a+1)-1}.

C → √(2π/e)=1.520347 (n→∞)
269263:04/07/03 15:28
>264-266

【263の(1)】
1≦i≦n-1 ⇒ Q_i ≡(P_i)^2 - P_{i-1}・P_{i+1} ≧0.

(略証) n(変数の数)に関する帰納法で示す。

For i=1,
Q_1 = (P_1)^2 - P_0・P_2 = (1/n)^2・(納j=1,n] x_j)^2 - {2/n(n-1)}(納j>j'] x_j・x_j')
= {1/[n^2・(n-1)]}{(n-1)納j=1,n](x_j^2) -2・納j>j'] x_j・x_j'}
= {1/[n^2・(n-1)]}納j>j'] (x_j-x_j')^2 ≧ 0.
n=2 ⇒ i=1 ∴ n=2のとき成立。

Consider a new member x'= x_{n+1}.
s'_i ≡ s_i + x'・s_{i-1}    (1≦i≦n),
P'_i ≡ [(n+1-i)/(n+1)]・P_i + [i/(n+1)]x'・P_{i-1},
Q'_i ≡ (P'_i)^2 - P'_{i-1}・P'_{i+1},
and
(n+1)^2・Q'_i = (n-i)(n-i+2)・Q_i + (n-i)(i-1)[P_i・P_{i-1}−P_{i+1}P_{i-2}]x' +(i^2-1)・Q_{i-1}(x')^2 +[P_i-P_{i-1}x'}^2.

Provided Q_i≧0, Q_{i-1}≧0 for a certain n, then
P_i/P_{i+1} increase monotonously with i: P_i/P_{i+1} ≧ P_{i-2}/P_{i-1}.
∴ Q'_i≧0.
270263:04/07/05 20:59
【263の系】
P_1 ≦ (P_2)^(1/2) ≦ ・・・・ ≦ (P_k)^(1/k) ≦ ・・・・ ≦ (P_n)^(1/n)
271263:04/07/05 21:01
スマソ

【263(1)の系】
P_1 ≧ (P_2)^(1/2) ≧ ・・・・ ≧ (P_k)^(1/k) ≧ ・・・・ ≧ (P_n)^(1/n)
272132人目の素数さん:04/07/05 21:04
   ___ 
 ./  ≧ \
 |::::  \ ./ | ムズカシ…
 |::::: (● (● | ハァハァ
 ヽ::::... .ワ....ノ
ここにも燃料を投下していたのですが、過疎スレだけに…。
http://science3.2ch.net/test/read.cgi/math/1002903143/703
275132人目の素数さん:04/07/06 22:15
>274
反例?を書いて来たよ。
いい加減自演止めろ、うざい。
>276
自演じゃないですよ。

数オリや 数セミ 外国の月刊誌のAMMなどから
nCrの和や 三角関数や 不等式の問題を収集してきて
各スレに貼っている数オタは私ですが、解く力は弱い。

その問題を、私の写し間違いまで訂正した上で
一般化までして解説してくれている神は別人ですよ。
>>276
失せろ, 嫌なら見るな
>>278
氏ね、池沼。
あちこちのスレでよく見かけるけど、
池沼の意味が分かんないんですが?
>276は ageられたスレを見て、ウザイと反応しているのかな?
>>280
おまいの事だよ。
荒らしにも困ったものだ。

>256
f(a,b,c,d,e) = a/(a+b) + b/(b+c) + c/(c+d) + d/(d+e) + e/(e+a) とおくと
 f(a,b,c,d,e)+f(a,e,d,c,b)=5 … (1)
また、a/(a+b) > a/(a+b+c+d+e) などから
 f(a,b,c,d,e) > 1 … (2)
これらより、 1 < f(a,b,c,d,e) < 4

f(a^4,a^3,a^2,a,1) → 1 (a→0) だから、これと(1)より、
f(a,b,c,d,e)は1から4の間のすべての値をとる
>また、a/(a+b) > a/(a+b+c+d+e) などから
これは、実数a,b,c,d,eに対して無条件で明らかですか?
ー(a+b)<c+d+e<0<a の時はまずそうですが?

これから、(2) が出るのも、おれには明らかではない?
あぁ、問題文のミスで、a,b,c,d,eは正の数って約束があるんですよ。
すんません。>>259あたりにそのことが書いてあった。
了解。
>227
> 0<a<π/4、0<b<π/4 であるa,bについて
> √{tan(a)・tan(b)} ≦ tan((a+b)/2) ≦ {tan(a)+tan(b)}/2


この相加平均と相乗平均の間に挟まれているタイプの不等式を拾ってきたYo!

異なる正の数a,bに対して
√(ab) < (a-b)/{log(a)-log(b)} < (a+b)/2
288227:04/07/08 18:18
>287
sinh(|x|) ≧ |x| ≧ tanh(|x|) より 1≦ sinh(x)/x ≦ cosh(x).
x=(1/2)・Ln(b/a) とおき, 各辺に√(ab) を掛ける。
等号成立は x=0 つまり a=b.
【問題】
0<x<y<1 または 1<x<y に対して
 {y^(x^y)}/{x^(y^x)} > y/x > (y^x)/(x^y)

  ∧_∧
  ( ;´∀`) ハァハァ…
  人 Y /  モウ タマラン
 ( ヽ し
 (_)_)
290UltraMagic ◆NzF73DOPHc :04/07/08 22:55
0<x<y<1または1<x<yに対して、
x^y*y-y^x*xをxyで割り、
x^(y-1)-y^(x-1)を得る。
yで偏微分してln(x)x^(y-1)-(x-1)y^(x-2)を得る。
y>=xでそれは正となる(?)。
>289
(右側)題意より (x-1)(y-1)>0.
Ln(x)/(x-1)>0 は単調減少だから
x<y ⇒ Ln(x)/(x-1)>Ln(y)/(y-1)>0 ⇒ x^(y-1)>y^(x-1)>1 ⇒ y/x > (y^x)/(x^y).
>291
さすが!

>290
荒らすな!
自演はげ
294132人目の素数さん:04/07/18 08:53
age
The Cauchy-Schwarz Master Class: An Introduction to the Art of Mathematical Inequalities
http://www.amazon.co.jp/exec/obidos/ASIN/052154677X/qid=1090270289/ref=sr_8_xs_ap_i1_xgl14/249-4789393-9502752#product-details
296132人目の素数さん:04/07/21 16:48
マルコフの不等式
  ∧_∧
  ( ;´∀`) ハァハァ…
  人 Y /  モウ タマラン
 ( ヽ し
 (_)_)
三角形ABCの内角について、
sin(A)+sin(B)+sin(C) と sin(A/2)+sin(B/2)+sin(C/2)
の大小関係は定まりますか?
>298
二等辺三角形(A=B)の場合を考えると
0<A<19.178215°では 2sin(A)+sin(π-2A) < 2sin(A/2)+cos(A)
19.178215°<A< 90°では 2sin(A)+sin(π-2A) > 2sin(A/2)+cos(A)
>299
ありがとうございます。あと2式のとりうる値について、凸不等式から
0 < sin(A)+sin(B)+sin(C) < 3sqrt(3)/2
sin(A/2)+sin(B/2)+sin(C/2) < 3/2
となりましたが、下側の式の最小値がうまくでません。
1/2のようなのですが、証明のしかたを教えて下さい。
書き間違い…

0 < sin(A)+sin(B)+sin(C) ≦ 3sqrt(3)/2
sin(A/2)+sin(B/2)+sin(C/2) ≦ 3/2
>300
 1 < sin(A/2)+sin(B/2)+sin(C/2) ≦ 3/2.
 0 < {sin(A)+sin(B)+sin(C)} / {sin(A/2)+sin(B/2)+sin(C/2)} ≦ √3.
>302
ありがとうございます。神キタ━(゚∀゚)━!!!
1行目の式の下限が1になるのは、どのような計算でしょうか?
2行目の不等式を考えてますが、
 sin(A)+sin(B)+sin(C) = 4cos(A/2)cos(B/2)cos(C/2)
 sin(A/2)+sin(B/2)+sin(C/2) = 4cos(A/4)cos(B/4)cos(C/4)
ここまで変形して、辺々割ってみたんですけど、その後が…
もう少し考えてみますが、よければヒント下さい。
>303
sin(A/2) + sin(B/2) + sin(C/2) = 4cos{(π+A)/4}・cos{(π+B)/4}・cos{(π+C)/4} +1
ぢゃないか??
>304
ほんとだ間違ってる、ありがとうございます
306132人目の素数さん:04/07/23 07:36
telescopic 不等式
任意の自然数 m,n と任意の正の数 x に対して
 {(x+m)/(n+m)}^(n+m) ≧ (x/n)^n
   ___ 
 ./  ≧ \
 |::::  \ ./ | 
 |::::: (● (● | ハァハァ
 ヽ::::... .ワ....ノ
一般化して、正の数 a,b,c に対して
 {(a+c)/(b+c)}^(b+c) ≧ (a/b)^b
が言えそうですね、ハァハァ。
>307
平均値の定理(コーシー)より、
{Ln(x+m)-Ln(n+m)}/{Ln(x)-Ln(n)} = {1/(ξ+m)}/{1/ξ} = ξ/(ξ+m).
ところで、ξはxとnの中間の点だから、(ξ-n)(x-n) >0, (ξ-n){Ln(x)-Ln(n)} >0.
∴ (n+m){Ln(x+m)-Ln(n+m)} - n{Ln(x)-Ln(n)} = {(n+m)ξ/(ξ+m)-n}{Ln(x)-Ln(n)}
= {m/(ξ+m)}(ξ-n){Ln(x)-Ln(n)} ≧ 0.
∴ (n+m)・Ln{(x+m)/(n+m)} ≧ n・Ln(x/n).

微分法を使わない方法きぼんぬ
>307
Ln(x)/(x-1)>0 は単調減少だから
{Ln(z)/(z-1) - Ln(y)/(y-1)}(y-z) >0.
これに z=1+a/(n+m), y=1+a/n を代入して、{(n+m)・Ln[1+a/(n+m)]-n・Ln[1+a/n]}/{n(n+m)} ≧0.
∴ (n+m)・Ln{1+a/(n+m)} ≧ n・Ln(1+a/n).
ここで a=x-n とおく。
>309
>微分法を使わない方法きぼんぬ
   ___ 
 ./  ≧ \
 |::::  \ ./ | 相加相乗でハァハァと…
 |::::: (● (● | 
 ヽ::::... .ワ....ノ
312309:04/08/02 02:16
>311
thanx. x/n がn個と1がm個でつね。

【系】
 (1+a/n)^n はnについて単調増加(極限はe^a)
 [略証] 相加相乗ハァハァと・・・ 1+a/n がn個、1が1個。
313132人目の素数さん:04/08/04 01:33
>312 【系】 (*゚∀゚)=3 ハァハァ
掘り出し物

任意の実数 x,y,z に対して、次式をみたすMの最小値を求めよ。
√(x^2+y^2)+2√(y^2+z^2)+3√(z^2+x^2) ≦ M√(x^2+y^2+z^2)
316132人目の素数さん:04/08/07 23:39
2√7
317132人目の素数さん:04/08/08 00:18
>>315
ワロタ。
>316
ハズレ
319132人目の素数さん:04/08/08 04:49
M=3+√5
>319
どないすんねん?
おらおら!
       |   _,.. -‐"/ ̄/  /|  ̄ l ヽ  \~`"'ー、ノ   たのも〜♪
       ケフ" / /  ,.-'‐ ̄/ .i   .i  ̄\- \ \ヾ
      / /.l l l .// / ./  l  /    ヾ  iヽ  i.\     たのも〜♪
      ノ  | l l l Y /"¨''ヽ .i / ァ''"¨ヾ i イ゙i. リ
       `ヽ r、 丶l i`       レ       |  イ/"
         \ ヽ  ヽ """  iー'ーv'  """ /  '
          ヽ ヾ- ゝ    ._/   ./
         /''"" \Y.': ∧∧   ∧∧ソ `"ヽ、
        ,ィ"   ,.ィ."ヽ(=゚ω゚)人(゚ω゚=)ノ`丶,”、
       /" ヾ,.-"  〜(  x)、 /(x  )〜   `丶、
      / /"    \⊃U U y U U⊂/     ヽ
>316 の失敗は、シュワルツの不等式でやったんだろうな。
正解は >319 だが、どうやるのかにゃ?
y=0 のときだと見破れば簡単
見破るって…、それでいいのか?
ちゃんとした証明をキボンヌ
普通に束縛条件x^2+y^2+z^2=1を設定して左辺の
x>0,y>0z>0での最小値と
x=0,y,z≧0、y=0,z,x≧0、z=0,x,y≧0での最小値をくらべればいいんじゃないの?
>>326
>x>0,y>0z>0での最小値と
↑これ極値ね。最小値であることまで示す必要はない。
・コンパクト領域x,y,z≧0、x^2+y^2+z^2=1のどこかで最小値をとる。
・内点で最小ならそれは極値である。
から境界での最小値と内点での極値のうちいちばん小さいのが最小値。
↑この論法でどんどん変数の次元をさげていくのが一般的な未定定数法の使い方だな。
なるほど。
>325
x^2=X, y^2=Y, z^2=Z, λ>0, μ=√(1+λ^2) とおく。
コーシーの不等式より、√(X+Y) + λ√(Y+Z) ≦ μ√(X+2Y+Z) ・・・・・ (1)
ルートは上に凸だから、 √(X+2Y+Z) + √(X+Z) ≦ 2√(X+Y+Z) ・・・・・ (2)
∴ √(X+Y) + λ√(Y+Z) + μ√(X+Z) ≦ 2μ√(X+Y+Z).
これに、√(X+Z) ≦ √(X+Y+Z) ・・・・・(3) の(3-μ)倍を加える。
等号成立は (x,y,z)=x(1,0,λ) のとき.
ぬるぽ
330329:04/08/11 00:00
【類題】
λ≧0とする。 任意の負でない実数 X,Y,Z≧0 に対して次式をみたすM 
の最小値M(λ)を求めてくださいです。。。 
√(X+Y) + λ・√(Y+Z) + (λ+1)・√(X+Z) ≦ M・√(X+Y+Z) 
ぬるぽ 
>330
μ=√(1+λ^2) とおく。
コーシーの不等式より、√(X+Y) + λ√(Y+Z) + μ√(Z+X) ≦ √(1+λ^2 +μ^2)・√{2(X+Y+Z)} = 2μ√(X+Y+Z).
∴ M(λ) = 1+λ+√(1+λ^2).
332132人目の素数さん:04/08/11 09:04
(゚Д゚ )ハァ?
>316のようにコーシーの不等式を使って、
とりえない解を出した前例があるッ!
気をつけろッ!
334331:04/08/11 11:43
(X,Y,Z)=X(1,0,λ^2) で等号成立でつ。。。
335331:04/08/11 21:03
>332
 √(Z+X) ≦ √(X+Y+Z)
の(1+λ−μ)倍を [331]の式 に加えれ。
>331
理解したッ!
君は英雄だ、最大の功績だ!
337132人目の素数さん:04/08/17 16:27
コーシー・シュワルツの不等式を、ロシアでは
コーシー・ブニャコフスキーの不等式という。
北朝鮮ではなんと言うのだろうか?
金日成の不等式?
暑いのぉ…

(a+b+c+d)^2 ≧ 8(ac+bd) が成り立つための実数 a,b,c,d の条件を求めよ。
正の数 a,b,c が a^2+b^2+c^2=1 をみたすとき、次式を証明せよ。
 (a^2)bc + a(b^2)c + ab(c^2) ≦ 1/3
   ___ 
 ./  ≧ \
 |::::  \ ./ | 
 |::::: (● (● | オラオラ
 ヽ::::... .ワ....ノ
>339
左辺 = (a^2)bc + a(b^2)c + ab(c^2) = (a+b+c)abc
= (1/27)(a+b+c){(a+b+c)^3 -3a(b-c)^2 -3b(c-a)^2 -3(a-b)^2} ≦ (1/27)(a+b+c)^4,

(a+b+c)^2 = 3(a^2+b^2+c^2) -(b-c)^2 -(c-a)^2 -(a-b)^2 ≦ 3(a^2+b^2+c^2).
341340:04/08/21 11:38
>339
a^2=A, b^2=B, c^2=C とおく。
左辺 = A{B+C-(b-c)^2}/2 + B{C+A-(c-a)^2}/2 + C{A+B-(a-b)^2}/2
≦ BC + CA + AB = (1/6){2(A+B+C)^2 -(B-C)^2 -(C-A)^2 -(A-B)^2}
≦ (1/3)(A+B+C)^2.

等号成立はA=B=C すなわち a=b=c.
342132人目の素数さん:04/08/21 15:32
しょうも無い不等式やな。
>338
0 ≦ (a+b+c+d)^2 -8(ac+bd) = (a+b-c-d)^2 +(a-b-c+d)^2 -(a-b+c-d)^2.
ぬるぽ
ハァ?
正の数 a,b,c に対して、次を示せ。
(a^3-a^2+3)(b^3-b^2+3)(c^3-c^2+3) ≧ (a+b+c)^3
   ___ 
 ./  ≧ \
 |::::  \ ./ | 
 |::::: (● (● | オラオラオラ…
 ヽ::::... .ワ....ノ
346凡例:04/08/21 23:08
>345
a=b=c の場合を考える。
1<t<√3 ⇒ (t^3 -t^2 +3) - 3t = (t-1)(t^2 -3) < 0.
>346
不等式の証明をしたことがありますか?
348反例:04/08/22 01:34
を挙げますた。証明できんと思われ。 >347
>345
問題文が違っていると思われ…
確認して切腹せよ!
チンチン! チンチン! (AA省略)
問題文の訂正と、切腹まだあー
351340:04/08/22 15:16
>339
abc = (1/27){(a+b+c)^3 -(3a+s)(b-c)^2 -(3b+s)(c-a)^2 -(3c+s)(a-b)^2} ≦ (1/27)(a+b+c)^3.
s = (a+b+c)/2.

すみません、間違ってました。
死んでお詫びを…

         ∧_∧
         (´Д` )   
         /  y/  ヽ      
    Σ(m)二フ ⊂[_ノ
        (ノノノ | | | l )
352345の訂正:04/08/22 18:56
アチャー、間違ってました。5乗でした…

正の数 a,b,c に対して、次を示せ。
(a^5-a^2+3)(b^5-b^2+3)(c^5-c^2+3) ≧ (a+b+c)^3

                     ∧_∧  死んでお詫びを…
      _ _  ξ            (´Д` )
    (´   `ヽ、>>340         /  y/  ヽ >>345
  ⊂,_と(    )⊃     Σ(m)二フ ⊂[_ノ
                    (ノノノ | | | l )
||
||---
||
||---
||
||
||
||
|| 電柱でござる、電柱でござるぞ。 >351-352
任意の複素数 x,y,z に対して
|x|+|y|+|z|+|x+y+z| ≧ |x+y|+|y+z|+|z+x|
   ___ 
 ./  ≧ \
 |::::  \ ./ | 懲りずに
 |::::: (● (● | ハァハァ
 ヽ::::... .ワ....ノ
       |   _,.. -‐"/ ̄/  /|  ̄ l ヽ  \~`"'ー、ノ   たのも〜♪
       ケフ" / /  ,.-'‐ ̄/ .i   .i  ̄\- \ \ヾ
      / /.l l l .// / ./  l  /    ヾ  iヽ  i.\     たのも〜♪
      ノ  | l l l Y /"¨''ヽ .i / ァ''"¨ヾ i イ゙i. リ
       `ヽ r、 丶l i`       レ       |  イ/"
         \ ヽ  ヽ """  iー'ーv'  """ /  '
          ヽ ヾ- ゝ    ._/   ./
         /''"" \Y.': ∧∧   ∧∧ソ `"ヽ、
        ,ィ"   ,.ィ."ヽ(=゚ω゚)人(゚ω゚=)ノ`丶,”、
       /" ヾ,.-"  〜(  x)、 /(x  )〜   `丶、
      / /"    \⊃U U y U U⊂/     ヽ
         ___
       ./  ≧ \   神降臨まだぁ〜
       |::::  \ ./ | ハァハァ
       |::::: (● (● |
       ヽ::::... .∀....ノ /  チン ☆
      _(  ⊃  ⊃  チン ☆
      |\ ̄ ̄ ̄ ̄旦 ̄\
      | | ̄ ̄ ̄ ̄ ̄ ̄ ̄|
      \|  愛媛みかん |
>354
Hlawkaの不等式.

x+y+z=-w, 左辺=|x|+|y|+|z|+|w|=S とおく。

|x+y|=|z+w|, |x+z|=|y+w|, |x+w|=|y+z| ・・・・・・(1)

0 = |x+y+z+w|^2
= (|x+y|^2 +|z+w|^2) +(|x+z|^2 +|y+w|^2) +(|x+w|^2 +|y+z|^2) -2(|x|^2 +|y|^2 +|z|^2 +|w|^2).
∴ S^2 = (|x|^2 +|y|^2 +|z|^2 +|w|^2) + 2{|x||y|+|x||z|+|x||w|+|y||z|+|y||w|+|z||w|}
= (1/2){|x+y|^2 +|z+w|^2 +|x+z|^2 +|y+w|^2 +|x+w|^2 +|y+z|^2}
+2{|x||y|+|x||z|+|x||w|+|y||z|+|y||w|+|z||w|}
= |x+y||z+w| +|x+z||y+w| +|x+w||y+z| +2{|x||y|+|x||z|+|x||w|+|y||z|+|y||w|+|z||w|}
= |x+y||z+w| +(|x|+|y|)(|z|+|w|)
+|x+z||y+w| +(|x|+|z|)(|y|+|w|)
+|x+w||y+z| +(|x|+|w|)(|y|+|z|) ・・・・・・・(2)

(1),(2)により、
S・{S -(|x-y|+|x+z|+|x+w|+|y+z|+|y+w|+|z+w|)/2}
= S^2 -S(|x+y|+|z+w|)/2 -S(|x+z|+|y+w|)/2 -S(|x+w|+|y+z|)/2
= |x+y||z+w| +(|x|+|y|)(|z|+|w|) +|x+z||y+w| +(|x|+|z|)(|y|+|w|) +|x+w||y+z| +(|x|+|w|)(|y|+|z|)
-(|z|+|w|)|x+y| -(|x|+|y|)|z+w| -(|y|+|w|)|x+z| -(|x|+|z|)|y+w| -(|y|+|z|)|x+w| -(|x|+|w|)|y+z|)}
= {|x|+|y|-|x+y|)(|z|+|w|-|z+w|) +(|x|+|z|-|x+z|)(|y|+|w|-|y+w|) +(|x|+|w|-|x+w|)(|y|+|z|-|y+z|) ≧0.
∴ S - (1/2)(|x+y|+|x+z|+|x+w|+|y+z|+|y+w|+|z+w|) ≧0.

大関:「不等式への招待」例題8, p.33 近代科学社 (1987.10) ¥1575
153p, A5判, ISBN:4764910063
お〜!
そんなところにありましたか
ありがとうございます。
359357:04/08/26 21:39
(補足)
0 = |x+y+z+w|^2 = (x+y+z+w, x+y+z+w)
= |x|^2 +|y|^2 +|z|^2 +|w|^2 +2{(x,y)+(x,z)+(x,w)+(y,z)+(y,w)+(z,w)}
= |x+y|^2 +|z+w|^2 +|x+z|^2 +|y+w|^2 +|x+w|^2 +|y+z|^2 -2(|x|^2 +|y|^2 +|z|^2 +|w|^2).
【問題】(D.D.Adamovic)
m>2, Σ[k=1,m] x_k↑ = 0↑ のとき、
1). (m-2) Σ[k=1,m] |x_k|^2 = Σ[1≦i<j≦m] |x_i +x_j|^2.
2). (m-2) Σ[k=1,m] |x_k| ≧ Σ[1≦i<j≦m] |x_i +x_j|.
を示してくださいです。。。
>352
F(x) = x^5 -x^2 +3 ≧ F{(2/5)^(1/3)} = 3 - (3/5)[(2/5)^(2/3)] = 2.67426989 ≡ f.

次の補題を基にして評価する。
【補題】
F(a)F(b)F(c) ≧ 27abc + (1-a)^2 [(f^2)a + (9f/2)(b+c)]
+ (1-b)^2 [(f^2)b + (9f/2)(c+a)] + (1-c)^2 [(f^2)c + (9f/2)(a+b)].

(1-a)^2 + (1-b)^2 = (1/2)(a-b)^2 + (1/2)(2-a-b)^2 ≧ (1/2)(a-b)^2 を使って変形すると、

F(a)F(b)F(c) ≧ 27abc + (a-b)^2 {[9f/4 -(f/2)^2]c + (f/2)^2(a+b)}
+ (b-c)^2{[9f/4 -(f/2)^2]a + (f/2)^2(b+c)} + (c-a)^2 {[9f/4 -(f/2)^2)b + (f/2)^2(c+a)}
≧ 27abc + (a-b)^2 (a+b+7c)/2 + (b-c)^2 (7a+b+c)/2 + (c-a)^2 (a+7b+c)/2
= (a+b+c)^3.

∵ 9f/4 -(f/2)^2 ≒ 4.2291774 > 7/2, (f/2)^2 ≒ 1.78792986 > 1/2.
362361:04/08/29 18:13
【補題】の略証
F(x) = x^5 - x^2 + 3 = 3x + (1-x)^2 (3+3x+2x^2+x^3) = 3x + G(x) とおく。

F(a)F(b)F(c) = [3a+G(a)][3b+G(b)][3c+G(c)] = 27abc + G(a)・D_a + G(b)・D_b + G(c)・D_c
D_a = 9bc + (3/2)[bG(c)+cG(b)] + (1/3)G(b)G(c),
D_b = 9ca + (3/2)[cG(a)+aG(c)] + (1/3)G(c)G(a),
D_c = 9ab + (3/2)[aG(b)+bG(a)} + (1/3)G(a)G(b).
ここで D_ の上限を2とおりに評価できる。
D_a ≧ (3/2){bF(c)+cF(b)} ≧ (3f/2)(b+c) = 4.01140483(b+c),
D_a ≧ F(b)F(c)/3 ≧ (1/3)f^2 = 2.38390647.
∴ (3+3a)D_a ≧ (f^2)a + (9f/2)(b+c)
また、G(x) ≧ (1-x)^2(3+3x) より,
F(a)F(b)F(c) = 27abc + G(a)・D_a + G(b)・D_b + G(c)・D_c
≧ 27abc + (1-a)^2(3+3a)・D_a + (1-b)^2(3+3b)・D_b + (1-c)^2(3+3c)・D_c
≧ 27abc + (1-a)^2[(f^)a + (9f/2)(b+c)] + (1-b)^2[(f^2)b + (9f/2)(c+a)] + (1-c)^2[(f^2)c + (9f/2)(a+b)].
363362:04/08/29 18:19
>352
等号成立は a=b=c=1 のとき。

[362]の]訂正
D_の下限を2とおりに・・・
>360 (1)
0 = |Σ[k=1,m] x_k↑ |^2 = Σ[k=1,m] |x_k|^2 + Σ[1≦i<j≦m] 2(x_i, x_j)
= Σ[1≦i<j≦m] {|x_i|^2 +|x_j|^2 +2(x_i, x_j)} − (m-2)Σ[k=1,m] |x_k|^2
= Σ[1≦i<j≦m] |x_i +x_j|^2 − (m-2)Σ[k=1,m] |x_k|^2.
365ぬるぽ:04/08/31 23:03
>361 
 f=2 で十分と思われ...
 x<1 のとき F(x)> -x^2 +3> 2
 x>1 のとき F(x) = x^2(x^3-1)+3 > 3

  ∧_∧
  ( ;´∀`)  <ぬるぽ 
いつもお世話になります、だめぽ不等式ヲタです。
いくつか入手したので、分からないのがあったら質問します。
よろしくお願いします。
   ___ 
 ./  ≧ \
 |::::  \ ./ | 懲りずに
 |::::: (● (● | ハァハァ
 ヽ::::... .ワ....ノ
>>352
F(x) = x^5 -x^2 +3 ≧ x^3 +2 だから次の[命題]に帰着する。
(証)F(x) - (x^3 +2) = x^5 - x^3 -x^2 +1 = (x^3 -1)(x^2 -1) ≧0.(終)

【命題】a,b,c≧0 ⇒ (a^3 +2)(b^3 +2)(c^3 +2) ≧ (a+b+c)^3.
(略証) 下の補題より、
(a^3 +2)(b^3 +2)(c^3 +2) = (abc)^3 + 2{(ab)^3 + (bc)^3 +(ca)^3} + 4(a^3 +b^3 +c^3) +8
= [(abc)^3 +1+1] + [(ab)^3 +a^3 +1] + [(ab)^3 +b^3 +1] +[(bc)^3 + b^3 +1]
+ [(bc)^3 +c^3 +1] + [(ca)^3 +c^3 +1] + [(ca)^3 +a^3 +1] + [a^3 +b^3 +c^3] +a^3 +b^3 +c^3
≧ 3{2abc + (a^2)(b+c) +(b^2)(c+a) +(c^2)(a+b)} + a^3 +b^3 +c^3
= (a+b+c)^3. 
等号成立は a=b=c=1 のとき.(終)

【補題】(相加相乗平均)
x+y+z≧0 ⇒ [x^3 +y^3 +z^3] ≧ 3xyz.
(証) x^3 +y^3 +z^3 - 3xyz
= (x+y+z)(x^2 +y^2 +z^2 -xy -yz -zx)
= (x+y+z){(x-y)^2 +(y-z)^2 +(z-x)^2}/2 ≧0. 
 等号成立は x=y=z のとき. (終)
>367 神降臨キタ━(゚∀゚)━!!!
> (a^3 +2)(b^3 +2)(c^3 +2) ≧ (a+b+c)^3

Helderの不等式でもハァハァ…。
 (a^3+1^3+1^3)^(1/3)*(b^3+1^3+1^3)^(1/3)*(c^3+1^3+1^3)^(1/3) ≧ a+b+c
既出かもしれないけど、保守ついでに…。

a≧b≧c≧0, a+b+c=3 のとき、ab^2 + bc^2 + ca^2 ≦ 27/8

[2002 中国]らしい、確認できなかったけど…)
>369
(a+b+c)^3 - 8(ab^2 +bc^2 +ca^2) -3abc = (a+5c)(a-b)^2 +(4a-4b+c)(b-c)^2 +c(a-b)(b-c) ≧ 0.
∴ ab^2 + bc^2 + ca^2 + (3/8)abc ≦ (1/8)(a+b+c)^3.
でもいいらしい。確認できなかったけど・・・

>368
Hoelder の式は >>137,>>139 にありまつ。略証は >>158
ぬるぽ 
371370:04/09/05 18:12
>369
(a+b+c)^3 - 8(ab^2 +bc^2 +ca^2) -3abc = (a+5b-5c)(a-b)^2 +(4a-4b+c)(b-c)^2 +c(a-b)(b-c) ≧ 0.
でないといけないらしい。すまそ...
   ___
 ./  ≧ \
 |::::  \ ./ |
 |::::: (● (● | 
 ヽ::::... .ワ.....ノ   | | ガッ 
  と     )   .| |
     Y /ノ    人
      / )    <  >__Λ∩
    _/し' //. V`Д´)/ ←>>370
   (_フ彡        /
373132人目の素数さん:04/09/08 00:47
任意の自然数nについて,∫[n-1/2→n+1/2]√xdx<√nを示せ.
わからんぽ…教えてくり
>>373
y=√xの点(n,√n)における接線をy=ax+bとするとax+b≧√x、等号はx=nのときのみ
なので
√n=∫[n-1/2,n+1/2](ax+b)dx>∫[n-1/2,n+1/2]√xdx
>374
y=√x は上に凸なので.....
ぬるぽ
376132人目の素数さん:04/09/08 18:09
任意の自然数nについて,∫_[n-1,n] √x dx > {√(n-1)+√n}/2 を示せ.
わからんぽ…教えてくり
>>376
面積比べれ、台形と。
   ___
 ./  ≧ \
 |::::  \ ./ |
 |::::: (● (● | 
 ヽ::::... .ワ.....ノ   | | ガッ 
  と     )   .| |
     Y /ノ    人
      / )    <  >__Λ∩
    _/し' //. V`Д´)/ ←>>375
   (_フ彡        /
379132人目の素数さん:04/09/09 13:01
もっとおもろいん無いのけ
>379

面白い問題おしえてーな
http://science3.2ch.net/test/read.cgi/math/1093676103/
>>357
x,y,zがベクトルのときの Hlawkaの不等式の等号成立条件は
「x,y,zが同じ向きのとき」 または 「x,y,zの少なくとも一つが0」
でよろしいですか?
382357:04/09/10 20:21:09
>381
等号成立は「x,y,z,wのうちの3つが同じ向きのとき」でつ。
このとき x+y+z+w=0 から他の1つは逆向きとなり、x,y,z,wは共線または0でつ。
「x,y,zの少なくとも一つが0」であっても等号が成立するとは限りません。
ぬるぽ
383132人目の素数さん:04/09/10 21:44:27
【Hlawkaの不等式】
任意のベクトル x,y,z に対して
|x|+|y|+|z|+|x+y+z| ≧ |x+y|+|y+z|+|z+x|

x,y,zのうちの少なくとも一つ、例えば z=0のとき
(左辺) = (右辺) = |x|+|y|+|x+y|

y=px, z=qx (p,q>0) のとき、
(左辺) = (右辺) = 2(1+p+q)|x|
384132人目の素数さん:04/09/10 23:03:48
   ___
 ./  ≧ \
 |::::  \ ./ |
 |::::: (● (● | 
 ヽ::::... .ワ.....ノ   | | ガッ 
  と     )   .| |
     Y /ノ    人
      / )    <  >__Λ∩
    _/し' //. V`Д´)/ ←>>382
   (_フ彡        /
385132人目の素数さん:04/09/11 00:09:08
>382の切腹まだぁ?
386382:04/09/11 01:01:59
>385
等号成立は「x,y,z,wが共線で、その3つが同じ向きのとき」でつ。
このとき x+y+z+w=0 から他の1つは逆向きとなりまつ。(0↑はどの方向とも共線とする)

すみません、間違ってました。
死んでお詫びを…

         ∧_∧
         (´Д` )   
         /  y/  ヽ      
    Σ(m)二フ ⊂[_ノ
        (ノノノ | | | l )
387132人目の素数さん:04/09/12 14:27:44
【問題】
f(x),g(x)は 0≦x≦1で連続とし、f(0)=0, f(x)>x, g(1)≦1 を満たし、
f(x)/x, g(x)/x はともに狭義の単調増加であるとする。 このとき
 0<x<1 で f(g(x)) ≦ f(x)g(x)/x ≦ g(f(x)).
を示してくださいです。 〔R.P.Boas: Math.Magazine,52(1979)〕
388132人目の素数さん:04/09/12 15:01:40
不等式(ヒルベルト)でげす。

1/p+1/q=1, p>1, f(x)>0, g(y)>0 (x≧0,y≧0) としまする。

{∫_[0,∞) f(x)^p dx}^(1/p){∫_[0,∞) g(x)^q dy}^(1/q) > {sin(π/p)/π}∫∫{f(x)g(y)/(x+y)}dxdy

を示してくださいです。

俺にはサパーリ
389132人目の素数さん:04/09/12 19:04:31
   ___ 
 ./  ≧ \
 |::::  \ ./ | 
 |::::: (● (● | ハァハァ
 ヽ::::... .ワ....ノ
390132人目の素数さん:04/09/12 19:12:36
>>387
f(x),g(x)は 0≦x≦1で0≦f(x)≦1とか0≦g(x)≦1とか仮定していいの?
でないとf(g(x))とかg(f(x))とかかんがえるとき困るとおもうんだけど。
391132人目の素数さん:04/09/12 22:42:02
>>388
まず極座標(r,θ)に変換して、rについてヘルダーの不等式を使う。
t = 1/p, tanθ = s とすると、
∫[0,∞) (1+s)s^t ds = π/sin(πt) (0<t≦1/2)を示せばよいことがわかる。
t ≦ 1/2 かつ t が有理数のときは留数定理で計算できる。
tについての連続性からt ≦ 1/2でもこの等式は成立する。
392132人目の素数さん:04/09/12 22:46:44
分かスレ185にありますた。

513 :132人目の素数さん :04/09/12 13:57:18
a,b,c,d>0 ならば (a+b+c+d)^3 - (a^3+b^3+c^3+d^3) ≧ 15abcd(1/a+1/b+1/c+1/d)
を示してくださいです。

http://science3.2ch.net/test/read.cgi/math/1094542985/513,547
393132人目の素数さん:04/09/12 22:47:36
訂正 ∫[0,∞) (1+s)s^t ds → ∫[0,∞) 1/{(1+s)s^t} ds
394132人目の素数さん:04/09/12 23:16:42
395132人目の素数さん:04/09/13 03:12:52
>394
ここは 不等式の収集場所だから いいのれす。
396132人目の素数さん:04/09/14 08:03:10
>393
> ∫[0,∞) 1/{(1+s)s^t} ds = π/sin(πt) (0<t<1)

高木:「解析概論」改訂第三版、岩波書店
 p.242 [例]
 p.264 練習問題(5)-(8)[注意]
397132人目の素数さん:04/09/16 02:17:47
マルチ
398132人目の素数さん:04/09/16 11:36:25
>392
【類題】n≧3, a_1〜a_nのk次の対称式をS_k とおくと、
(S_1)^3 - Σ[k=1〜n](a_k)^3 ≧ {6(n+1)/(n-2)}S_3.

(略証)
左辺 - 右辺 = 3(S_1・S_2 - S_3) - {6(n+1)/(n-2)}S_3
= [3/(n-2)]{(n-2)S_1・S_2 - 3nS_3}

(n-2)S_2・S_1 −3nS_3
= (n-2)Σ[i<j] a_i・a_j Σ[k=1〜n] a_k - 3nΣ[i<j<k] a_i・a_j・a_k
= (n-2)Σ[i<j] a_i・a_j・(a_i+a_j) - 6Σ[i<j<k] a_i・a_j・a_k
= Σ[i<j] {Σ[k≠i,j]a_k}{(a_i)^2 +(a_j)^2} - Σ[i<j] {Σ[k≠i,j]a_k}2a_i・a_j
= Σ[i<j] {Σ[k≠i,j]a_k}(a_i-a_j)^2 ≧ 0.
ぬるぽ
399132人目の素数さん:04/09/16 14:25:23
   ___
 ./  ≧ \
 |::::  \ ./ |   グッジョブ!
 |::::: (● (● | 
 ヽ::::... .ワ.....ノ   | | ガッ 
  と     )   .| |
     Y /ノ    人
      / )    <  >__Λ∩
    _/し' //. V`Д´)/ ←>>398
   (_フ彡        /
400398:04/09/17 11:52:08
>392
【類題】n≧r≧1, a_1〜a_n>0, j次の基本対称式をS_jとおくと、
 (S_1)^r −Σ[k=1〜n](a_k)^r ≧ {((n^r)-n)/C[n,r]}S_r,
 等号成立は r=1,2, n=1 または a_1=・・・=a_n のとき。
 C[n,r]は2項係数でつ。
401132人目の素数さん:04/09/17 16:59:01
>398の最後の式変形は、解読するのにすごく時間が掛かった。
>400は、もっと大変そうな予感…。
402132人目の素数さん:04/09/21 13:00:47
任意の実数a,bに対して、a^2+b^2+1 ≧ (√2)a(b+1) を示せ。
403132人目の素数さん:04/09/21 21:59:03
>402
(a^2 +b^2 +1)^2 - 2{a(b+1)}^2
= a^4 +2(a^2)(b^2 +1) +(b^2 +1)^2 -2(a^2)(b^2 +2b +1)
= a^4 -2(a^2)(2b) + (2b)^2 + (b^2 -1)^2
= (a^2-2b)^2 + (b^2 -1)^2 ≧0.
∴ a^2 +b^2 +1 ≧ √2 |a(b+1)|.
404132人目の素数さん:04/09/21 22:17:09
>402
2(b^2 +1) = (b+1)^2 + (b-1)^2 ≧ (b+1)^2.
∴ 左辺 = a^2 + (b^2 +1) ≧ 2|a|√(b^2 +1) = (√2)|a(b+1)| ≧ 右辺.
等号成立は a=√2, b=1 のとき。
ぬるぽ
405132人目の素数さん:04/09/22 07:49:20
>402
(1/2)a^2 + b^2 = (√2)ab + (a/√2 -b)^2 ≧ (√2)ab, 等号は a/(√2)=b.
(1/2)a^2 + 1^2 = (√2)a + (a/√2 -1)^2 ≧ (√2)a, 等号は a/(√2)=1.
辺々たす。
ぬるぽ
406132人目の素数さん:04/09/23 14:03:31
キタ━(゚∀゚)━!!!!
405の解き方には全く気づきませんでした
   ___
 ./  ≧ \
 |::::  \ ./ |    グッジョブ!
 |::::: (● (● | 
 ヽ::::... .ワ.....ノ   | | ガッ 
  と     )   .| |
     Y /ノ    人
      / )    <  >__Λ∩
    _/し' //. V`Д´)/ ←>>403-405
   (_フ彡        /
407132人目の素数さん:04/09/25 00:15:05
既出だったらスマソ。
正の数 a,b,c に対して、次の不等式を示せ。

1/{a(1+b)} + 1/{b(1+c)} + 1/{c(1+a)} ≧ 3/(1+abc)
408132人目の素数さん:04/09/25 17:28:13
納k=1→n]1/2^k<(n/(2^n))納k=0→n](nCk)/(2k+1)を証明せよ

ムズ
409132人目の素数さん:04/09/26 02:15:02
>400
降参です。
模範解答を教えて下さい。
410132人目の素数さん:04/09/26 08:35:46
>>400
相加相乗つかうだけじゃないの?
以下1以上の整数nと整数1≦r≦nを固定する。
D={(di)∈Z^n | diは非負実数}、d∈Dと不定元X1・・・Xnに対し
X^d=Π[i=1,n]Xi^diと書く。Πr={(πi) | π1≧π2≧・・・は非負整数列で買ホi=r}、
π∈Πに対しd(π)=(π1,π2,・・・πn)とする。
n次対称群G=SnをDに自然に作用させてG(d)={σ|σ(d)=d}とさだめる。以下をしめせばよい。
----
またπ∈ΠにたいしてS(π)=納σ∈G]X^(σ(d(π)))とする。このとき任意の正の実数の
組(x1,・・・xn)にたいしてS(π)(x1,・・・,xn)≧(n!/C[n,r])Sr(x1,・・・,xn)、等号成立はπ1=π2=・・・πr=1のとき
またはx1=x2=・・・=xnのとき。
(証明)d0=(1,1,・・・,1,0,・・・0,)(最初のr個が1である多重次数)とおく。
このとき相加相乗平均の関係式より
(1/r)納σ∈<(1,2,3,・・・,r)>]x^{σ(d(π))}≧(Π[σ∈<(1,2,3,・・・,r)>]x^{σ(d(π))})^(1/r)=x^d0
∴納σ∈<(1,2,3,・・・,r)>]x^{σ(d(π))}≧rx^d0
∴納τ∈G,σ∈<(1,2,3,・・・,r)>]x^{τσ(d(π))}≧r納τ∈G]x^τ(d(0))
等号成立は主張の等号成立条件が成立するとき。左辺はrS(π)
であり右辺は(n!/C[n,r])Sr(x1,・・・,xn)。
411132人目の素数さん:04/09/27 08:33:20
>408
右辺 = {n/(2^n)}Σ[k=0,n] C[n,k]/(2k+1)
= ∫_[x=0,1] {n/(2^n)}Σ[k=0→n] C[n,k] x^(2k) dx
= ∫_[x=0,1] n{(1+x^2)/2}^n dx
> ∫_[x=0,1] n{(1+x^2)/2}^(n-1) xdx
= ∫_[u=1/2,1] n{u^(n-1)} du
= [u^n](u:1/2→1)
= 1-(1/2)^n
= Σ[k=1→n] 1/(2^k)
= 左辺.
ここに u=(1+x^2)/2 ≧x, du=xdx.
ぬるぽ
412132人目の素数さん:04/09/27 22:15:18
>>411
すばらしいね。
うまく最後まで持っていくところは感銘を覚える。
ぬるぽ
413132人目の素数さん:04/09/28 20:31:01
分かスレ187にありますた。

459 :132人目の素数さん :04/09/28 18:22:22
(前略)
0<b+c,0<c+a,0<c<a+bならば,1/(a+b) < 1/(b+c) + 1/(c+a)
の証明を教えてください.

http://science3.2ch.net/test/read.cgi/math/1095840922/459,460
414132人目の素数さん:04/09/29 15:34:11
460 :132人目の素数さん :04/09/28 18:40:18
>>459
(a+b)(a+b+2c)-(b+c)(c+a)
= (a+b)^2 +2(a+b)c -(c^2) -(a+b)c-ab
= (a+b)^2 +(a+b)c -(c^2) -ab
= (a^2)+ab+(b^2) +c(a+b-c) > 0

(b+c)(c+a) < (a+b)(a+b+2c)
1/(a+b) < (a+b+2c)/{(b+c)(c+a)} = {1/(b+c)}+{1/(c+a)}
415132人目の素数さん:04/09/29 15:42:57
不等式のコレクションがイパーイ!
(;´Д`)ハァハァ /lァ/lァ //ア//ア!!

a,b,c,d は実数。a^2+b^2≦1 のとき次を示せ。
 (a^2+b^2-1)(c^2+d^2-1) ≦ (ab+cd-1)^2
416132人目の素数さん:04/09/29 15:46:14
正の数 a_1,…,a_n が、
  1/(a_1+1998) + … 1/(a_n+1998) = 1/1998
をみたすとき、次を示せ
  {(a_1…a_n)^(1/n)}/(n-1) ≧ 1998
417132人目の素数さん:04/09/29 15:49:33
正の数 a,b,c,d が abcd=1 をみたすとき、次を示せ。
  (a^3+b^3+c^3+d^3) ≧ max{a+b+c+d, (1/a)+(1/b)+(1/c)+(1/d)}
418132人目の素数さん:04/09/29 16:26:25
>415-417をたのもー

       |   _,.. -‐"/ ̄/  /|  ̄ l ヽ  \~`"'ー、ノ   たのも〜♪
       ケフ" / /  ,.-'‐ ̄/ .i   .i  ̄\- \ \ヾ
      / /.l l l .// / ./  l  /    ヾ  iヽ  i.\     たのも〜♪
      ノ  | l l l Y /"¨''ヽ .i / ァ''"¨ヾ i イ゙i. リ
       `ヽ r、 丶l i`       レ       |  イ/"
         \ ヽ  ヽ """  iー'ーv'  """ /  '
          ヽ ヾ- ゝ    ._/   ./
         /''"" \Y.': ∧∧   ∧∧ソ `"ヽ、
419132人目の素数さん:04/09/29 22:45:45
>417
相加相乗の関係(Rべき)より、0≦r≦1 に対して
 (1-r)・a^n + (r/3)(b^n +c^n +d^n) ≧ a^{n(1-r)}・(bcd)^(nr/3) = (abcd)^(nr/3)・a^s.
ここに、s=n(1-4r/3) とおいた。
文字変数を巡回的に入れ替えて加えると、
 a^n +b^n +c^n +d^n ≧ (abcd)^(nr/3)・(a^s +b^s +c^s +d^s)
r=(3/4)(n-1)/n のとき s=1, r=(3/4)(n+1)/n のとき s=-1.
ぬるぽ
420132人目の素数さん:04/09/29 23:19:08
>415
c^2+d^2≧1 のときは 左辺≦0≦右辺 より明らか。
c^2+d^2≦1 のときは、0 ≦ 1-a^2 -b^2 ≦ 1-2ab, 0 ≦ 1-c^2-d^2 ≦ 1-2cd より、
左辺 ≦ (1-2ab)(1-2cd) ≦ (1/4){(1-2ab)+(1-2cd)}^2 = (1-ab-cd)^2 = 右辺.
ぬるぽ
421132人目の素数さん:04/09/29 23:37:05
神キタ━(゚∀゚)━!!!!  いつもありがとうございます。
 グッジョブ!    __   ∩            ∩ _
        -´─- 、\H- 、_,,,,......イヘへ、_/7'´_~二ニ
           r==⊂エニ/__::_:::\:://ハ::::::::V´二二二
         l   /::/:::::::::::>::::::::::/::ゞ;l;::::::八コ⊃ /
          l //::/:::/:::/::::::::::::/:i::l:::l::l:::::::::::!ヽ\
         |/::/:::/:::/::/::/|:|:::::/|::|:::|::|!:l:::::::::::|::::l  \
         /|:/|::/:|:::|:::||::|_,!|/::::::!:||::j::|!::l::::::::::|::::::l_ノ
        //.|! .!::|::|::|ゞ|V_i:|;、:::::::/jノjムノ|/!:::l:::::l:::::::l
       //   .|::|::|;;|/〇:゙li ゞノ fl〇::lト |::::j::::l_::::::|l
     __ `ー-、 |::|::f^ヘ.ゞ:ノj    {|ゞ::ノj !:::/:::j リ::∧! /
    /  l    \!:|:八! `¨´ rー-v、``¨´ l:::/::/ノ  K´
  ,.- !、_. {   //|:ト;:::::ゝ、.  !  ノ   _ノ:/!::/:||   |l
. j  __  ) ゙i_// ゞf⌒ゞ=>_`_ニ - ェヱ:/〆し||   |l
. !    ̄).八::: ̄T''亠-く_冫 /:l ト//:::://    _〕   |l
. l  `¨ )  !:::::::::l、__ノ::::::〕, ,':::ゞ|/:::rー!/   <||   |l
 \ ''´ノノ-l:::::::::::::::::::::::(_/└ー亠──┘ 丶、 ノ~\ |l
    ̄|~|;;;;/::::::::::::::::::::::::::::)          l  厂:::::::::\
      !::!/::::::::::::::::::_f⌒ビ/          ト-'゙:::::::::::::::::::
     レ':::::::::::::::::(_ ※.f゙              !;:::::::::::::::::::::::
      /_f⌒L;:::::::::::ゝ┌┤            l;::_f⌒L;::::::::
.    /〔 ※ )::::r─亠‥      _r-∩_ ├¬  )::::
422132人目の素数さん:04/09/30 05:46:35
>415の類題
a,b,c,d は実数。a^2+b^2≦1 のとき次を示せ。
 (a^2+b^2-1)(c^2+d^2-1) ≦ (ac+bd-1)^2
423132人目の素数さん:04/09/30 11:59:31
>422
c^2 +d^2 ≧1 のときは 左辺≦0≦右辺 で明らか。
c^2 +d^2 ≦1 のときは、(a,b)=r↑, (c,d)=R↑, r↑・R↑= s とおくと
1-a^2-b^2=1-r^2, 1-c^2-d^2=1-R^2, 1-ac-bd =1-s, s≦min(rR,2-rR) で、
右辺−左辺 = (1-s)^2 - (1-r^2)(1-R^2) = (r-R)^2 + (rR-s)(2-rR-s) ≧0.
ぬるぽ
424132人目の素数さん:04/09/30 22:16:33
>419
>相加相乗の関係(Rべき)

なんですか、それは?
425132人目の素数さん:04/10/01 11:37:32
>416
a_k/1998 = x_k とおくと、Σ[k=1,n] 1/(1+x_k) - 1 = 0.
(1+x_1)(1+x_2)・・・・・(1+x_n) を掛けて通分すると
Σ[k=0,n] (n-k)S_k −Σ[k=0,n] S_k =0.
ここに、S_k は {x_1,x_2,・・・・,x_n} のk次の基本対称式,
相乗平均 (x_1・x_2・・・・・x_n)^(1/n) = (Sn)^(1/n) =u.
0 = Σ[k=0,n] (n-k-1)S_k
≧ Σ[k=0,n] (n-k-1)C[n,k]u^k   (相加相乗平均)
= Σ[k=0,n] (n-k)C[n,k]u^k −Σ[k=0,n] C[n,k]u^k
= nΣ[k=0,n-1] C[n-1,k]u^k −Σ[k=0,n] C[n,k]u^k
= n{(1+u)^(n-1)} −(1+u)^n
= (n-1-u){(1+u)^(n-1)}.
∴ u/(n-1) ≧ 1.
ぬるぽ
426419:04/10/01 16:56:12
>424
相加相乗の関係について
 a,b,c,・・・・≧0, r,s,t,・・・・≧0, r + s + t + ・・・・・・ = 1 のとき
 a・r + b・s + c・t + ・・・・・ ≧ (a^r)(b^s)(c^t)・・・・.

べき r,s,t,・・・・∈Q(有理数)の場合、通分すれば相加相乗平均に帰する。
べき r,s,t,・・・・∈R(実数)の場合は、連続性による。(やや面倒)
427132人目の素数さん:04/10/01 21:30:22
>426
(;´Д`)ハァハァ /lァ/lァ //ア//ア!!

文献の紹介をキボンヌ。
428426:04/10/02 01:36:26
>427
 ↓岩波数学辞典の付録の公式の不等式のところ。

 a_1,a_2, ・・・・・ ,a_n≧0 のとき ・・・・・ また重みつき平均値について,
 納ν=1,n] λ_ν・a_ν ≧ Π[ν=1,n] a_ν^λ_ν (納ν=1,n] λ_ν =1, λ_ν>0).
 (略証)y=Ln(x) が上に凸であることからJensenの定理を使って導く。
 ぬるぽ
429426:04/10/02 02:02:12
 y=exp(x) が下に凸であることから導いてもよい。
430425:04/10/02 14:27:30
>416
a_k/1998 = x_k とおく。
【補題】(Klamkin,1974)
 x_k>1 ⇒ 納k=1,n] 1/(1+x_k) ≧ n/(1+u), ただし u = (x_1・x_2・・・・・・x_n)^(1/n):相乗平均。
(略証)nに関する帰納法による。
 x_kがすべて等しいときは明らかに成立するので、 u は x_n と x_{n-1} の間にあるとしてよい。
 いま y_n =u, y_{n-1} =x_n・x_{n-1}/u とおくと 積は不変で、和は
x_{n-1} + x_n - y_{n-1} - y_n = (u-x_{n-1})(x_n-u)/u ≧0 だけ減少する。
∴ 1/(1+y_{n-1}) + 1/(1+y_n) ≦ 1/(1+x_{n-1}) + 1/(1+x_n).
 {x_1 ・・・・ x_{n-2}, y_{n-1}}の n-1 個で考えると、相乗平均はu.
∴帰納法の仮定(n-1)により、
 納k=1,n] 1/(1+x_k) ≧ {納k=1,n-2] 1/(1+x_k) + 1/(1+y_{n-1})} +1/(1+u)
 ≧ (n-1)/(1+u) + 1/(1+u)
 = n/(1+u).
 ぬるぽ
431132人目の素数さん:04/10/02 20:07:46
(;´Д`) ハァハァ /lァ/lァ /lア/lア!!

参考文献をキボンヌ!
432132人目の素数さん:04/10/02 20:29:35
(Klamkin's Inequality)

-1 < x,y,z < 1 のとき、
  1/{(1-x)(1-y)(1-z)} + 1/{(1+x)(1+y)(1+z)} ≧ 2
433132人目の素数さん:04/10/03 00:53:28
>432
 相加相乗ハァハァと・・・
 左辺 ≧ 2/√{(1-x^2)(1-y^2)(1-z^2)} ≧ 2.
 ぬるぽ
434132人目の素数さん:04/10/03 02:55:14
【問題】 正の数 a,b,cに対して、次式を証明せよ。

(1) \sqrt{(a^2b+b^2c+c^2a)(ab^2+bc^2+ca^2)}
   ≧ abc + \sqrt[3]{(a^3+abc)(b^3+abc)(c^3+abc)}

(2) \sqrt(a^4+b^4+c^4) + \sqrt(a^2b^2+b^2c^2+c^2a^2)
   ≧ \sqrt(a^3b+b^3c+c^3a) + \sqrt{(ab^3+bc^3+ca^3)

  ∧_∧
  ( ;´∀`) < 勃起しますた。 ハァハァ…
  人 Y /
 ( ヽ し
 (_)_)
435430:04/10/03 15:11:14
>416
(430の続き)
【補題】の条件を、 ∀(i≠j); x_i・x_j -1 >0 に緩めても成立する。
(略証) 1/(1+x_{n-1}) + 1/(1+x_n) = 1 - [x_{n-1}・x_n -1]/[1-(x_{n-1}+x_n)+x_{n-1}・x_n] は (x_{n-1}+x_n) について単調増加。
∴ 1/(1+y_{n-1}) + 1/(1+y_n) ≦ 1/(1+x_{n-1}) + 1/(1+x_n).

[416]では 1 > 1/(1+x_i) + 1/(1+x_j) = 1 - [x_i・x_j -1]/[(1+x_i)(1+x_j)] により x_i・x_j -1 >0.


>434
 (2) 相加相乗とコーシーでハァハァと・・・・・
 {√(a^4 +b^4 +c^4) + √[(ab)^2 +(bc)^2 +(ca)^2]}/2 ≧ {(a^4 +b^4 +c^4)[(ab)^2 +(bc)^2 +(ca)^2]}^(1/4) ≧ √(a^3・b +b^3・c +c^3・a)
 {√(b^4 +c^4 +a^4) + √[(ab)^2 +(bc)^2 +(ca)^2]}/2 ≧ {(b^4 +c^4 +a^4)[(ab)^2 +(bc)^2 +(ca)^2]}^(1/4) ≧ √(a・b^3 +b・c^3 +c・a^3)
 辺々たす。
 ぬるぽ
436387:04/10/03 16:08:07
>390
 f(0)=0, 0≦g(x)≦1 ですが、f(x)≦1 は不要。
 たとえば f(x)=arcsin(x) も可。
 大関信雄・大関清太 共著:「不等式への招待」,近代科学社 (1987) p.62の定理
437435:04/10/03 20:23:45
>434 (1) ハァハァ
 右辺 = abc・{1+[(a^2 +bc)(b^2 +ca)(c^2 +ab)/(abc)^2]^(1/3)} ≡ abc(1+u).
 左辺 = √{(abc)^2 +(a^2 +bc)(b^2 +ca)(c^2 +ab)} = abc・√{1+(u^3)}.
 u^3 = 8 +{bc/(a^2)+(a^2)/bc -2} +{ca/(b^2)+(b^2)/ca -2} +{ab/(c^2)+(c^2)/ab -2} ≧ 8.
∴ u ≧ 2.
 (左辺)^2 -(右辺)^2 = (abc)^2・{(1+u^3)-(1+u)^2} = (abc)^2・u(u+1)(u-2) ≧0.
∴ 左辺 ≧ 右辺.
 ぬるぽ
438132人目の素数さん:04/10/05 17:05:22
>430
>いま y_n =u, y_{n-1} =x_n・x_{n-1}/u とおくと 積は不変で、

(y_kの積)
= (x_kの積)^2/{x_1*x_2*u^(n-2)}
= (x_kの積)u^2/(x_1*x_2)
≠(x_kの積)
だと思うのですが…
439430:04/10/05 18:21:05
>438
分かりにくくてすまそ。全部書けば、
 y_k = x_k (1≦k≦n-2), y_{n-1} = x_{n-1}・x_n/u, y_n = u.
したがって
 y_{n-1}・y_n = x_{n-1}・x_n,  (y_kの積) = (x_kの積) ≡ u^n.
ってことでつ。
ぬるぽ
440132人目の素数さん:04/10/05 21:33:59
嗚呼、成程。
441132人目の素数さん:04/10/06 01:38:03
この機会に、根号のついた不等式でハァハァしちゃうぞ!

     凵@    ○   ∇ 、___,、´`゙;~、  ';冫 ☆
           ┏  ━ゝヽ''/  ≧ \━〆A!゚━━┓。
 ╋┓"〓┃  < ゝ\',冫。' |::::  \ ./ |゛△│´'´,.ゝ'┃.      ●┃ ┃┃
 ┃┃_.━┛ヤ━━━━━━|::::: (● (● |━━━━━━━━━  ━┛ ・ ・
        ∇  ┠─Σ-  ヽ::::... .ワ.....ノ  冫 そ',´; ┨'゚,。
           .。冫▽ <   ⊂     ./⊃     乙  ≧   ▽
         。 ┃   Σ   (⌒ゞ ,l, 、''  │   て く
           ┠─ム┼   ゝ,,ノ ノゝ. 、,, .┼ ァ Ζ┨ ミo''`
         。、゚`。、   i/   レ' o。了 、'' ×  个o
        ○  ┃   `、,~´+√ ▽   ',!ヽ.◇    o┃
            ┗〆━┷ Z,.' /┷━''o ヾo┷+\━┛,゛;
       ヾ   凵@              '、´    ∇
442132人目の素数さん:04/10/06 01:41:17
(1) [Carson's Inequality] a,b,c>0に対し
 \sqrt[3]{(a+b)(b+c)(c+a)/8} ≧ \sqrt{(ab+bc+ca)/3}

(2) a,b,c,d>0に対し
 \sqrt{(a^2+b^2+c^2+d^2)/4} ≧ \sqrt[3]{(abc+bcd+cda+dab)/4}

(3) a,b,c>0に対し
 (|a-b|+|b-c|+|c-a|)/3 + \sqrt[3]{abc} ≧ (a+b+c)/3

(4) [1998,Hong Kong] a,b,c>1に対し
 \sqrt{a-1} + \sqrt{b-1} + \sqrt{c-1} ≦ \sqrt{a(bc+1)}

(5) [1998 APMO] a,b,c>0に対し
 (1+ a/b)(1+ b/c)(1+ c/a) ≧ 2(1+ (a+b+c)/\sqrt[3]{abc})

(6) [1997 Latvia] a,b>0、nは自然数のとき、
 1/(a+b) + 1/(a+2b) + … + 1/(a+nb) < n/\sqrt{a(a+nb)}

(7) [1997 Hong Kong] a,b,c>0に対し
 abc(a+b+c+\sqrt{a^2+b^2+c^2})/{(a^2+b^2+c^2)(ab+bc+ca)} ≦ (3+\sqrt{3})/9

(8) [1999 Austria-Poland] a,b≧0に対し
 {(\sqrt[3]{a^2b}+\sqrt[3]{ab^2})/2}^(3/2)
  ≧ (a+\sqrt{ab}+b)/3
  ≧ (a+\sqrt[3]{a^2b}+\sqrt[3]{ab^2}+b)/4
  ≧ {(\sqrt{a}+\sqrt{b})/2}^2
443LettersOfLiberty ◆rCz1Zr6hLw :04/10/06 11:03:05
さて、
xを0より大きい実数とするとき、
Γ(x+1)≥(x/3)^x
となることを証明せよ。
444数学科布施 ◆FUSEz5Eqyo :04/10/06 13:27:14
このスレまじ勃起する
445132人目の素数さん:04/10/06 13:32:46
>443の馬鹿がまた荒らしとる!
446132人目の素数さん:04/10/06 13:49:19
俺は不等式ヲタの神々とひっそりとハァハァしたいのに、
糞kingは、糞レスしかできないくせにage荒らししやがる!

>>194-204
447132人目の素数さん:04/10/06 15:37:26
 | |            | |            |┃| :|
 | |            | |            |┃|i | />>408 ̄ ̄ ̄ ̄ ̄ ̄ ̄ ̄ ̄ ̄ ̄
 | |            | |      ガタガタ |┃| < nCrと聞いちゃ、黙っていられ・・・・
 | |            | |______|ミ | .i.| | あれ?開かない・・・?
 | | ̄ ̄ ̄ ̄ ̄ ̄| |            |┃|:. ,| \______________
 | |            | |            |┃| i|
 | |            | |            |┃| :|
 | |            | |            |┃|i |
 | |            | |            |┃|, :.|
 |_|====――●==|_|______|┃| i|_______
448132人目の素数さん:04/10/06 15:43:13
>>408
示すべき不等式を変形すると (2^n-1)/n < Σ[k=0,n]C[n,k]/(2k+1)
>>411のその証明を真似ると、
 Σ[k=0,n]C[n,k]/(2k+1) < Σ[k=0,n]C[n,k]/(k+1) = … (以下同様)
結局
 (2^n-1)/n < Σ[k=0,n]C[n,k]/(2k+1) < (2^{n+1}-1)/(n+1)
          ___
    |┃三 ./  nCr \   ________
    |┃   |::::  \ ./ |  / もっといい
    |┃ ≡|::::: (● (●| <  評価式はできないかなぁ…
____.|ミ\_ヽ::::... .∀....ノ  \   ハァハァ /lァ/lァ  
    |┃=__    \     ̄ ̄ ̄ ̄ ̄ ̄ ̄ ̄ ̄
    |┃ ≡ )  人 \ ガラッ
449132人目の素数さん:04/10/06 16:48:04
自然数nに対して

Σ[k=0,n]C[n,k]/(2k+1)
 ≦ Σ[k=0,n]C[n,k]/(k+2)
 = ∫[0,1]Σ[k=0,n]C[n,k]x^{k+1}dx
 = ∫[0,1]x(1+x)^kdx
 = (部分積分して)
 = (n*2^{n+1}+1)/(n+1)(n+2)
    ___
  ./  nCr \   まだ いけるかなぁ
  |::::  \ ./ | ハァハァ
  |::::: (● (● |
  ヽ::::... .∀....ノ /  チン ☆
 _(  ⊃  ⊃  チン ☆
 |\ ̄ ̄ ̄ ̄旦 ̄\
 | | ̄ ̄ ̄ ̄ ̄ ̄ ̄|
 \|  愛媛みかん |
450132人目の素数さん:04/10/06 20:22:56
>442
 (8)の初めの不等号は a or b→0 の場合は成り立たな伊予柑....
451132人目の素数さん:04/10/07 04:31:33
 | |            | |            |┃| :|
 | |            | |            |┃|i | />>442,450 ̄ ̄ ̄ ̄ ̄ ̄ ̄ ̄
 | |            | |      ガタガタ |┃| < 確かにAB=0のとき成り立たないね、(8)番
 | |            | |______|ミ | .i.| | あれ?開かない・・・?
 | | ̄ ̄ ̄ ̄ ̄ ̄| |            |┃|:. ,| \_ (8)は1993年の問題で、…
 | |            | |            |┃| i|
 | |            | |            |┃| :|
 | |            | |            |┃|i |
 | |            | |            |┃|, :.|
 |_|====――●==|_|______|┃| i|_______
452442(8)の訂正:04/10/07 04:38:51
>450 グッジョブ
(8)は引用元の問題が 既に誤植でした。
別のところから探してきて確認したら、正しい形は以下の通りです。

(8) [1993 Austria-Poland] a,b≧0に対し
 {(\sqrt[3]{a^2}+\sqrt[3]{b^2})/2}^(3/2)
  ≧ (a+\sqrt{ab}+b)/3
  ≧ (a+\sqrt[3]{a^2b}+\sqrt[3]{ab^2}+b)/4
  ≧ {(\sqrt{a}+\sqrt{b})/2}^2

         ∧_∧  それでは、不等式solverの皆様方、
         (´Д` )  よろしくお願いします。 
         /  y/  ヽ  
    Σ(m)二フ ⊂[_ノ      とりあえず、死んでお詫びを…  
        (ノノノ | | | l )
453132人目の素数さん:04/10/07 04:58:07
>444
不等式に勃起する君も、今日から不等式ヲタだ!

  ∧_∧
  ( ;´∀`) < 勃起しますた。 ハァハァ…
  人 Y /
 ( ヽ し
 (_)_)
454132人目の素数さん:04/10/08 01:43:02
>443
 Γ(x+1) = xΓ(x) = x∫_[t=0,∞) t^(x-1) e^(-t) dt
  ≧ x∫_[t=0,x] t^(x-1) e^(-t) dt
  ≧ e^(-x)∫_[t=0,x] x・t^(x-1) dt
  = e^(-x) [t^x](t=0,x)
  = e^(-x) (x^x)
  = (x/e)^x.
ぬるぽ
455442(2)に追加:04/10/08 05:46:03
>>442(2)
a,b,c,d>0に対し
 \sqrt{(abc+bcd+cda+dab)/4} ≧ \sqrt[3]{(ab+ac+ad+bc+bd+cd)/6}
456132人目の素数さん:04/10/08 05:52:30
>>455
f(x)=(x-a)(x-b)(x-c)(x-d)=0 は重解を含めて4個の正の解をもつ。
y=f'(x)のグラフを考えれば、f'(x)=0も重解を含めて3個の正の解 α,β,γ をもつ。
解と係数の関係を用いて、示すべき不等式を α,β,γ で表すと
  (αβ+βγ+γα)/3 ≧ \sqrt[3]{(αβγ)^2}
となる。これは相加相乗平均の関係だから不等式は示された。
等号成立条件は α=β=γ で、このとき y=f(x) のグラフを考えて a=b=c=d.
(*゚∀゚)=3
457442に追加:04/10/08 06:00:38
もひとつルートの不等式追加。

(9) 実数 a,b が ab>0 をみたすとき、
 \sqrt[3]{a^2b^2(a+b)^2/4} ≧ (a^2+10ab+b^2)/12
458132人目の素数さん:04/10/08 08:37:31
ちょっと話し変わるけど、
http://www.kalva.demon.co.uk/short/soln/sh01a3.html

この不等式の証明で、CS不等式を使った後
 √{Σ(b_i)^2} ≦ 1
をどうやって示すのですか? リンク先の最後の行
 (b_1)^2 ≦ (x_1)^2/{1+(x_1)^2} = 1 - 1/{1+(x_1)^2}
の意味は分かりますが、b_2,… について同様にしても得られないのですが。
459132人目の素数さん:04/10/08 10:00:59
       |   _,.. -‐"/ ̄/  /|  ̄ l ヽ  \~`"'ー、ノ   >458をたのも〜♪
       ケフ" / /  ,.-'‐ ̄/ .i   .i  ̄\- \ \ヾ
      / /.l l l .// / ./  l  /    ヾ  iヽ  i.\     たのも〜♪
      ノ  | l l l Y /"¨''ヽ .i / ァ''"¨ヾ i イ゙i. リ
       `ヽ r、 丶l i`       レ       |  イ/"
         \ ヽ  ヽ """  iー'ーv'  """ /  '
          ヽ ヾ- ゝ    ._/   ./
460高1連立不等式:04/10/08 11:21:12
16x+260-13x+20≦300
を整理すると・・
3x≦20

整理途中の計算を教えて下さい
マジレス宜しく御願いしますm(__)m
461132人目の素数さん:04/10/08 11:22:41
そういう質問は質問スレに書け!
462132人目の素数さん:04/10/08 11:26:09
ワロタ
463LettersOfLiberty ◆rCz1Zr6hLw :04/10/08 11:36:44
Re:>460
不等式の基本事項:
a≤bかつc≤dならば、a+c≤b+d.
任意のcに対して、a≤bとa+c≤b+cは同値。(*)
0<aかつ0<bならば、0<ab.
任意のc>0に対して、a≤bとca≤cbは同値。
(*)により、
16x+260-13x+20≤300
⇔16x+260-13x+20-280≤300-280.
464高1連立不等式:04/10/08 11:51:20
>>463ご丁寧に説明いただき有り難う御座いました
やっと、解く事が出来ました。

465132人目の素数さん:04/10/08 12:19:22
>458
(b_i)^2 = (x_i)^2/[1+(x_1)^2 +(x_2)^2 +...+(x_i)^2]^2
≦ (x_i)^2/[1+(x_1)^2 +(x_2)^2 +...+(x_{i-1})^2][1+(x_1)^2 +(x_2)^2 +...+(x_i)^2]
= 1/[1+(x_1)^2 +(x_2)^2 +...+(x_{i-1})^2] - 1/[1+(x_1)^2 +(x_2)^2 +...+(x_i)^2].
 ∴ Σ[i=1,n] (b_i)^2 < 1.
 と読むんだろうな。
ぬるぽ
466132人目の素数さん:04/10/08 15:53:04
ぬるぽ神キタ━(゚∀゚)━!!!

そうか、その手があったか!! ありがとうございますです!!!
   ┏┓  ┏━━┓        / ̄ ̄ ̄ ̄ ̄ ̄\          .┏━┓┏━┓
 ┏┛┗┓┃┏┓┃        /     ≧     \       ┃  ┃┃  ┃
 ┗┓┏┛┃┗┛┃┏━━━/               ヽ.━━━┓┃  ┃┃  ┃
 ┏┛┗┓┃┏┓┃┃      l:::::::::    \    /    |     ┃┃  ┃┃  ┃
 ┗┓┏┛┗┛┃┃┗━━━|::::::::::   (●)   (●)   |━━━┛┗━┛┗━┛
   ┃┃      ┃┃        |:::::::::::::::::   \___/    |       ┏━┓┏━┓
   ┗┛      ┗┛      ヽ::::::::::::::::::.  \/     ノ      ┗━┛┗━┛
467132人目の素数さん:04/10/08 23:48:58
>457
 (9)の不等号は逆向きの悪寒....
468457の訂正:04/10/09 00:01:13
>467 仰せのとおりにございます。

(9) 実数 a,b が ab>0 をみたすとき、
 \sqrt[3]{a^2b^2(a+b)^2/4} ≦ (a^2+10ab+b^2)/12

  ,、|,、
 (f⌒i
  U j.|
  UJ
   :
  ‐=‐
469132人目の素数さん:04/10/09 22:49:28
>442 (1〜5)
(1) (a+b+c)^2 - 3(ab+bc+ca) = {(a-b)^2+(b-c)^2+(c-a)^2}/2 ≧0.
  (abc)^(2/3) ≦ (ab+bc+ca)/3 ・・・・ 相加相乗平均
  ∴ (左辺)^3 = {(a+b+c)(ab+bc+ca)-abc}/8 = (1/8)[(√3)-1/(3√3)](ab+bc+ca)^(3/2) ={(ab+bc+ca)/3}^(3/2) = (右辺)^3.

(2) 左側: (a^2 +b^2 +・・・・)/n - {(a+b+・・・・)/n}^2 = {1/(n-1)n}{[a^2+b^2] + ・・・ -(1/n)[a^2+b^2+2(n-1)ab] -・・・・}
   = (1/n)^2 {[a^2 +b^2 -2ab] + ・・・・}
   = (1/n)^2 {(a-b)^2 +・・・・} ≧0.
  右側: {(a+b+・・・・)/n}^3 - (abc + ・・・・)/C[n,3]
   = (1/n)^3 {a^3 +b^3 +・・・・ +3(a^2b+b^2c+c^2a) +3(ab^2 +bc^2 +ca^2) + ・・・・ +6(abc+・・・・)} - (abc+・・・・)/C[n,3]
   = {1/n^2C[n,3]}{(1/3)[a^3 +b^3 +c^3]・・・・+ ((n-1)/2)[a^2b+b^2c+c^2a] +((n-1)/2)[ab^2 +bc^2 +ca^2] + ・・・・+((n-1)(n-2)-n^2)(3n-2)(abc+・・・)}
   = {1/n^2C[n,3]}{(1/3)[a^3 +b^3 +c^3 -3abc]・・・・+ ((n-1)/2)[a^2b+b^2c+c^2a-3abc] +((n-1)/2)[ab^2 +bc^2 +ca^2 -3abc]+・・・・・} ≧0.

(3) u≧min(a,b,c) のとき、左辺 = 2(MAX-min) +3u ≧ 2MAX +min ≧ a+b+c =右辺.

(4) b,c≧1のとき、{√(b-1) +√(c-1)}^2 = bc - {√(b-1)√(c-1) -1}^2 ≦ bc.
  ∴ √(b-1) +√(c-1) ≦√(bc), 等号成立は 1/b +1/c =1.
  同様に、a≧1, bc≧0 のとき {√(a-1) +√(bc)} ≦√{a(bc+1)}, 等号成立は 1/b + 1/c = a-1.

(5) 相加平均 ≧ 相乗平均 ≧ 調和平均 より
   (1/3)(a+b+c)(1/a +1/b +1/c) -1 = (1/3){(a/b +b/a)+(b/c +c/b)+(c/a +a/c)} ≧ 2.
(2/3)(a+b+c)(1/a +1/b +1/c) ≧ 2(a+b+c)/[(abc)^(1/3)].
  辺々たす。
  ぬるぽ
470132人目の素数さん:04/10/09 22:57:09
>442 (6〜7)
(6) nに関する帰納法による。
  n=1のときは明らかに成立
  (n+1)/√[a(a+(n+1)b)] - n/√[a(a+nb)] = [(n+1)/f -n]/√[a(a+nb)] = cd/[a+(n+1)b]
  f = √{[a+(n+1)b]/(a+nb)} > 1,
  c = (2n+1)/[(n+1)/f +n] > 1,
  d = {a+(n/2)[1+1/(2n+1)]b}/√[a(a+nb)] > 1.
  ∴ 1/[a+(n+1)b] < (n+1)/√[a(a+(n+1)b)] - n/√[a(a+nb)].

(7) a^2 +b^2 +c^2 = (1/3){(a+b+c)^2 +(a-b)^2 +(b-c)^2 +(c-a)^2} ≧ (1/3)(a+b+c)^2.
  (a+b+c)(ab+bc+ca) = 9abc + a(b-c)^2 + b(c-a)^2 +c(a-b)^2 ≧ 9abc.
  ∴ abc{(a+b+c)/{(a^2 +b^2 +c^2)(ab+bc+ca)} ≦ abc{(a+b+c)/{(1/3)(a+b+c)^2 (ab+bc+ca)}
  = 3abc/{(a+b+c)(ab+bc+ca)} ≦ 3/9.
  abc{√(a^2 +b^2 +c^2)/{(a^2 +b^2 +c^2)(ab+bc+ca)} ≦ abc/{(1/√3)(a+b+c)(ab+bc+ca)}
  = (√3)abc/{(a+b+c)(ab+bc+ca)} ≦ (√3)/9.
  辺々たす。
  ぬるぽ
471132人目の素数さん:04/10/09 22:58:39
>442 (8〜9)
(8) a^(1/6) =A, b^(1/6)=B とおく。
  上: {[a^(2/3) + b^(2/3)]/2}^3 - {[a+√(ab)+b]/3}^2
  = (1/8){A^4 +B^4}^3 - (1/9){A^6+(AB)^3+B^6}^2
  = (1/8){A^12 +3(A^2B)^4 +3(AB^2)^4 +B^12} - (1/9){A^12 +2(A^3B)^3 +3(AB)^6 +2(AB^3)^3 +B^12}
= (1/72){A^12 -16A^9B^3 +27A^8B^4 -24A^6B^6 + 27A^4B^8 -16A^3B^9 +B^12)
  = (1/72)(A-B)^4{(A^4 -B^4)^2 +AB(4A^2 +10AB +4B^2)(A^4 +B^4)} ≧0.

  中: [a+√(ab)+b]/3 - [a^(1/3) +b^(1/3)][a^(2/3) +b^(2/3)]/4
  = [A^6 +(AB)^3 +B^6] - [A^2 +B^2][A^4 +B^4]/4
  = [A^6 -3A^4B^2 +4(AB)^3 -3A^2B^4 +B^6]/12
  =(A-B)^2 {A^4 +2A^3B +2AB^3 +B^4}/12 ≧0.

  下: [a^(1/3) +b^(1/3)][a^(2/3) +b^(2/3)]/4 - [(√a +√b)/2]^2
  = (1/4)[a^(1/3) +b^(1/3) -2(ab)^(1/6)](ab)^(1/3)
  = (1/4)[A^2 + B^2 -2AB](AB)^2
= (1/4)(A-B)^2 (AB)^2 ≧ 0.

 なお、{(a^r +b^r)/2}^(1/r) はrについて単調増加
  (略証) r<R とすると、y=x^(R/r) は下に凸だから、
  {(a^r +b^r)/2}^(R/r) ≦ (a^R +b^R)/2, {(a^r +b^r)/2}^(1/r) ≦ {(a^R +b^R)/2}^(1/R).

(9) (ab+ab+c^2)/3 ≧ (abc)^(2/3) ・・・・ 相加相乗平均
  c=(a+b)/2 とおく。
  ぬるぽ
472132人目の素数さん:04/10/09 23:13:08
神キタ━(゚∀゚)━!!! いつもながら流石でございます。
ありがとうございます。 今夜はタップリ抜けそうです。

 ∧_∧
( ;´∀`)とにかく一発!    
人 Y /        
( ヽωつ ο°o。 
(_)_)
473469:04/10/09 23:26:51
(2)は次式の略証でつ・・・
 a,b,c,・・・・・>0 に対し
 √{(a^2 +b^2 +c^2 +・・・・・)/n} ≧ (a+b+c+・・・・・)/n ≧ {(abc+・・・・・)/C[n,3]}^(1/3).

[455]は
 \sqrt[3]{(abc+bcd+cda+dab)/4} ≦ \sqrt{(ab+ac+ad+bc+bd+cd)/6} ?
474132人目の素数さん:04/10/09 23:40:28
>473
嗚呼、またもや書き間違い。
[455]ですが 仰せのとおりにございます。

          r〜〜〜〜〜
   __    _ノ うっうっうっ・・・
  /__  `ヽ_ ⌒ヽ〜〜〜〜〜
  |〈___ノf レ1(
 ,L| しL.し'゙"
 "`  "′
475cubism:04/10/10 00:30:03
↓分かスレ188から借用

404 :132人目の素数さん :04/10/09 15:59:40
                                     ,/|ミ=、
                                     /  .|ミミミ|
                                    .|    |ミミミ|
                                  ,/|ミ|   |ミミミ|
                                ,/  |ミ|   |ミミミ|
                                  |    |ミ|   |ミミミ|
                                  |    |ミ|   |ミミミ|
                                  |    |ミ|    |ミミミ|
                                  |    |ミ|    |ミミミ|
                                  |    |ミ|    |ミミミ|
                               _,-'"|.   |ミ|   |ミミミ|
             _,. -'' " ̄~゙三=-_、_    _,.-'"   |.   |ミ|   !ミミミ|
          ,,.-''" r _、      三三タ_,.-''"      |   |ミ|  ,.彡ヾミ|
        /    i {ぃ}}       _ニ/        -=三|  」ミヒ彡彡イミヾ
       /,.、     `--"      ニl     -=ニ三=-''レ彡ミミr'"   |ミミミ|
       l {ゞ}    i        .ニl==三三ニ=''"   ,>'"|ミ|    |ミミミ|
      .l `" i_,,...-''|           ニ`=-=i'"       |   |ミl,..-=彡ヾミ|
     _,.-!    !  i         -ニ三三/         L.. -ニヾ|ヾ彡'='''"
      l´,.- l    \/        -ニ三三/        ヾ-‐''"
  _.  ! ri l\       __--三三三='"
  j'‘´l `´ | !  ` ミ三三三三三=''"
 i',.. '´}  | |
  l,.. r´   '´
476132人目の素数さん:04/10/10 01:01:50
世俗と離れた山奥で、ひっそりとやっている感じが たまらなく (゚∀゚) イイッ!
477132人目の素数さん:04/10/10 07:10:14
不等式の証明問題って、両辺の文字の次数が一致するものばかりだと思っていた。
一致しなくても、積一定などの与えられた条件を使って書き換えたら一致するのとか。
そうじゃないのもあるって今ごろ知った。

【問題】 正の数 a,b,c に対して、次の不等式を示せ。
  1/{a(1+b)} + 1/{b(1+c)} + 1/{c(1+a)} ≧ 3/(1+abc)
478132人目の素数さん:04/10/10 08:55:57
>>430の補題の仮定 x_k>1 は、x_k≧1 でもOKですよね?
479430:04/10/10 15:52:54
>478

>>118-129 も嫁
480473:04/10/10 16:11:36
>455 (474)
 (k次の基本対称式)/C[n,k] = P_k とおくと、問題は (P_3)^(1/3) ≦ (P_2)^(1/2).

【定理】
 相加平均 = P_1 ≧ (P_2)^(1/2) ≧ (P_3)^(1/3) ≧ ・・・・・ ≧ (P_n)^(1/n) = 相乗平均 ・・・・(1).
 (略証) 補題の(2)をk乗して掛けあわせ、共通項を約せばよい。

【補題】
 P_{k-1}・P_{k+1} ≦ (P_k)^2 ・・・・・・・・・・・・・・ (2).
 (略証) f(x) ≡ (x-a)(x-b)・・・・ = 納L=0,n] ((-1)^L)・C[n,L]・P_L・x^(n-L)
     g_k(x) ≡ {x^(k+1)}・(d/dx)^(n-k-1) f(x) = {n!/(k+1)!}・納L=0,k+1] ((-1)^L)・C[k+1,L]・P_L・x^L
     h_k(x) ≡ (d/dx)^(k-1) g(x) = ((-1)^(k+1))・(n!/2)[P_{k-1} -2P_k・x +P_{k+1}・x^2].
     f(x)=0 の根はすべて正の実数だから、ロルの定理により、g_k(x), h_k(x) についても同様である。
     h_k(x)=0 は実根をもつから、判別式Q_k = (P_k)^2 - P_{k-1}・P_{k+1} ≧0. (終)

【参考書】
 「数学の問題=第(1)集」, 第21問 日本評論社 (1977)
 E.F.Beckenbach and R.Bellman: "Inequalities", p.11, Ergebnisse叢書, Springer Verlag (1961)
 ↑ >>84-86 の本

【別法】nに関する帰納法
  >>263-271
 ぬるぽ
481132人目の素数さん:04/10/10 16:39:31
【参考書】
ビブンのことはビブンでする向きには
 G.H.Hardy, J.E.Littlewood, G.Polya共著, : 「不等式」, シュプリンガー・フェアラーク東京
     細川尋史 訳, A5, 450p. \4800, ISBN 4-431-71056-6
 D.S.Mitrinovic': "Analytical Inequalities", Springer-Verlag (1970)
482132人目の素数さん:04/10/11 19:57:11
>>442(7) は、最小値の方は出ないでしょうか?
483132人目の素数さん:04/10/11 21:19:56
>482
 a,b,cのうち1つだけ→0 のとき →0 と思われ...
 ぬるぽ
484132人目の素数さん:04/10/13 04:14:43
Mを正定数, a,b,cを実定数とする。
f(x)=ax^2+bx+c において |f(-1)|, |f(0)|, |f(1)|≦M ならば、
|x|≦1において |f(x)|≦5M/4 を示せ。
こやつめをたのもー。

a=0 のときは f(x) は一次式以下だから、区間の端点で最大最小値をとるので
|f(x)| ≦ max{|f(-1)|, |f(1)|} = M

a>0のときを考える。(a<0のときも同様にできる)
このとき f(x) は下に凸だから、グラフを考えると
|f(x)| ≦ max{|f(-1)|, |f(-b/{2a})|, |f(1)|} = \max{M, |f(-b/{2a})|}

ということで、|f(-b/{2a})|≦5/4 を示せれば一件落着だと思うのだけど、
そのあとが分からんちん。
485132人目の素数さん:04/10/13 06:14:32
訂正
>ということで、|f(-b/{2a})|≦5M/4 を示せれば…

軸の位置によって、|-b/(2a)|>1のときは
|f(x)| ≦ max{|f(-1)|, |f(1)|} ≦ M

|-b/(2a)|≦1のときは、D=b^2-4ac≧0のときには、
|f(-b/{2a})| の最大値を考えないといけないんだけど
|f(-b/{2a})| = |-b^2/(4a)+c| ≦ (|b|/2)*|b/2a|+|c| ≦ (M/2)*1+M = 3M/2

ここで、|2b|=|f(1)-f(-1)|≦2M、|c|=|f(0)|≦M を用いた。
5M/4にならんのですが…。




486 ◆BhMath2chk :04/10/13 08:00:00
 |f(x)|
=|f(−1)x(x−1)/2+f(0)(1−x^2)+f(1)x(x+1)/2|
≦M(|x(x−1)/2|+|1−x^2|+|x(x+1)/2|)。

−1≦x≦0のとき
 |x(x−1)/2|+|1−x^2|+|x(x+1)/2|
=1−x−x^2
≦5/4。

0≦x≦1のとき
 |x(x−1)/2|+|1−x^2|+|x(x+1)/2|
=1+x−x^2
≦5/4。
487132人目の素数さん:04/10/13 10:05:09
>486
なるほど、その手がありましたか! 激しくありがとうございます。
係数 a,b,c を f(-1), f(0), f(1) で表すんですね。

この条件の下で、同様に考えると |x|≦1において |f'(x)|≦4M
また、g(x)=cx^2+bx+a を考えると、|x|≦1において |g(x)|≦9M/4, |g'(x)|≦3M

となりますね。たぶん。
488132人目の素数さん:04/10/13 10:09:58
[1992 MeXico]
正の数 a,b,c が a+b+c=3 をみたすとき、次の不等式を示せ。
6 < \sqrt{2x+3}+\sqrt{2y+3}+\sqrt{2z+3} ≦ 3\sqrt{5}

右側は Cauchy-Schwarz でも Jensen の不等式でも得られますが、
左側は一体どうやるのでせうか? たのも〜。
489132人目の素数さん:04/10/13 11:30:30
>487の訂正。
ぬーん、|g(x)|≦2M なのか…。

類題発見。京大1995後期理系3番
http://hw001.gate01.com/akiyoshi/pdf/95ks.pdf

はっきり言って、a≧0, b≧0 の条件は要らない。
絶対値がついているから a の符号はどっちでもいいし、
x=t,-tの場合を考えれば、bの符号もどうとでもなるし。

|f(x)|≦M の条件は強すぎると思う。
3箇所、例えば |f(-1)|, |f(0)|, |f(1)| ≦ M で十分じゃないの?
490132人目の素数さん:04/10/13 13:33:42
>488
 √ は上に凸ゆえ 0<x<3 ⇒ (3+x)/2 < √(2x+3)
 辺々たす。
(3+2√3 も可能)
ぬるぽ
491132人目の素数さん:04/10/13 16:38:13
>490
ありがとうございまする。

0<x<3 において、y=(3+x)/2 と y=√(2x+3) のグラフを考えるんですね?
y=(3+√3)x/3 +√3 と y=√(2x+3) のグラフから、3+2√3 を出したんですね

     |
 \  __  /
 _ (m) _ピコーン
    |ミ|
 /___\ 
 ./  ≧ \ 
 |::::  \ ./ | そうか!
 |::::: (● (● | 
 ヽ::::... .ワ.....ノ
492132人目の素数さん:04/10/15 06:43:55
(1) 正の数 a,b,c が abc=1 をみたすとき、
 (1+a)(1+b)(1+c) ≧ 2(1+\sqrt[3]{b/a}+\sqrt[3]{c/b}+\sqrt[3]{a/c})

(2) x>0 に対し、
 [(x+ 1/x)^6-(x^6+ 1/{x^6})-2]/[(x+ 1/x)^3+(x^3+ 1/{x^3})] ≧ 6

(3) 1≦a≦b≦c≦4のとき、
 (a-1)^2 + (b/a -1)^2 + (c/b -1)^2 + (4/c -1)^2 ≧ 12-8√2
         ___
    |┃三 ./  nCr \   ________
    |┃   |::::  \ ./ |  /
    |┃ ≡|::::: (● (●| <  ハァハァ /lァ/lァ
____.|ミ\_ヽ::::... .∀....ノ  \   
    |┃=__    \     ̄ ̄ ̄ ̄ ̄ ̄ ̄ ̄ ̄
    |┃ ≡ )  人 \ ガラッ
493132人目の素数さん:04/10/15 12:17:01
>492
(1) 相加相乗平均より (b/a)^r +(c/b)^r +(a/c)^r ≧3.
左辺 = -1+(b+1 +1/a)+(c+1 +1/b)+(a +1/c) ≧ -1 +3{(b/a)^(1/3) +(c/b)^(1/3) +(a/c)^(1/3)}
≧ 2 + 2{(b/a)^(1/3) +(c/b)^(1/3) +(a/c)^(1/3)} = 右辺.

(2) x^6 +1/(x^6) +2 = {x^3 +1/(x^3)}^2 より,
 左辺 = (x+ 1/x)^3 - {x^3+ 1/(x^3)} = t^3 -(t^2 -3)t = 3t ??

(3) Σ[k=1,n](x_k)^2 - (1/n){Σ[k=1,n] x_k}^2 = (1/n)Σ[i≠j] (x_i-x_j)^2 ≧0 より
 左辺 ≧ 4{(a/1 +b/a +c/b +4/c)/4 -1}^2 ≧ 4([4^(1/4)] -1}^2 = 4(√2 -1)^2 = 4(3-2√2) = 右辺.
ぬるぽ
494493:04/10/15 12:19:58
訂正
(2) 左辺 = (x+ 1/x)^3 - {x^3+ 1/(x^3)} = 3(x +1/x) ≧ 6.
495132人目の素数さん:04/10/15 19:47:10
>492
(1) [442]の(5)と同じ...

(3) r>1 のとき y=x^r は下に凸ゆえ、 {Σ[k=1,n](x_k)^r}/n ≧ {(Σ[k=1,n] x_k)/n}^r.
 ∴ Π[k=1,n] a_k =P ⇒ Σ[k=1,n] (a_k -1)^r ≧ n{(1/n)(Σ[k=1,n] a_k) -1}^r ≧ n{P^(1/n) -1}^r
496132人目の素数さん:04/10/15 20:20:16
>493-495
(゚∀゚) うひょっ! ありがとうございます。
(1)が442(5)と同じことに気づかなかったです…。
497132人目の素数さん:04/10/17 10:32:00
(1) 0≦a,b,c,d≦1 と 0≦x≦1 に対して
 1/|x-a| + 1/|x-b| + 1/|x-c| + 1/|x-d| < 40

(2) [ASU 1969.14] 正の数 a_k に対して
 a_1/(a_2+a_3) + a_2/(a_3+a_4) + … + a_n/(a_1+a_2) ≧ n/4
    ___
  ./  ≧ \   (2)を見ると Shapiro の巡回不等式を思い出すけど…。
  |::::  \ ./ | ハァハァ
  |::::: (● (● |
  ヽ::::... .∀....ノ /  チン ☆
 _(  ⊃  ⊃  チン ☆
 |\ ̄ ̄ ̄ ̄旦 ̄\
 | | ̄ ̄ ̄ ̄ ̄ ̄ ̄|
 \|  愛媛みかん |
498132人目の素数さん:04/10/17 10:46:50
(3) [Ukrine 1992]
a≧b≧c>0 のとき
 (a^2-b^2)/c + (b^2+c^2)/a + (c^2-a^2)/b ≧ 3a-4b+c

(4) a≧b≧c≧d>0, a+b+c+d≧1 のとき
 7a^2+5b^2+3c^2+d≧1

(5) [Vietnam 1980]
正の数 a_k が a_1+ … +a_n=s をみたすとき
 {a_1+ 1/(a_1)}^2 + … +{a_n+ 1/(a_n)}^2 ≧ n{n/s + s/n}^2
499132人目の素数さん:04/10/17 23:51:40
>498
(3) 因数定理より (a^2 -b^2)/c + (b^2 -c^2)/a +(c^2 -a^2)/b = (a-b)(b-c)(a-c)(a+b+c)/abc.
  (2/3)a = b = c のとき 左辺 = 2(c^2)/a = (8/9)a < a = 右辺. むずい??

(4) a≧d≧0, b≧c≧0 より、左辺 ≧ 4(a^2 +b^2 +c^2 +d^2) ≧ (a+b+c+d)^2 ≧ 1.

(5) r≧1 ⇔ (x_1)^r +(x_2)^r +・・・・・+(x_n)^r ≧ n{(x_1 +x_2 +・・・・・+ x_n)/n}^r
  調和≦相加 より (1/n){1/(a_1) + 1/(a_2) +・・・・+ 1/(a_n)} ≧ n/s.
ぬるぽ
500132人目の素数さん:04/10/18 14:32:30
>499(4) その手があったか、さすが! /lァ/lァ
501132人目の素数さん:04/10/19 21:40:20
>497
(2) 左辺をSとおく。
 a/(b+c) +2b/(c+d) - {(a+b)/(b+c) + (b+c)/(c+d) -1} = (b^2 +cd)/[(b+c)(c+d)] >0.
 ∴ a/(b+c) +2b/(c+d) > (a+b)/(b+c) + (b+c)/(c+d) -1. 
 循環的に加えて相加・相乗平均を使えば
 3S > 2Σ[k=1,n] (a_k +a_{k+1})/(a_{k+1} +a_{k+2}) -n > 2n-n = n.
 ただし a_{n+1}=a_1, a_{n+2}=a_2 とした。
 ∴ S≧ n/3.
ぬるぽ
502132人目の素数さん:04/10/20 07:13:38
>501 a/(b+c) +2b/(c+d) > (a+b)/(b+c) + (b+c)/(c+d) -1
       不等式神キタ━(゚∀゚)━!!!
こんな不等式、自力では逆立ちしても思いつきませんです。 (;´Д`) ハァハァ

               ゝ_i/ / // / | l / ! | l i   〉' ⌒⌒_/
               「ミ| / /ィ'「´! ! l l 」___| l |l _,くミ 、_/ ̄´
               〈 j | ハ{ ,l-ニ!、 | l | ,|=、|ヽ|l/! 'ヽ,-ァ
               ヽ } j' {i'r':j!   ,.=、ヾ!/    r'
                | i' ,, ゞ='   i!-':i! i!'    /
  ┃   ┏━┃       { ト、  r‐‐- 、_゙''=' /  /  /.     ┃┃┃
━┏┛ ┏━┃ ━━━━━━ト、   ノ ━━━━━━━┛  ┃┃┃
━┏┛ ┛  ┃      .〉/ / ( `ー-ァ' .r「 / /  /.        ┛┛┛
  ┛       ┃       l/ /_____ゝ<´  l | ∧ '  i/        ┛┛┛
                ∧--‐‐‐''"ヽヽ_! |  〉  /
               /ト、   @/  ヽ_/ /`-/
               / ! ヽ  { r.‐‐‐┐ / /
              j     @ l !(゚∀゚)l / /
503132人目の素数さん:04/10/20 12:38:57
[477] 正の数 a,b,c に対して、次の不等式を示せ。
  1/{a(1+b)} + 1/{b(1+c)} + 1/{c(1+a)} ≧ 3/(1+abc)

(解1) 次式を整理すれば得られる。 こんなもの思いつかんわい!
ab(1+b)(1-ca)^2+bc(1+c)(1-ab)^2+ca(1+c)(1-ab)^2 ≧ 0

(解2) 相加相乗平均の関係を用いる。 簡単だが気づかんわい!
(1+abc)(左辺)+3
= {(1+abc)/(a+ab)+1} + {(1+abc)/(b+bc)+1} + {(1+abc)/(c+ca)+1}
= {(1+a)/(a+ab) + (b+bc)/(1+b)} + {(1+b)/(b+bc) + (c+ca)/(1+c)} + {(1+c)/(c+ca) + (c+ca)/(1+a)}
= {(1+a)/(a+ab) + (a+ab)/(1+a)} + {(1+b)/(b+bc) + (b+bc)/(1+b)} + {(1+c)/(c+ca) + (c+ca)/(1+c)}
≧ 6

(解3) 並べ替え不等式 (同順序積)≧(逆順序積) を用いる。かっこよすぎ!
(1+abc)(左辺)
= (1/a)*{1/(1+b)}+b*{1/(1+ 1/a)} + (1/b)*{1/(1+c)}+c*{1/(1+ 1/b)} + (1/c)*{1/(1+a)}+a*{1/(1+ 1/c)}
≧ (1/a)*{1/(1+ 1/a)}+b*{1/(1+b)} + (1/b)*{1/(1+ 1/b)}+c*{1/(1+c)} + (1/c)*{1/(1+ 1/c)}+a*{1/(1+a)}
= 1/(1+a) + a/(1+a) + 1/(1+b) + b/(1+b) + 1/(1+c) + c/(1+c)
= 3

(元ネタ) http://www.komal.hu/verseny/2003-09/A.e.shtml
         ___
    |┃三 ./  nCr \   ________
    |┃   |::::  \ ./ |  /
    |┃ ≡|::::: (● (●| <  ハァハァ /lァ/lァ
____.|ミ\_ヽ::::... .∀....ノ  \   
    |┃=__    \     ̄ ̄ ̄ ̄ ̄ ̄ ̄ ̄ ̄
    |┃ ≡ )  人 \ ガラッ
504501:04/10/20 21:41:05
>503
【補題】a,b,c>0 のとき、u=(abc)^(1/3)とおくと
   1/{a(1+b)} + 1/{b(1+c)} + 1/{c(1+a)} ≧ 3/{u(1+u)}.

 (略証)(解2)と同様に 1+abc = (1+a) -a(1+b) +ab(1+c). 
 ∴ (1+abc)/{a(1+b)} = (1+a)/{a(1+b)} -1 +b(1+c)/(1+b). 
 循環的に加えて相加・相乗平均を使えば 
 (1+abc)(左辺) ≧ 3(1/u -1 +u) = 3(1+u^3)/{u(1+u)}
 ∴ 左辺 ≧ 3/{u(1+u)} (終). 

なお、3/{u(1+u)} ≧ 3/(1+u^3) = 3/(1+abc) は (1+u^3)-u(1+u) = (1-u)(1-u^2) ≧0 から明らか。
505132人目の素数さん:04/10/20 22:00:01
             .  + .  *     / ̄ ̄ ̄ ̄ ̄ ̄\                 ■ ■
 ■      ■■■     .      /   _ノ ≧  ,_ノ\   .+  ☆  .      ■ ■
■■■■  ■  ■           /    / iニ)ヽ,   /rj:ヽヽ ヽ              ■ ■
  ■    ■  ■ ■■■■■■■l::::::::: ;〈 !:::::::c!  ' {.::::::;、! 〉 .|■■■■■■■■  ■ ■
.■■■■ ■ ■■           |::::::::::  (つ`''"   `'ー''(つ   |             ■ ■
   ■     ■  +.  ☆  。. . |:::::::::::::::::   \___/    | ☆ . *  +.
   ■     ■            ヽ:::::::::::::::::::.  \/     ノ  .  . .   +☆  .● ●
506132人目の素数さん:04/10/21 13:41:11
[442](2) a,b,c,d>0に対し
 \sqrt{(a^2+b^2+c^2+d^2)/4} ≧ \sqrt[3]{(abc+bcd+cda+dab)/4}

ここで、次式が成り立つ。
 (a+b+c+d)/4 ≧ \sqrt{(ab+bc+cd+da)/4}

ということで、次の2つの大小関係は定まりますか?
 \sqrt[3]{(abc+bcd+cda+dab)/4}, \sqrt{(ab+bc+cd+da)/4}
507132人目の素数さん:04/10/21 13:48:36
あぁ書き忘れた。もう一回書き直すと
[473](2)と[506]の真ん中より、a,b,c,d>0 に対し
 \sqrt{(a^2 +b^2 +c^2 +d^2)/4} ≧ (a+b+c+d)/4 ≧ \sqrt[3]{(abc+bcd+cda+dab)/4}
 (a+b+c+d)/4 ≧ \sqrt{(ab+bc+cd+da)/4}

右側の2つは比較できないかなと言うことでした。
508132人目の素数さん:04/10/21 14:24:32
>506-507
>>455 (474), >>480 の辺りにないか?
ぬるぽ
509508:04/10/21 18:01:28
>506
Q. 次の2つの大小関係は定まりますか?
  {(abc+bcd+cda+dab)/4}^(1/3), sqrt{(ab+bc+cd+da)/4}

A. 定まらないと思われ...
 a=b≠c=d のとき, 左辺 = {ac(a+c)/2}^(1/3) < (a+c)/2 = 右辺.
 a=c≠b=d のとき, 左辺 = {ab(a+b)/2}^(1/3) > sqrt(ab) = 右辺.
 a=b=c≠d のとき, 左辺 = {a^2・(a+3d)/4}^(1/3) < sqrt{a(a+d)/2} = 右辺.
 ∵ {a・[(a+3d)/4]^2}^(1/3) < (a+d)/2. より {a^(1/2)・(a+3d)/4}^(1/3) < sqrt{(a+d)/2}.
ぬるぽ
510132人目の素数さん:04/10/21 21:59:25
なるほど、ありがとうございまする。
511132人目の素数さん:04/10/22 08:27:32
[>>455] の不等号の根号の中身は逆ですね。正しくは
 \sqrt{(ab+ac+ad+bc+bd+cd)/6} ≧ \sqrt[3]{(abc+bcd+cda+dab)/4}

死んでお詫びを…(AA略) まとめると、こんな感じですか。

A = \sqrt{(a^2 +b^2 +c^2 +d^2)/4}
B = (a+b+c+d)/4
C = \sqrt{(ab+ac+ad+bc+bd+cd)/6}
D = \sqrt[3]{(abc+bcd+cda+dab)/4}
E = \sqrt{(ab+bc+cd+da)/4}
F = \sqrt[4]{abcd}

A ≧ B ≧ C ≧ D ≧ F
    B ≧ E    ≧ F

CとE、DとEの大小は定まらない。
512132人目の素数さん:04/10/22 16:16:30
あげ
513480:04/10/22 20:30:31
>511 ついでに
 C ≧ {(C^4)/B}^(1/3) ≧ D ≧ {C(F^2)}^(1/3) ≧ F.

>492,503
 なぜかnCrヲタがやって来る...
ぬるぽ
514132人目の素数さん:04/10/23 02:15:00
a,b,c,d,p,q,r,s>0 かつ a + b + c + d = 1 かつ p + q + r + s = 1 のとき,
a log(a/p) + b log(b/q) + c log(c/r) + d log(d/s) ≧ 0 を示せ.
515514:04/10/23 02:16:05
log は自然対数.
516132人目の素数さん:04/10/23 05:40:00
>515
下に凸なf(x)=log(1/x)に対して、Jensenの不等式を用いると
af(a/p)+bf(b/q)+cf(c/r)+df(d/s)
≧ f(a*(p/a)+b*(q/b)+c*(r/c)+d*(s/d))
= f(p+q+r+s)
= 0          ___
    |┃三 ./  ≧ \   ________
    |┃   |::::  \ ./ |  / 不等式と聞いちゃぁ
    |┃ ≡|::::: (● (●| <  黙っちゃゐられねゑ…
____.|ミ\_ヽ::::... .∀....ノ  \   ハァハァ /lァ/lァ  
    |┃=__    \     ̄ ̄ ̄ ̄ ̄ ̄ ̄ ̄ ̄
    |┃ ≡ )  人 \ ガラッ
517132人目の素数さん:04/10/23 05:53:20
>513
さすが不等式神ッ! 常に一歩先を行くぅ〜。そこに痺れる憧れるぅ〜。

3変数でやると、A ≧ B ≧ C ≧ D が成立。ただし

A = (a+b+c)/3
B = \sqrt[3]{(a+b)(b+c)(c+a)/8}
C = \sqrt{(ab+bc+ca)/3}
D = \sqrt[3]{abc}





(AAの額が nCr だったのは、言われるまで気づかなかったミス。
不等式ヲタ = nCrヲタ = 三角関数ヲタ = 関数方程式ヲタ なのは公然の秘密)
518132人目の素数さん:04/10/23 07:16:09
>>516
お見事!
519132人目の素数さん:04/10/23 07:28:47
【問題A】 正の数 a,b,c が ab+bc+ca=1 をみたすとき

(1) [1994 Hong Kong]
a(1-b^2)(1-c^2)+b(1-c^2)(1-a^2)+c(1-a^2)(1-b^2) ≦ (4√3)/9

(2)
a/(1+a^2) + b/(1+b^2) + c/(1+c^2)
≧ 2a(1-a^2)/{(1+a^2)^2} + 2b(1-b^2)/{(1+b^2)^2} + 2c(1-c^2)/{(1+c^2)^2}


【問題B】 正の数 a,b,c が abc=1 をみたすとき

(3) [1997 Romania]
(a^3+b^3)/(a^2+ab+b^2) + (b^3+c^3)/(b^2+bc+c^2) + (c^3+a^3)/(c^2+ca+a^2) ≧ 2

(4) [2000 Hong Kong] さらに a≧b≧c のとき
(1+ab^2)/(c^3) + (1+bc^2)/(a^3) + (1+ca^2)/(b^3) ≧ 18/(a^3+b^3+c^3)


【問題C】 正の数 a,b,c が a^2+b^2+c^2=s をみたすとき

(5) [1991 Poland] s=2のとき、a+b+c ≦ 2+abc

(6) [1999 Belarus] s=3のとき、1/(1+ab) + 1/(1+bc) + 1/(1+ca) ≧ 3/2

 | |            | |            |┃| :|
 | |            | |            |┃|i | / 条件不等式と聞いちゃあ
 | |            | |      ガタガタ |┃| <  黙っちゃゐられねゑ…
 | |            | |______|ミ | .i.| | ? 開かない・・・?
 | | ̄ ̄ ̄ ̄ ̄ ̄| |            |┃|:. ,| \
 | |            | |            |┃| i|
520513:04/10/23 16:08:27
>517
 そこまで言われるとつい....でに
  A ≧ B ≧ (AC^2)^(1/3) ≧ C ≧ {(C^4)/A}^(1/3) ≧ D.

(略証)相加・相乗平均を使う。
 A = (1/2){(a+b)+(b+c)+(c+a)}/3} ≧ B

 A ≧ D, C ≧D より
 B^3 = (a+b)(b+c)(c+a)/8 = [(a+b+c)(ab+bc+ca) -abc]/8 = (9AC^2 -D^3)/8 ≧ AC^2,
 ∴ B ≧ (AC^2)^(1/3).

 A^2 = (1/9)(a+b+c)^2 ≧ (ab+bc+ca)/3 = C^2.
 ∴ (AC^2)^(1/3) ≧ C ≧ {(C^4)/A}^(1/3).

 C^4 = {(ab+bc+ca)/3}^2 = {(x+y+z)/3}^2 ≧ (xy+yz+zx)/3 = [(a+b+c)/3]abc = A(D^3),
 ∴ {(C^4)/A}^(1/3) ≧ D.
ぬるぽ
521問題追加:04/10/23 21:50:40
【問題D】正の数 a,b,c に対して

(7) [1997 Ireland] a+b+c≧abc のとき、a^2+b^2+c^2 ≧ abc



>520 キタ━(゚∀゚)━!!!
522追加と疑問?:04/10/24 06:17:45
【問題B】 に追加。 正の数 a,b,c が abc=1 をみたすとき

(8) [1997 TOT]
1 ≧ 1/(1+a+b) + 1/(1+b+c) + 1/(1+c+a)

(9) [1997 Bulgaria]
1/(2+a) + 1/(2+b) + 1/(2+c) ≧ 1/(1+a+b) + 1/(1+b+c) + 1/(1+c+a)

この2つの不等式は、一つにまとめられそうな予感。でも、どうなるんでしょう?
         ___
       ./   ≧ \   神降臨待ち
       |::::  \ ./ | ハァハァ
       |::::: (● (● |
       ヽ::::... .∀....ノ /  チン ☆
      _(  ⊃  ⊃  チン ☆
      |\ ̄ ̄ ̄ ̄旦 ̄\
      | | ̄ ̄ ̄ ̄ ̄ ̄ ̄|
      \|  愛媛みかん |
         ̄ ̄ ̄ ̄ ̄ ̄ ̄
523132人目の素数さん:04/10/24 08:22:43
Youngの不等式の多変数バージョンってあったっけ?
524132人目の素数さん:04/10/24 09:09:12
>523 見たことないです。

>522 (8)(9) できた。
正の数 a,b,c が abc=1 をみたすとき、次式が成立。
1 ≧ 1/(2+a) + 1/(2+b) + 1/(2+c) ≧ 1/(1+a+b) + 1/(1+b+c) + 1/(1+c+a)

〔証明〕
a+b+c=x, ab+bc+ca=y, abc=1 を用いると、示すべき不等式は
 1 ≧ (4x+y+12)/(4x+2y+9) ≧ (x^2+4x+y+3)/(x^2+xy+2x+y)
ただし、相加平均・相乗平均の関係から x, y≧3 に注意する。

(左側) 示すべき不等式は y≧3 だから成立。

(右側) x-3=s, y-3=t とおくと s, t≧0。分母を払って差をとると
(4x+y+12)(x^2+xy+2x+y)-(4x+2y+9)(x^2+4x+y+3)
= 3x^2y+xy^2+6xy-5x^2-y^2-24x-3y-27
= 3s^2t+st^2+30st+4s^2+2t^2+27s+54t
≧ 0
     |
 \  __  /
 _ (m) _ピコーン
    |ミ|
 /___\ 等号成立条件は、いずれも a=b=c=1 のとき。
 ./  ≧ \ 
 |::::  \ ./ | 簡単でした。
 |::::: (● (● | 
 ヽ::::... .ワ.....ノ
525501:04/10/24 10:42:43
>497 (1)
 0≦a≦b≦c≦d≦1 は所与とする。
 (1+a)-d, d-c, c-b, b-a の和が1だから
 Max{(1+a)-d, d-c, c-b, b-a} = w ≧1/4.
 そこで xとして 幅wの区間の中点をとると、
 Min{|x-a|,|x-b|,|x-c|,|x-d|} = w/2 ≧ 1/8.
 左辺 < 8+8+8+8 = 32.
ぬるぽ
526132人目の素数さん:04/10/24 19:49:31
   ___   >525 グッジョブ!
 ./  ≧ \    いつもながら素晴らしい!結局こうですね。
 |::::  \ ./ |  
 |::::: (● (● |    4 ≦ 1/|x-a| + 1/|x-b| + 1/|x-c| + 1/|x-d| ≦ 32
 ヽ::::... .ワ....ノ    n  
 ̄ ̄   \    ( E)
フ     /ヽ ヽ_//
527132人目の素数さん:04/10/25 10:32:44
>>525
いや、やっぱり分かりません。
たとえば a = b = c = d = 0 のとき、
 (左辺) < 4/|x| → ∞ (x→0)
だから、いくらでも大きくなるような気がします。
528132人目の素数さん:04/10/25 12:42:54
>525 2-3行目は
1-d, d-c, c-b, b-a, a-0 の和が1だから
 Max{1-d, d-c, c-b, b-a, a} = w ≧1/5
とすべきでは?
まだ問題の意味が分かってないけれど…
529132人目の素数さん:04/10/25 13:44:48
いや、525でよかった。すみません。
530132人目の素数さん:04/10/26 08:02:16
>519 (3〜7)

【問題B】 abc=u≧1 のとき
(3) 3(a^2 -ab+b^2) - (a^2 +ab+b^2) = (a-b)^2 +(b-c)^2 +(c-a)^2 ≧0.
 ∴ (a^3 +b^3)/(a^2 +ab+b^2) ≧ (a+b)/3.
 ∴ 左辺 ≧ 2(a+b+c)/3 ≧2u^(1/3).

(4) 左辺 ={1/(a^3)+1/(b^3)+1/(c^3)} + {a(b^2)/(c^3) +b(c^2)/(a^3) +c(a^2)/(b^3)}
  ≧ 3/u +1 ≧ 6/u ≧ 右辺.

【問題C】a^2 +b^2 +c^2 =s のとき
(5)
(i) a,b,c≦1 のとき、
 左辺 = 3-(1-a)-(1-b)-(1-c) ≦ 3-(1-a)-a(1-b)-ab(1-c) = 3 -(1-abc) =右辺.
(ii) a,b≦1<c,s≦2 のとき、x = {1+x^2 - (1-x)^2}/2 より
 左辺 = {3+s-(1-a)^2 -(1-b)^2 -(c-1)^2}/2 = 1 +s/2 +ab -{(a+b-1)^2 +(c-1)^2}/2
= 1 +s/2 +abc -{(a+b-1)^2 +(c-1)^2 +ab(c-1)}/2 ≦ 1 +s/2 +abc ≦ 右辺.

(6) 左辺 ≧ 9/(3+ab+bc+ca) ≧ 9/(3+s) = 右辺.

【問題D】(a+b+c)/(abc)=k のとき
(7) 左辺 ≧ 3{(a+b+c)/3}^2 ≧ √{3(a+b+c)(abc)}=√(3k)・abc
ぬるぽ
531501=525:04/10/26 09:19:52
>526
 まだまだ改良できると思われ...
 1/4≦w≦1/2 のとき: 幅wの区間の反対側の区間幅≦w, 残り2つの幅の合計x+y≧1-2w.
  左辺 = 2/(w/2) +1/{(w/2)+x} +1/{(w/2)+y}
(i) 1/4≦w≦1/3 のとき x,y≧1-3w
  左辺 ≦ 2/(w/2) +1/{(w/2)+w} +1/{(w/2)+(1-3w)} = (4 +2/3)/w +2/(2-5w) ≦ 64/3.
(ii) 1/3≦w≦1/2 のとき x,y≧0
  左辺 ≦ 3/(w/2) +1/{(w/2)+(1-2w)} = 6/w +2/(2-3w) ≦ 18+2 = 20.
(iii) 1/2<w のとき
  左辺 ≦ 4/(w/2) =8/w ≦ 16.
ぬるぽ
532132人目の素数さん:04/10/26 14:50:31
>530
神キタ━(゚∀゚)━!!!

(3) 書き間違いですね。 3(a^2 -ab+b^2) - (a^2 +ab+b^2) = 2(a-b)^2

(5) ですが、元の問題みたら、a,b,c の条件は実数でした。 死んでお詫びを…。

(7) そんな手があるとは…。

むずぽ
533530:04/10/26 21:54:44
>532
(3)は仰せのとおり。 死んでお詫びを...(AA略)

(5)の修正でつ
 (i) -1≦a,b,c≦1 のときは変更なし...
 (ii) -1≦a,b≦1<c のとき、d≡ a+b-1 = ab-(1-a)(1-b)≦ab より
  右辺 - 左辺 = {(a+b-1)^2 +(c-1)^2 +ab(c-1)}/2 ≧ {d^2 +(c-1)^2 +(c-1)d}/2 ≧0.
 (iii) c<-1≦a,b≦1 のとき、 左辺 = (a+b+|c|) +2c ≦ 2 +(ab-2)|c| ≦ 2-ab|c| = 右辺.

(7)は 相加・相乗平均 {(a+b+c)/3}^(3/2) ≧ √{abc} を使いますた。 ハァハァ
ぬるぽ
534533:04/10/27 08:04:30
(5) またまた修正
 (ii) 右辺 - 左辺 = (1 +s/2 +abc) - (a+b+c) = (1/2)(a+b+c-2)^2 + (1-a)(1-b)(c-1) ≧ 0.
すまそ
535132人目の素数さん:04/10/27 12:24:51
>534
>右辺 - 左辺 = (1/2)(a+b+c-2)^2 + (1-a)(1-b)(c-1)

すげー! こんな変形 気づきません。(;´д`)ハァハァ
536132人目の素数さん:04/10/28 14:21:43
正の数 a, b, c が a^2+b^2+c^2=1 をみたすとき、次式の最小値をキボンヌ。
 (a^5)/(b+c) + (b^5)/(c+a) + (c^5)/(a+b)

巡回的に対称だから、a≧b≧c または a≧c≧b としてよい。
 前者のとき a/(b+c)≧b/(c+a)≧c/(a+b)、
 後者のとき a/(b+c)≧c/(a+b)≧b/(c+a) だから、
チェビシェフの不等式により、どちらも次の同じ不等式を得る。
 与式 ≧ (1/3)(a^4+b^4+c^4){a/(b+c) + b/(c+a) + c/(a+b)}

3(a^4+b^4+c^4)-(a^2+b^2+c^2) = (a^2-b^2)^2+(b^2-c^2)^2+(c^2-a^2)^2 ≧ 0 より
 a^4+b^4+c^4 ≧ (1/3)(a^2+b^2+c^2)^2 = 1/3

あとは、a/(b+c) + b/(c+a) + c/(a+b) の最小値が分かれば…。 たのも〜!

          r〜〜〜〜〜〜〜〜〜〜〜
   __    _ノ このあと、どうすれば・・・
  /__  `ヽ_ ⌒ヽ〜〜〜〜〜〜〜〜〜〜〜
  |〈___ノf レ1(
 ,L| しL.し'゙"
 "`  "′
537536:04/10/28 14:44:50
自己解決。 Jensenで瞬殺だった。
538132人目の素数さん:04/10/28 15:14:18
ついでに条件不等式を投下。 [>>519(1)(2)]もたのも〜。

(1) [1996 Poland]
a, b, c≧-3/4、a+b+c=1 のとき、a/(1+a^2) + b/(1+b^2) + c/(1+c^2) ≦ 9/10

(2) [1998 Poland]
a, b, c, d, e, f>0、a+b+c+d+e+f=1、ace+bdf≧1/108 のとき、abc+bcd+cde+def+efa+fab ≦ 1/36

も一つおまけに絶対不等式を投下。

(3) [1992 Poland]
実数 a, b, c に対して、(a+b-c)^2(b+c-a)^2(c+a-b)^2 ≧ (a^2+b^2-c^2)(b^2+c^2-a^2)(c^2+a^2-b^2)

         | |‖│||
    ┌― | |‖│|| ―┬────
    |   | |‖│||   |
    |             | | ̄ ̄ ̄    不等式と聞いちゃあ
    |   / ̄ ̄∨ヽ.  |  |          黙っちゃゐられねゑ…
    |   /     ∨.  | |___
    |  /___________ヽ  |ガシャン
    |  / | \/_|ヽ  |
    |  | |  ゚| □| \.|  ← 不等式ヲタ
    |  | |  ゚|  |\__|つ
    |  | |  ゚|  |  |
539538:04/10/28 15:23:52
書き忘れ。上の問題はここにありまつ。模範解答はないけど…。(;´д`)ハァハァ
http://www.mimuw.edu.pl/~chel/Olimp/archive-om.html
540132人目の素数さん:04/10/28 19:26:22
>538
(2) [1998 Poland] 49th, 2nd round, 1st day(1998.2.27), No.3a
 a+d>0, b+e>0, c+f>0, a+b+c+d+e+f=s, ace+bdf=u のとき、相加相乗平均より
 左辺 =(a+d)(b+e)(c+f) -(ace+bdf) ≦ {[(a+d)+(b+e)+(c+f)]/3}^3 -(ace+bdf)=(s/3)^3 -u.
 http://www.mimuw.edu.pl/~chel/Olimp/2etang98.html

ついでに >>519
【問題A】 ab+bc+ca=t とおく。
(1) t≧1, a+b+c≧√(3t), abc≦(t/3)^(3/2) を使って
  左辺 = (a+b+c)-a(ca+ab)-b(ab+bc)-c(bc+ca)+abct = (1-t)(a+b+c) + (3+t)abc
  ≧ (1-t)√(3t) + (3+t)・(t/3)^(3/2) = √(3t)・(1-t/3)^2.
ぬるぽ
541501=504:04/10/28 20:19:36
>536
 よけいなお世話だが...
(解1) b+c=A, c+a=B, a+b=C とおくと a=(B+C-A)/2, b=(C+A-B)/2, c=(A+B-C)/2.
 ∴ a/(b+c) = (B/A + C/A -1)/2.
 循環的に加えて相加・相乗平均を使えば、
 左辺 ≧ (3+3-3)/2 = 3/2.

(解2)通分して a(c+a)(a+b) +b(a+b)(b+c) +c(b+c)(c+a) - (3/2)(b+c)(c+a)(a+b)
  = (1/2){(a-b)(a^2 -b^2) +(b-c)(b^2 -c^2) +(c-a)(c^2 -a^2)} ≧0.
ぬるぽ
542132人目の素数さん:04/10/28 23:55:27
>541 神キタ━(゚∀゚)━!!!
ありがとうございまする。解法のコレクションが増えました。
今更ながら >537 に書いた方法は…

a+b+c=s とおくと 0<s≦√6。 この範囲で任意に s を固定する。
f(x) = x/(s-x) = s/(s-x)-1 は 0<x<s において下に凸だから、

a/(b+c) + b/(c+a) + c/(a+b) = f(a)+f(b)+f(c) ≧ 3f((a+b+c)/3) = 3f(s/3) =3/2

となって、a=b=c (=1/√3) で最小値をとる。
543132人目の素数さん:04/10/29 02:56:10
[>>519(2)] について…。

正の数 a, b, c が ab+bc+ca=1 をみたすとき
a/(1+a^2) + b/(1+b^2) + c/(1+c^2) ≧ 2a(1-a^2)/{(1+a^2)^2} + 2b(1-b^2)/{(1+b^2)^2} + 2c(1-c^2)/{(1+c^2)^2}

条件式から得られるものは、
(a+b+c)^2 ≧ 3(ab+bc+ca) = 3
a^2+b^2+c^2 ≧ ab+bc+ca = 1
1 = {(ab+bc+ca)/3}^(3/2) ≧ abc > 0

右辺の分子を 1-a^2 = (1+a^2)-2a^2 と変形して整理すると、示すべき不等式は
(4a^3)/(1+a^2)^2 + (4b^3)/(1+b^2)^2 + (4c^3)/(1+c^2)^2 ≧ a/(1+a^2) + b/(1+b^2) + c/(1+c^2) … (A)

右辺の分子を 1-a^2 = 2-(1+a^2) と変形して整理すると、示すべき不等式は
3a/(1+a^2) + 3b/(1+b^2) + 3c/(1+c^2) ≧ 4a/(1+a^2)^2 + 4b/(1+b^2)^2 + 4c/(1+c^2)^2 … (B)

(A), (B) のどちらか一方が示せればいいんだけど…。 むずぽ。
544132人目の素数さん:04/10/29 03:06:40
a=cot(A),b=cot(B),c=cot(C)を満たす鋭角三角形ABCを考えたら?
545543の続き:04/10/29 03:38:17
(A) の左辺にチェビシェフの不等式を用いると、
(Aの左辺)
= {(4a^2)/(1+a^2)}*{a/(1+a^2)} + {(4b^2)/(1+b^2)}*{b/(1+b^2)} + {(4c^2)/(1+c^2)}*{c/(1+c^2)}
≧ (1/3)*{(4a^2)/(1+a^2) + (4b^2)/(1+b^2) + (4c^2)/(1+c^2)}*{a/(1+a^2) + b/(1+b^2) + c/(1+c^2)}
となるから、次が示されれば…。
(1/3)*{(4a^2)/(1+a^2) + (4b^2)/(1+b^2) + (4c^2)/(1+c^2)} ≧ 1

(A) の左辺に、上とは別の方法でチェビシェフの不等式などを用いると、
(Aの左辺)
= 4a*{a/(1+a^2)}^2 + 4b{b/(1+b^2)}^2 + 4c*{c/(1+c^2)}^2
≧ (1/3)*(4a+4b+4c)*[{a/(1+a^2)}^2 + {b/(1+b^2)}^2 + {c/(1+c^2)}^2]
≧ (1/3)*(4a+4b+4c)*(1/3)*{a/(1+a^2) + b/(1+b^2)+ c/(1+c^2)]^2
≧ (4\sqrt{3}/9)*{a/(1+a^2) + b/(1+b^2)+ c/(1+c^2)]^2
となるから、次が示されれば…。
(4\sqrt{3}/9)*{a/(1+a^2) + b/(1+b^2)+ c/(1+c^2)] ≧ 1

どっちも むずぽ。
546132人目の素数さん:04/10/29 03:39:35
>>544
下書きしているうちに レスが…。
ありがとうございます、考えてみまする。
547132人目の素数さん:04/10/29 03:53:56
>540
> 左辺 =(a+d)(b+e)(c+f) -(ace+bdf)

この変形に勃起しました。 (;´д`)ハァハァ
548540:04/10/29 20:54:45
>538
 (1) [1996 Poland] 47th, 2nd round, 1st day(1996.2.23), No.3
 曲線 y=x/(1+x^2) 上の点(1/3, 3/10)で接線を引く: y= (9/50)(1/3 +4x).
 x≧-3/4 ⇒ x/(1+x^2) = (9/50)(1/3 +4x) - (18/25)(x+3/4)(x-1/3)^2 ≦ (9/50)(1/3 +4x).
 x=a,b,c について加えれば、 左辺 ≦ (9/50){1+4(a+b+c)}.
 http://www.mimuw.edu.pl/~chel/Olimp/2etang96.html
ぬるぽ

>519 (1) 下の方の不等号が逆向き、すまそ。
549132人目の素数さん:04/10/30 10:06:27
このスレ、まるで初等幾何のスレだな。
違うのは図が無いところだけ。
550132人目の素数さん:04/10/30 11:46:19
>>549
それは簡単に言うと馬鹿にしているのですか?
551132人目の素数さん:04/10/30 18:28:53
不等式を制する者は、解析を制する。
不等式は下からの評価が結構難しい。
552working woman:04/10/30 18:34:40
マニアックな不等式をいくら積み重ねてもしようが無いわね。
553132人目の素数さん:04/10/30 19:12:52
L^2 とか、uniform space の不等式を積み重ねなさいよ。
554132人目の素数さん:04/10/31 01:03:17
             L -‐ '´  ̄ `ヽ- 、   〉
          /           ヽ\ /
        //  /  /      ヽヽ ヽ〈
        ヽ、レ! {  ム-t ハ li 、 i i  }ト、
         ハN | lヽ八l ヽjハVヽ、i j/ l !
         /ハ. l ヽk== , r= 、ノルl lL」
        ヽN、ハ l   ┌‐┐   ゙l ノl l
           ヽトjヽ、 ヽ_ノ   ノ//レ′
    r777777777tノ` ー r ´フ/′
   j´ニゝ        l|ヽ  _/`\
   〈 ‐ 書き込み lト、 /   〃ゝ、
   〈、ネ..         .lF V=="/ イl.
   ト |  と思ったら  ニヽ二/  l
   ヽ.|l.        〈ー-   ! `ヽ.  
      |l 荒らしでした lトニ、_ノ    ヾ、
      |l__________l|   \    ソ
555132人目の素数さん:04/10/31 01:05:54
>>554
それもking信者による荒らし。
working woman をNGワードしる!
556132人目の素数さん:04/10/31 11:19:09
解けない人の妬みにしか聞こえんな。
557132人目の素数さん:04/10/31 11:19:34
>>519 (2)
 [544]の続き。 a=cot(A), b=cot(B), c=cot(C), 0<A,B,C<π/2.
 A+B+C=π より
 左辺 = (1/2){sin(2A)+sin(2B)+sin(2C)} = 2sin(A)sin(B)sin(C).
 右辺 = (1/2){sin(4A)+sin(4B)+sin(2C)} = 2sin(2A)sin(2B)sin(2C).
 右辺/左辺 = 8cos(A)cos(B)cos(C)
 f(x)=cos(x) は [0,π/2) で正で上に凸なので、log|cos(x)| も上に凸(∵補題)
 ∴ 8cos(A)cos(B)cos(C) < 8{cos[(A+B+C)/3]}^3 = {2cos(π/3)}^3 = 1.

 【補題】f(x)≧0 が上に凸ならば log|f(x)| も上に凸。
 (略証){log|f(X)|} " = (f '/f) ' = {(ff " -(f ')^2}/(f^2) <0

[519] の解答のレス番(主なもの)
 (1) 540 (2) 544+556 (3)〜(7) 530 (8),(9) 524
ぬるぽ
558557:04/10/31 17:48:54
[557]の後半を修正
 右辺/左辺 = 8cos(A)cos(B)cos(C) ≦ {2[cos(A)+cos(B)+cos(C)]/3}^3
 y=cos(x) は [0,π/2) で上に凸なので、
 2[cos(A)+cos(B)+cos(C)]/3 ≦ 2cos[(A+B+C)/3] = 2cos(π/3) = 1.
すまそ
559132人目の素数さん:04/10/31 19:09:38
>519 (2) について
うひょっ。dクスです。自分なりに解決しました。またもや Jensen を使いました。

正の数 a, b, c が ab+bc+ca=1 をみたすとき
 a/(1+a^2) + b/(1+b^2) + c/(1+c^2) ≧ 2a(1-a^2)/{(1+a^2)^2} + 2b(1-b^2)/{(1+b^2)^2} + 2c(1-c^2)/{(1+c^2)^2}

>543(A) に書いたように同値変形して
 (4a^3)/(1+a^2)^2 + (4b^3)/(1+b^2)^2 + (4c^3)/(1+c^2)^2 ≧ a/(1+a^2) + b/(1+b^2) + c/(1+c^2)

>545前半に書いたように、左辺にチェビシェフの不等式を用いて
 (Aの左辺) ≧ (1/3)*{(4a^2)/(1+a^2) + (4b^2)/(1+b^2) + (4c^2)/(1+c^2)}*{a/(1+a^2) + b/(1+b^2) + c/(1+c^2)}

したがって、次式を示せばよい。
 (1/3)*{(4a^2)/(1+a^2) + (4b^2)/(1+b^2) + (4c^2)/(1+c^2)} ≧ 1

同値変形して、結局 ab+bc+ca=1 をみたす正の数 a, b, c に対して、次を示せばよい。
 1/(1+a^2) + 1/(1+b^2) + 1/(1+c^2) ≦ 9/4

f(x) = 1/(1+x) は x>0 において、下に凸な減少関数であることと、(a^2+b^2+c^2)/3 ≧ (ab+bc+ca)/3 =1/3 だから、
 f(a^2)+f(b^2)+f(c^2) ≦ 3f((a^2+b^2+c^2)/3) ≦ 3f(1/3) = 9/4
   ___
 ./  ≧ \
 |::::  \ ./ |
 |::::: (● (● | グッジョブ!Jensen
 ヽ::::... .ワ....ノ    n  
 ̄ ̄   \    ( E)
フ     /ヽ ヽ_//
560132人目の素数さん:04/11/01 03:19:45
>557-558 激乙。いつもありがとうございまする。
残るは >>538(3) ですね。またネタを探してきます。
561132人目の素数さん:04/11/01 09:22:23
後で見るときに分かりやすいだろうから、一気に出しておきます。
とりあえず分類したものから、ボコッと投下。

発掘元は、>>539 や以下のサイトなど。(解答のないものばかり)
http://www.math.nwu.edu/~mlerma/problem_solving/

     凵@    ○   ∇ 、___,、´`゙;~、  ';冫 ☆
           ┏  ━ゝヽ''/  ≧ \━〆A!゚━━┓。
 ╋┓"〓┃  < ゝ\',冫。' |::::  \ ./ |゛△│´'´,.ゝ'┃.      ●┃ ┃┃
 ┃┃_.━┛ヤ━━━━━━|::::: (● (● |━━━━━━━━━  ━┛ ・ ・
        ∇  ┠─Σ-  ヽ::::... .ワ.....ノ  冫 そ',´; ┨'゚,。
           .。冫▽ <   ⊂     ./⊃     乙  ≧   ▽
         。 ┃   Σ   (⌒ゞ ,l, 、''  │   て く
           ┠─ム┼   ゝ,,ノ ノゝ. 、,, .┼ ァ Ζ┨ ミo''`
         。、゚`。、   i/   レ' o。了 、'' ×  个o
        ○  ┃   `、,~´+√ ▽   ',!ヽ.◇    o┃
            ┗〆━┷ Z,.' /┷━''o ヾo┷+\━┛,゛;
       ヾ   凵@              '、´    ∇
562絶対不等式など:04/11/01 09:23:26
再掲 >>538(3) [1992 Poland]
実数 a, b, c に対して、(a+b-c)^2(b+c-a)^2(c+a-b)^2 ≧ (a^2+b^2-c^2)(b^2+c^2-a^2)(c^2+a^2-b^2)


(1) 複素数 a, b, c に対して、|\sqrt(a^2+b^2+c^2)| ≦ max(|a|+|b|, |b|+|c|, |c|+|a|)

(2) [1999 Poland] 実数 a, b, c, d に対して、(a+b+c+d)^2 ≦ 3(a^2+b^2+c^2+d^2)+6ab

(3) 正の数 a, b, c に対して、(a^2b+b^2c+c^2a)(ab^2+bc^2+ca^2) ≧ 9(abc)^2

(4) 正の数 a, b, c, d に対して、ab^4+bc^4+cd^4+da^4 ≧ abcd(a+b+c+d)

(5) [2003 Poland] 正の数 a, b, c, d に対して、(a+b+c+d)^3 ≦ 4(a^3+b^3+c^3+d^3)+24(abc+bcd+cda+dab)

(6) [2002 Poland] 正の数 a_k, b_k に対し、Π[k=1 to n]a_k + Π[k=1 to n]b_k ≦ Σ[k=1 to n]√{(a_k)^2+(b_k)^2}

(7) [1999 Poland] 整数 a_k, b_k に対して、 Σ[i<j](|a_i-a_j|+|b_i-b_j|) ≦ Σ[i<j]|a_i-b_j|
563条件不等式:04/11/01 09:24:10
(1) [2000 Poland] 正の数 a, b, c が a+b+c=1 をみたすとき、a^2+b^2+c^2+2√(3abc) ≦ 1

(2) [1996 Poland] a, b, c ≧ 0 と 1/2 ≧ p, q, r ≧ 0 が、a+b+c = p+q+r = 1 をみたすとき、pa+qb+rc ≧ 8abc

(3) [1999 Turkey] a≧b≧c≧0 に対して、(a+2c)(c+3b)(b+4a) ≧ 60abc

(4) [1991 Vietnum] a≧b≧c>0 に対して、b^2c/a + c^2a/b + a^2b/c ≧ a^2+b^2+c^2

(5) [1993 Itary] 0≦a, b, c≦1 に対して、a^2+b^2+c^2 ≦ a^2b+b^2c+c^2a+1

  類題に [1994 Rumania] 0≦a, b, c≦1 に対して、a+b+c ≦ ab+bc+ca+1 がありました。

(6) 0 < a, b, c < 1/2 に対して、(1/a -1)(1/b -1)(1/c -1) ≧ {3/(a+b+c) -1}^3

(7) 自然数 m, n と実数 0≦x≦1 に対して、(1-x^n)^m+{1-(1-x)^m}^n ≧1
564最大最小値問題:04/11/01 09:24:35
(1) 正の数 a, b, c に対して、(a^3+b^3+c^3)/(a+b)(b+c)(c+a) の最小値。

(2) 実数 a, b, c が a^2+b^2+c^2≠0 をみたすとき、 abc(a+b+c)/{(2a^2+b^2)(b^2+2c^2)} の最大値。

(3) a^2+b^2+c^2=1 をみたす実数 a, b, c と、非負実数 p, q, rに対して、次式の最大値と最小値。
 \sqrt{(pa)^2+(qb)^2+(rc)^2} + \sqrt{(pb)^2+(qc)^2+(ra)^2} + \sqrt{(pc)^2+(qa)^2+(rb)^2}

(4) 異なる実数 a, b, c が bc+ca ≧ 1+ab+c^2 をみたすとき、次式の最大値。ただし n は自然数。
 {(a-b)^(2n+1)+(b-c)^(2n+1)+(c-a)^(2n+1)}/{(a-b)(b-c)(c-a)}

(5)-(i) 非負実数 a, b, c が a+b+c=1 をみたすとき、(1+a^2)(1+b^2)(1+c^2) の最小値。
(5)-(ii) 非負実数 a, b, c が a+b+c=1 をみたすとき、(1+√a)(1+√b)(1+√c) の最小値。

(6) 非負実数 a, b, c と自然数 n に対して、常に次式が成り立つような定数 k の最小値。
 k(a^3+b^3+c^3)+(9-3k)abc ≧ (a+b+c)(a^2+b^2+c^2)
(1) [1963 Eotvos] 0<x<π/2 のとき、(1/(sin x)+1)(1/(cos x)+1) > 5

(2) [1978 Austria] tan k (k = 1度, …, 44度)の相加平均をA、相乗平均をGとおくとき、A > (√2)-1 > G

(3) [1959 IMO shortlist] 0≦x≦π/2、π/6<y<π/3 のとき、
tan{(π sin x)/(4 sin y)} + tan{(π cos x)/(4 cos y)} > 1

(4) 0<x<π に対して、{sin(x)/x}^3 < {(π^2-x^2)/(π^2+x^2)}^2  [類 : 不等式への招待 P.39 ]

(5) 0<x<1 に対して、(1-x^2){1+(x-x^2)^3}/(1+x^2) < (sin πx)/(πx)

(6) [1994 Poland] △ABCに対して、1/a + 1/b + 1/c ≦ 1/(a+b-c) + 1/(b+c-a) + 1/(c+a-b)

(7) △ABCに対して、{(b+c)cosA}/a + {(c+a)cosB}/b + {(a+b)cosC}/C ≧ 9(a+b+c)

(8) 鋭角三角形ABCに対して、内接円の半径を r とするとき、
a^2(cos A/2)/\sqrt(b^2+c^2) + b^2(cos B/2)/\sqrt(c^2+a^2) + c^2(cos C/2)/\sqrt(a^2+b^2) ≧ (9r√2)/2
566132人目の素数さん:04/11/01 15:27:06
>557
A+B+C=π、 0 < A, B, C < π/2 において、cosAcosBcosC ≦ 1/8 の別証明。

cosA、cosB、cosC > 0 だから、相加平均・相乗平均の関係を用いた後、
y=cos x は 0 < A, B, C < π/2 において上に凸だから、Jensenの不等式を用いる。

cosA cosB cosC ≦ [(cosA+cosB+cosC)/3]^3 ≦[cos{(A+B+C)/3}]^3 = (cos π/3)^3 = 1/8
567132人目の素数さん:04/11/01 16:22:42
くだらねゑ問題が混ざってゐた。スマソ。

>>562(3)
(解1) 左辺の2つの括弧にそれぞれ相加相乗不等式で瞬殺。等号成立条件は a=b=c=1。
(解2) Schwarzの不等式を使ってから、相加相乗不等式で瞬殺。
568もう一問だけ…:04/11/01 18:01:35
【問題】 正の数 a_1, …, a_n の総和を s とするとき、 Σ[k=1 to n]\sqrt{(s-a_k)/a_k} ≧ n\sqrt(n-1)

コーシー・シュワルツの不等式から、
 Σ[k=1 to n]\sqrt(a_k) ≦ \sqrt(ns)
 Σ[k=1 to n]\sqrt(s-a_k) ≦ \sqrt{n(n-1)s}
相加平均・調和平均の関係から、
 Σ[k=1 to n]1/\sqrt(a_k) ≧ (n^2)/Σ[k=1 to n]\sqrt(a_k)

ここで行き詰まってます。たのも〜(AA略)

ついでに、分子と分母を逆にした式は、チェビシェフの不等式を使って、
 Σ[k=1 to n]\sqrt{a_k/(s-a_k)} ≧ (Σ[k=1 to n]\sqrt(a_k))/\sqrt{n^3(n-1)s}
で合ってますか?

発掘元:S73
http://www.math.nwu.edu/~mlerma/problem_solving/problems/math_hor-10-3-feb03.pdf
569568の修正:04/11/01 18:48:34
>ついでに、分子と分母を逆にした式は、チェビシェフの不等式を使って、
> Σ[k=1 to n]\sqrt{a_k/(s-a_k)} ≧ (Σ[k=1 to n]\sqrt(a_k))/\sqrt{n^3(n-1)s}
>で合ってますか?

チェビシェフの不等式のあと、相加平均・調和平均の関係を使って
 Σ[k=1 to n]\sqrt{a_k/(s-a_k)} ≧ \sqr[n/{s(n-1)}]*Σ[k=1 to n]\sqrt(a_k)
これ以上は綺麗にならないでしょうか?
570132人目の素数さん:04/11/01 20:24:37
>559
 結局、 ab+bc+ca=1 をみたす正の数 a,b,c に対して、次を示せばよい。
   1/(1+a^2) + 1/(1+b^2) + 1/(1+c^2) ≦ 9/4.

> f(x) = 1/(1+x) は x>0 において、下に凸な減少関数であることと、・・・・・から、
> f(a^2)+f(b^2)+f(c^2) ≦ 3f((a^2+b^2+c^2)/3) ≦ ・・・・

ちょっと変な希ガス。そこで迂回路↓

 基本対称式を a+b+c=s, ab+bc+ca=t, abc=u とおく。
 a^2 +b^2 +c^2 = s^2 -2t, (ab)^2 +(bc)^2 +(ca)^2 = t^2 -2su.
 ∴ 9/4 -1/(1+a^2) -1/(1+b^2) -1/(1+c^2)
 = {(9/4)[1+(s^2 -2t)+(t^2 -2su)+u^2]-[3+2(s^2 -2t)+(t^2 -2su)]}/{(1+a^2)(1+b^2)(1+c^2)}
 = {-3+(s^2-2t)+5(t^2 -2su)+9u^2}/{4(1+a^2)(1+b^2)(1+c^2)}
 = {(-3-2t+5t^2)+(s-u)(s-9u)}/{4(1+a^2)(1+b^2)(1+c^2)}.
 ここで t=1, st-9u≧0 だから 左辺≧0.
ぬるぽ
571570:04/11/01 21:00:41
>545,559
 蛇足だけれど、
 チェビシェフの不等式を使うところ:
 (x^2)/(1+x^2) の大小と x/(1+x^2) の大小が一致することを示してほすぃ。
 (前者は単調増加、後者はx=1で極大だが...)
ぬるぽ
572132人目の素数さん:04/11/01 21:07:40
>570 グッジョブ!
気づきませんでした。Jensenの不等式を使ったところが逆でした。
 f(a^2)+f(b^2)+f(c^2) ≧ 3f((a^2+b^2+c^2)/3)
ありがとうございます。
573132人目の素数さん:04/11/01 21:17:18
>571 ハッ!
しまった。後者は単調増加じゃないですね。
あちこちダメポ…。
574132人目の素数さん:04/11/01 21:28:02
>545,559 のチェビシェフを使うための大小が一致することの確認。
x≧y>0のとき、
(x^2)/(1+x^2) - (y^2)/(1+y^2) = (x^2-y^2)/{(1+x^2)(1+y^2)} ≧ 0

x/(1+x^2) - y/(1+y^2) = (x-y)(1-xy)/{(1+x^2)(1+y^2)} …[1]

[1] は xy ≦1 のときに 0以上になるが、
a. b. c は正の数であることと、条件式 ab+bc+ca=1 より、
ab も bc も ca も1より小さい正の数であるので、[1]>0であることが分かる。



>571 いつもながら、ありがとうございます。
575132人目の素数さん:04/11/01 22:21:56
>574の最後の行の訂正。
> …[1]≧0であることが分かる。
576132人目の素数さん:04/11/01 23:54:07
>563(1)
a+b+c=s、ab+bc+ca=t、abc=u とおくと、示すべき不等式は (s^2-2t)+2√(3u) ≦ 1
s=1 を代入して整理すると、√(3u) ≦ t
両辺ともに正だから、2乗の差を比較して
 t^2-3u = t^2-3su = (ab-bc)^2+(bc-ca)^2+(ca-ab)^2 ≧ 0
等号成立条件は、a=b=c=1/3。

左辺しか与えられてなかったら、どうやって最大が1を示すのだろう…。
577132人目の素数さん:04/11/03 06:02:59
>563(3) が解けそうで解けない。
いろんな単語でWeb上を探したら、時間が掛かったけど見つけた。
http://www.google.com/search?hl=en&lr=&c2coff=1&q=60abc+1998

このサイト内で略解を見つけたけど、ハンガリー語(?)で解読不能。
http://matek.fazekas.hu/portal/feladatbank/gyujtemenyek/Nem/AM17.htm

数式だけ追ってみると、次のようになっていた。

【問題】 0≦a≦b≦c に対し、(a+3b)(b+4c)(c+2a) ≧60abc

(a+3b)(b+4c)(c+2a)
≧ 2(a+b)*(5/2)(b+c)*(3/2)(c+a)
= (15/2)(a+b)(b+c)(c+a)
≧ (15/2)*2√(ab)*2√(bc)*2√(ca)
= 60abc
    ___
  ./  nCr \  最初の不等号の評価が謎。
  |::::  \ ./ |   たのも〜
  |::::: (● (● |
  ヽ::::... .∀....ノ /  チン ☆
 _(  ⊃  ⊃  チン ☆
 |\ ̄ ̄ ̄ ̄旦 ̄\
 | | ̄ ̄ ̄ ̄ ̄ ̄ ̄|
 \|  愛媛みかん |
    ̄ ̄ ̄ ̄ ̄ ̄ ̄
578 ◆BhMath2chk :04/11/03 09:00:00
 (a+3b)(b+4c)(c+2a)
≧(4(ab^3)^(1/4))(5(bc^4)^(1/5))(3(ca^2)^(1/3))
=60(a^55・b^57・c^68)^(1/60)
≧60(a^60・b^60・c^60)^(1/60)
=60abc。
579132人目の素数さん:04/11/03 16:17:11
>578
キタ━(゚∀゚)━!!!
ソレダッ!!
ありがとうございます!
580132人目の素数さん:04/11/03 22:02:49
>>558,566 の別証明。

【補題】A+B+C=π, 0<A,B,C<π のとき、
 cos(A)cos(B)cos(C) ≦ [1-cos(A)][1-cos(B)][1-cos(C)] ≦ 1/8.

(略証) 1-cos(x) = 2{sin(x/2)}^2 に注意.
 (右側): sin(x) は(0,π)で上に凸だから、
  sin(A/2)sin(B/2)sin(C/2) ≦ {(1/3)[sin(A/2)+sin(B/2)+sin(C/2)]}^3 ≦ {sin[(A+B+C)/3]}^3
  = {sin(π/3)}^3 = (1/2)^3 = 1/8.
  これを2乗して8倍する。
 (左側): 左辺 = cos(A)cos(B)cos(C) = -cos(A)cos(B)cos(A+B) = {sin(A)sin(B)-cos(C)}cos(C).
  中辺 - 左辺 = [1-cos(A)][1-cos(B)] - {[1-cos(A)][1-cos(B)]+sin(A)sin(B)}・cos(C) + [cos(C)]^2
  = [1-cos(A)][1-cos(B)] - 2√{[1-cos(A)][1-cos(B)]}・cos[(A-B)/2]・cos(C) + [cos(C)]^2
  = X^2 -2XY・cos(θ) +Y^2 = |X-Y|^2 + 2XY[1-cos(θ)] ≧ 0.
  (鈍角三角形のときは 左辺<0 より自明....)
ぬるぽ
581132人目の素数さん:04/11/03 22:18:01
>>580
その補題を使って>>538(3)が出来るはず。
582132人目の素数さん:04/11/03 22:24:35
>538
 (3) [1992 Poland] 44th, 1st round, (1992.Sept-Dec.), No.9
 右辺の3つの因子のうちの2つの和は ≧0 だから、負の因子はあっても1つだけ。
 (i) 負の因子があるとき、0>右辺 より明らか。
 (ii)3つとも ≧0 のとき。
   文字a,b,cの符号を変えても右辺は変わらない。また左辺は
   (a+b+c)^2, (-a+b+c)^2, (a-b+c)^2, (a+b-c)^2 の中の3つの積になる。
   これが最も小さくなるのは、 最大の因子 (|a|+|b|+|c|)^2 を欠く場合、すなわち同符号の場合。
   ∴ a,b,c >0 場合を考えれば十分。このとき、a,b,cを3辺とする鋭角三角形が存在する。
   (a+b-c)(b+c-a) = b^2 -(c-a)^2 = 2ca[1-cos(B)] などにより、
   左辺 = {8(abc)^2}[1-cos(A)][1-cos(B)][1-cos(C)].
   右辺 = {8(abc)^2}cos(A)cos(B)cos(C).
[580]の補題(左)により、左辺>右辺.
 http://www.mimuw.edu.pl/~chel/Olimp/1etang93.html

[538]の解答のレス番(主なもの)
 (1) 548 (2) 540 (3) 581
ぬるぽ
583580:04/11/04 02:01:32
[580] の途中に写しまちがい
 (右側): ・・・・・・ ≦ {sin[(A+B+C)/6]}^3 = {sin(π/6)}^3 = (1/2)^3 = 1/8.
  死んでお詫びを...(AA省略)
584132人目の素数さん:04/11/04 14:41:43
>580-583 キタキタキタキタ━━━(゚∀゚≡(゚∀゚≡゚∀゚)≡゚∀゚)━━━━!!!!!!!!!!
585132人目の素数さん:04/11/04 16:51:13
>>580
中辺 - 左辺
= [1-cos(A)][1-cos(B)] - {[1-cos(A)][1-cos(B)]+sin(A)sin(B)}・cos(C) + [cos(C)]^2
= [1-cos(A)][1-cos(B)] - 2√{[1-cos(A)][1-cos(B)]}・cos[(A-B)/2]・cos(C) + [cos(C)]^2

のところですが、第2項の

{[1-cos(A)][1-cos(B)]+sin(A)sin(B)} = 2√{[1-cos(A)][1-cos(B)]}・cos[(A-B)/2]

は、どうやって変形したのですか?
586580:04/11/04 22:11:27
>585
 1-cos(x) = 2{sin(x/2)}^2 に注意して
 [1-cos(A)][1-cos(B)] = {2sin(A/2)sin(B/2)}^2.
 sin(A)sin(B) = {2sin(A/2)sin(B/2)}・{2cos(A/2)cos(B/2)}.
 辺々加えて、cos()の加法定理を使いまする。
ぬるぽ
587132人目の素数さん:04/11/05 09:46:52
なるほど、分かりました。
sinAsinBsinC ≦ (3√3)/8 に (1-sinA)(1-sinB)(1-sinC) を挟んでみたけど
力不足で証明できませんでした。成り立つのかさえ分かりませんが…。

>581 関係があるとは気づかなかったです。
588132人目の素数さん:04/11/05 21:04:01
>562
 (6) 任意の正の数 a_k, b_k に対し、Π[k=1〜n]a_k + Π[k=1〜n]b_k ≦ Π[k=1〜n]√{(a_k)^2+(b_k)^2}
  を満足するような正整数nをすべてキボンヌ.

  ぢゃない?
 http://www.mimuw.edu.pl/~chel/Olimp/2etang02.html
589132人目の素数さん:04/11/05 23:29:14
>>588Σ('д'*;)!!






|  // /
|// /┃
 / ̄''' ┃  プラーン
 |   (-_-)
 |    U U
 |    UU
 |       (○)
 |       ヽ|〃
 ̄ ̄ ̄ ̄ ̄ ̄ ̄ ̄ ̄ ̄ ̄ ̄
590.:04/11/06 02:53:05
>588
 n=2 はコーシー、n>2は帰納法で。n=1はだめぽ.

>587
 むりぽ.
591132人目の素数さん:04/11/06 04:50:54
実数 x, y に対して、xy/\sqrt{(x^2+y^2)(3x^2+y^2)} の最大値を求めよ。

>562-565 が難しいので、息抜きに簡単なのをやってみたら、できなかった…。
おねがいします。むずぽ。

問題A15.3
http://matek.fazekas.hu/portal/feladatbank/gyujtemenyek/Nem/A15.htm
解答
http://matek.fazekas.hu/portal/feladatbank/gyujtemenyek/Nem/AM15.htm
592132人目の素数さん:04/11/06 14:55:29
>>568
相加・相乗平均だけでよい。 a_1, a_2,・・・・・,a_n の積を u とおくと、
 s - a_k =Σ[i≠k] a_i ≧ (n-1)・(u/a_k)^[1/(n-1)].
 ∴ 左辺 ≧ sqrt(n-1)・Σ[k=1〜n] {(u/a_k)^[1/2(n-1)]}/sqrt(a_k)≧ n・sqrt(n-1).
ぬるぽ
593132人目の素数さん:04/11/06 16:05:26
>591 x^2 =a, y^2 =b とおくと、
 (x^2 +y^2)(3x^2 +y^2) = (a+b)(3a+b) = (a√3 -b)^2 +(4+2√3)ab ≧ [(1+√3)xy]^2.
 ∴ 与式 ≦ 1/(1+√3) = 1/k.

 【問題15.5】 [2002 Irish] Test 1
 0<a,b,c<1 のとき a/(1-a) + b/(1-b) + c/(1-c) ≧ s/(1-s/3) ≧ 3u/(1-u).
 ただし s=a+b+c, u=(abc)^(1/3). 
594132人目の素数さん:04/11/07 06:03:10
>592
ありがとうございます。 最後の部分が分かりません。
 Σ[k=1〜n] {(u/a_k)^[1/2(n-1)]}/sqrt(a_k) ≧ n
書き換えると、Σ[k=1〜n] (u/{(a_k)^n})^[1/2(n-1)] ≧ n ですけど
なんでそうなるのかが分かりません。
595132人目の素数さん:04/11/07 06:13:27
>593
なるほど! そういうことだったのか。
ありがとうございます。


(問題15.5の証明)
下に凸な増加関数 f(x)=1/(1-x) に、Jensen と相加相乗平均を用いる。
等号成立条件は、a=b=c。
 f(a)+f(b)+f(c) ≧ 3f( (a+b+c)/3 ) ≧ 3f( \sqrt[3]{abc} )
596そういえば…:04/11/07 06:54:45
【問題】 正の数 a, b, c が a<b+c をみたすとき、a/(1+a) < b/(1+b) + c/(1+c)

x>0 において f(x)=x/(1+x) は増加関数だから、a<b+c より f(a) < f(b+c)
f(b)+f(c)-f(b+c) = bc(2+b+c)/(1+b)(1+c)(1+b+c) > 0
よって、f(a) < f(b+c) < f(b)+f(c)


第3回シュプリンガー数学コンテスト問題A(a)
http://www.springer-tokyo.co.jp/contest/contest3.pdf
(b)なんてただの飾りです…(以下略)
597592:04/11/07 07:04:46
>594
 相加・相乗平均により
Σ[k=1〜n] (u/{(a_k)^n})^c ≧ n・Π[k=1〜n] [u^(1/n)/a_k])^(nc) = n・{1}^(nc) = n
598132人目の素数さん:04/11/07 21:10:50
なるほど、相加相乗を2回使ったのですか。
599593:04/11/07 21:54:45
>595 (問題15.5)
 なるほど! そういうことだったのか。
 ありがとうございます。

 なお、両端だけなら相加相乗3回で出まつ。 ぬるぽ
600593:04/11/07 22:17:15
[599]の補足 > 593
> なお、相加相乗3回で出まつ。
1/(1-a) +1/(1-b) +1/(1-c) ≧ 3/[(1-a)(1-b)(1-c)]^(1/3) ≧ 3/(1- s/3) ≧ 3/(1-u).
 の各辺から3を引く。 ぬるぽ
601132人目の素数さん:04/11/07 23:40:33
【問題15.1】
 a,b,c≧0 のとき、 7(ab+bc+ac)(a+b+c) ≦ 2(a+b+c)^3 +9abc.
 よろしくおながいします。
602132人目の素数さん:04/11/08 01:52:56
>601
a+b+c=s のとき、a/s, b/s, c/s を改めて a, b, c とおけば、同じ不等式をみたす。
したがって、a+b+c=1としてよく、【問題15.1】 は次の問題に同値になる。

[1999.3 BMO]
a, b, c≧0、a+b+c=1 のとき、ab+bc+ca-9abc/7 ≦ 2/7
http://www.kalva.demon.co.uk/bmo/bmo99.html

最小値も追加した次の不等式を示す。


【問題】 a,b,c≧0, a+b+c=1 のとき、0 ≦ ab+bc+ca-9abc/7 ≦ 2/7

(左側の証明)
対称式なので a≧b≧c≧0 としてよい。
1 = a+b+c ≧ 3c より、1/3 ≧ c ≧ 0 だから
ab+bc+ca-9abc/7 = (a+b)c + ab(1- 9c/7) ≧ 0
等号成立条件は、対称性を取っ払って a, b, c のうちの少なくとも2個が0のとき

(右側の証明)
(i) 9/7 ≦ a ≦ 1 のとき、1 ≦ 9a/7 より、bc ≦ 9abc/7.
また b+c = 1-a ≦ 2/9 より、ab+ca = a(b+c) ≦ 2a/9 <2/7 だから、不等式は成り立つ。

(ii) 0 ≦ a ≦ 9/7 のとき、(b+c)^2-4bc = (b-c)^2 ≧ 0 より、
bc ≦ (1/4)(b+c)^2 =(1/4)(1-a)^2 だから、以下のように示される。等号は a=b=c=1/3.

ab+bc+ca-(2+9abc)/7
= a(b+c)+bc(1- 9a/7)-2/7
≦ a(1-a)+(1/4)(1- 9a/7)(1-a)^2 -2/7
= -(1/28)(a+1)(3a-1)^2
≦ 0
603601の類題:04/11/08 01:54:13
[1984.A1 IMO]
a, b, c≧0、a+b+c=1 のとき、0 ≦ ab+bc+ca-2abc ≦ 7/27
http://www.kalva.demon.co.uk/imo/imo84.html

[1993.26 IMO shortlist]
a, b, c, d≧0、a+b+c+d=1 のとき、abc+bcd+cda+dab ≦ (1+176abcd)/27
http://www.kalva.demon.co.uk/short/sh93.html

[1999 CMO]
a, b, c≧0、a+b+c=1 のとき、a^2b+b^2c+c^2a ≦ 4/27
http://www.kalva.demon.co.uk/canada/can99.html

[2000.33 MOCP]
三角形の3辺 a, b, c が a+b+c=2 のとき、2 > a^2+b^2+c^2+2abc ≧ 52/27
http://www.cms.math.ca/Competitions/MOCP/2000/prob_sep.pdf

[1989.10 ソ連]
三角形の3辺 a, b, c が a+b+c=1 のとき、1/2 > a^2+b^2+c^2+2abc
http://www.kalva.demon.co.uk/soviet/sov89.html

[2003.B1 アイルランド]
三角形の3辺 a, b, c が a+b+c=2 のとき、1 < ab+bc+ca-abc ≧ 28/27
http://www.kalva.demon.co.uk/irish/irish03.html

          ___
    |┃三 ./  ≧ \   >601 呼んだ?
    |┃   |::::  \ ./ | 私のコレクションは
    |┃ ≡|::::: (● (● |   半端じゃありませんよ
____.|ミ\_ヽ::::... .ワ......ノ      ハァハァ…
    |┃=__    \      
    |┃ ≡ )  人 \ ガラッ
604132人目の素数さん:04/11/08 01:59:08
さぁ、遠慮なく (;´д`)ハァハァ してください。
605132人目の素数さん:04/11/08 03:11:44
>600
なるほど。


>>565(6) [1994 Poland] について…
△ABCに対して、1/(a+b-c) + 1/(b+c-a) + 1/(c+a-b) ≧ 1/a + 1/b + 1/c ≧ 9/(a+b+c)

両端だけなら、相加平均・調和平均の関係で出るんだけどなぁ…
606132人目の素数さん:04/11/08 05:36:19
>601
直接に差をとって証明しようとすると、a+b+c=s, ab+bc+ca=t, abc=u として

2(a+b+c)^3 +9abc-7(ab+bc+ac)(a+b+c) = 2s^3-7st+9u = 2s(s^2-3t)-(st-9u)

となって、だめぽ。いい方法ないですか?
607132人目の素数さん:04/11/08 10:54:33
>>605
1/xの凸不等式で右側も左側もすぐに証明できるんじゃないの?
608132人目の素数さん:04/11/08 12:03:51
>605,607
 なるほど! そういうことだったのか。
 ありがとうございます。
 「相加平均・調和平均の関係で」「右側も左側もすぐに証明できるんじゃないの?」
 (左側)  [1/(a+b-c) +1/(b+c-a)]/2 ≧ 1/b を循環的に加える。

>606
 2(a+b+c)^3 +9abc -7(ab+bc+ac)(a+b+c) = (a-b)(a^2 -b^2) +(b-c)(b^2 -c^2) +(c-a)(c^2 -a^2).
ぬるぽ
609608:04/11/08 12:13:27
>606
 s(s^2-3t) -(st-9u) ≧0 も成り立つらしいYo.
610132人目の素数さん:04/11/08 17:07:03
>607-609
キタ━(゚∀゚)━!!!
なるほど! ベリィdクスです。
611132人目の素数さん:04/11/08 20:31:05
>609
b が a, c の中間にあるとすると、(a-b)(b-c)≧0, a-b+c≧0 ゆえ,
 s^3 -4st +9u = a(a-b)^2 +c(b-c)^2 +(a-b)(b-c)(a-b+c) ≧0 ・・・・・(1)
これと
s^2 -3t = (1/2)[  (a-b)^2 +  (b-c)^2 +  (c-a)^2] ≧0 ・・・・・(2)
ts - 9u =   [ c(a-b)^2 + a(b-c)^2 + b(c-a)^2] ≧0 ・・・・・(3)
t^2 -3su =(1/2)[c^2(a-b)^2 +a^2(b-c)^2 +b^2(c-a)^2] ≧0 ・・・・・(4)
 を使えば かなりできそうだが....(△を除き)
ぬるぽ
612132人目の素数さん:04/11/09 03:36:09
>>564(3)(5) ができそうで出来ません。 たのも〜

それと、下の問題も小一時間考えても分からないので教えて下さい。
たのも〜

【問題】 実数 a, b, c が abc=1 をみたすとき、
a^3+b^3+c^3+(ab)^3+(bc)^3+(ca)^3 ≧ 2(a^2b+b^2c+c^2a)
http://www.journals.cms.math.ca/CRUX/synopses/2004/n5/PDF/v30n5syn.pdf
613132人目の素数さん:04/11/09 12:07:48
>>564 (3)
a,b,cの符号によらないから、a,b,c ≧0 の場合を考える。 与式を X+Y+Z とおくと、
 最大値: X+Y+Z ≦ √{3(X^2 +Y^2 +Z^2)} = √{3(p^2+q^2+r^2)(a^2+b^2+c^2)}
     = √{3(p^2+q^2+r^2)}.
 最小値: X≧(pa+qb+rc)/(√3), Y≧(pb+qc+ra)/(√3), Z≧(pc+qa+rb)/(√3).
   辺々たすと X+Y+Z ≧ (p+q+r)(a+b+c)/(√3) ≧ (p+q+r)/(√3).

>612
 相加・相乗平均により、(2a^3 +c^3)/3 + (ab)^2/c ≧ (a^2)[c +(b^2)/c] ≧ 2(a^2)b.
 循環的に加える。 ぬるぽ
614132人目の素数さん:04/11/09 16:08:16
>>613
負の時もそれでいいの?
615132人目の素数さん:04/11/09 16:12:55
>613
キタ━(゚∀゚)━!!!
なるほどッ!

>614
考察する式には、a^2, b^2, c^2 だけで a, b, c は入っていないから
最大値最小値を考えるには、a, b, c ≧ 0 の場合を考えただけで十分でしょ。
616132人目の素数さん:04/11/09 16:20:16
>>615
>>612の問題の方のことだけど
617132人目の素数さん:04/11/09 19:39:43
>>534 (5)-(i)
 f(x)=log(1+x^2) は 0<x<1 で下に凸だから
 与式 = exp{f(a)+f(b)+f(c)} ≧ exp{3f([a+b+c]/3)} = {1 +[(a+b+c)/3]^2}^3 = (10/9)^3.
 
>612, 616
任意の実数に対しては成り立たない希ガス...(たとえば c=-1 のとき 左辺=-2, 右辺=-2b^2)
 おそらく右辺の chirality が原因... ぬるぽ
618132人目の素数さん:04/11/10 03:45:45
>>563(6) 0 < a, b, c < 1/2 に対して、(1/a -1)(1/b -1)(1/c -1) ≧ {3/(a+b+c) -1}^3

f(x) = 1/x -1 は、0 < x < 1/2 において下に凸な減少関数で f(x) > 1。
したがって、0 < a, b, c <1 において f(a), f(b), f(c) > 1。
問題に手を加えて、次式を証明したい。

 [f(a)+f(b)+f(c)]/3 ≧ \sqrt[3][f(a)f(b)f(c)] ≧ f((a+b+c)/3)

左辺 ≧ 中辺 … 相加相乗平均より成立
左辺 ≧ 右辺 … Jensenの不等式より成立

中辺 ≧ 右辺 … これが原題ですが、うまい方法ないでしょうか?

    ___
  ./  ≧ \  ついでに調和平均が、この不等式の
  |::::  \ ./ |    どこに入るか分かれば教えて下さい。
  |::::: (● (● |
  ヽ::::... .∀....ノ /  チン ☆    たのも〜
 _(  ⊃  ⊃  チン ☆        たのも〜
 |\ ̄ ̄ ̄ ̄旦 ̄\
 | | ̄ ̄ ̄ ̄ ̄ ̄ ̄|
 \|  愛媛みかん |
    ̄ ̄ ̄ ̄ ̄ ̄ ̄
619618:04/11/10 04:38:04
>>564(1)
a^3+b^3+c^3 = (a+b+c)^3-3(a+b)(b+c)(c+a) より、
 (与式) = [(a+b+c)^3]/[(a+b)(b+c)(c+a)] -3

相加相乗平均より、2(a+b+c) = (a+b)+(b+c)+(c+a) ≧ 3\sqrt[3]{(a+b)(b+c)(c+a)}
∴ (与式) ≧ (3/2)^3-3 = 3/8

等号成立条件は a=b=c
620132人目の素数さん:04/11/10 09:44:44
>618
 >>563(6) f(x) = 1/x -1 とおくと {log|f(x)|} '' = {log(1-x) -log(x)} '' = 1/(x^2) -1/(1-x)^2.
  log|f(x)| は 0<x<1/2 で下に凸だから、 f(a)f(b)f(c) ≧ {f([a+b+c]/3)}^3.
ぬるぽ
621132人目の素数さん:04/11/10 09:59:18
>620
(*゚∀゚)=3 ありがとうございます!
622132人目の素数さん:04/11/11 01:04:07
age
623132人目の素数さん:04/11/11 02:27:09
>622
屑はageか3桁の数字を書き込むことしか出来ない
624132人目の素数さん:04/11/11 12:39:49
>>563(5) [1993 Itary] 0≦a, b, c≦1 に対して、a^2+b^2+c^2 ≦ a^2b+b^2c+c^2a+1

b, c を固定して f(a) = (a^2b+b^2c+c^2a+1)-(a^2+b^2+c^2) を考える。
b=1のとき直線、0≦b<1のとき上に凸な放物線だから、区間の端点で最小値をとる。
 f(0) = (1-c){c+(1-b^2)} ≧ 0
 f(1) = b(1-b)+c^2 ≧ 0

類題も同様にすればいい。
625132人目の素数さん:04/11/11 12:44:09
>>563(2) 綺麗な形をしているけど分からんちん。
>>563(4) できそうで できない。
>>564(4) 条件式が汚いからサッパリ。
>>564(5) できるのか、これ?
626132人目の素数さん:04/11/11 12:44:46
というわけで、せんせー方、たのも〜 (AA略)
627132人目の素数さん:04/11/11 18:53:42
>>564(5) [>>617]
 (i) f(x)=log(1+x^2) は 0<x<1 で下に凸だから
 与式 = exp{f(a)+f(b)+f(c)} ≧ exp{3f([a+b+c]/3)} = {1 +[(a+b+c)/3]^2}^3 = (10/9)^3.

(ii) g(x)=1+√x は上に凸なので、log|g(x)| も上に凸
 与式 = g(a)g(b)g(c) ≧ g(0)g(0)g(1) = 1+√2.
628627:04/11/11 19:19:45
訂正、  与式 = g(a)g(b)g(c) ≧ g(0)g(0)g(1) = 2.
629132人目の素数さん:04/11/12 06:13:10
>627-628
ありがdございます。
なるほど、凸を使うのか。 (*゚∀゚)=3
630132人目の素数さん:04/11/12 07:40:08
>627-628
(5)(ii) ですが、やっぱり分からんです。
g(x) = 1+√x は、x≧0 において上に凸で、g(x)>1 なので、
G(x) = log g(x) も、x≧0 において上に凸。 Jensenの不等式より
 G(a)+G(b)+G(c) ≦ 3G((a+b+c)/3) = 3G(1/3)
 ∴ g(a)g(b)g(c) ≦ {g(1/3)}^3 = 2+(5/9)√3

と最大値はでますが、最小値は何故 g(a)g(b)g(c) ≧ g(0)g(0)g(1) なのですか?
631627-628:04/11/12 08:20:43
>629-630   564(5)-(ii)の補足
 G(x)=log|g(x)| は上に凸:  G(x) ≧ G(0)・(1-x) + G(1)・x
 ∴ G(a)+G(b)+G(c) ≧ G(0)・(3-a-b-c) + G(1)・(a+b+c) = G(0)+G(0)+G(1).
 ∴ g(a)g(b)g(c) ≧ g(0)g(0)g(1).
ぬるぽ
632132人目の素数さん:04/11/12 09:03:07
>631
>G(x)=log|g(x)| は上に凸:  G(x) ≧ G(0)・(1-x) + G(1)・x

凸関数をそう使うのか (;´Д`) ハァハァ /lァ/lァ /lア/lア!!
ありがとうございます。少し賢くなったような気がしまする。
凸関数があったら Jensen ばかり使っていた自分は…
633132人目の素数さん:04/11/12 09:18:56
同様にすると、(5)(i) の最大値は、
f(x)=log(1+x^2) は 0≦x≦1 で下に凸だから、f(x) ≦ f(0)(1-x)+f(1)x
 f(a)+f(b)+f(c) ≦ f(0)(3-a-b-c)+f(1)(a+b+c) = f(1)
 ∴ (1+a^2)(1+b^2)(1+c^2) ≦ 2
634132人目の素数さん:04/11/12 09:22:56
等号が成り立つのは、x=0 または 1 のときだから、
 (a, b, c) = (1, 0, 0), (0, 1, 0), (0, 0, 1)
635132人目の素数さん:04/11/12 10:17:23
>>538(2) を 条件を a, b, c > 0、a+b+c=1 に変えると、
 1/2 ≦ a/(1+a^2) + b/(1+b^2) + c/(1+c^2) ≦ 9/10

f(x) = x/(1+x^2) は 0≦x≦1 で上に凸だから、
左側は、f(x) ≦ f(0)(1-x)+f(1)x = x/2 より、右側はJensenの不等式。

うひょ〜 (*゚∀゚)=3
636132人目の素数さん:04/11/12 16:51:35
a, b, c≧0 が a+b+c=1 をみたすとき、a^2(b+c)+b^2(c+a)+c^2(a+b) のとりうる値の範囲について、
最大値について、Jensenの不等式を用いたら出ないのですが、どこがおかしいか教えて下さい。

a^2(b+c)=a^2(1-a) より、f(x) = x^2(1-x) を考える。
0≦x≦1において f(x) は上に凸だから、Jensenの不等式より
 a^2(b+c)+b^2(c+a)+c^2(a+b) = f(a)+f(b)+f(c) ≦ 3f((a+b+c)/3) = 3f(1/3) = 2/9
等号成立条件は a=b=c=1/3。

ところが、例えば (a, b, c) = (1/2, 1/2, 0) のとき、1/4 の値をとり、これは 1/4 > 2/9 なんです。
Jensenの不等式の使い方間違ってますか?

          r〜〜〜〜〜
   __    _ノ うっうっうっ・・・
  /__  `ヽ_ ⌒ヽ〜〜〜〜〜
  |〈___ノf レ1(
 ,L| しL.し'゙"
 "`  "′
637636:04/11/12 16:55:37
ゴメン。書いてて気がついた。はずかし…
上に凸じゃないや、ダメダメだね俺。吊ってくるわ (AA略)
638132人目の素数さん:04/11/12 17:07:49
スレ汚しの罰に、問題UP

0≦a, b, c≦1のとき、S=\sqrt{a(1-b)(1-c)}+\sqrt{b(1-c)(1-a)}+\sqrt{c(1-a)(1-b)} について

(1) S ≦ 1+\sqrt{abc} を示せ。等号成立条件も。
(2) S の最大値を求めよ。

Problem 175 (これって、何かの雑誌?)
http://www.math.nwu.edu/~mlerma/problem_solving/problems/math_hor-11-1-sep03.pdf
639132人目の素数さん:04/11/12 17:51:58
>>563(4)  できそうで...
 左辺−右辺 = (a^2)(b-c)/c - (b^2)(a-c)/a + (c^2)(a-b)/b
 = a(a-c)(b-c)/c + b(a-b)(a-c)/a -c(b-c)(a-b)/b
 = (a/c)(b-c)^2 + (b/a)(a-b)^2 + {a/c +b/a -c/b}(b-c)(a-b)
 = (a/c)(b-c)^2 + (b/a)(a-b)^2 + {(a/c-1)+(b/a)+(1-c/b)}(b-c)(a-b)≧0.
ぬるぽ
640三角函数ヲタ:04/11/12 22:18:24
>638 √a =sin(A/2), √b =sin(B/2), √c= sin(C/2) (0≦A,B,C≦π)とおく。

(1) S(A,B,C) = sin([A+B+C]/2) + sin(A/2)・sin(B/2)・sin(C/2) ≦ 1+√(abc), 等号成立はA+B+C=π.

(2) 2∂S/∂A = cos([A+B+C]/2) + cos(A/2)・sin(B/2)・sin(C/2) = 0,
  2∂S/∂B = cos([A+B+C]/2) + sin(A/2)・cos(B/2)・sin(C/2) = 0,
  2∂S/∂C = cos([A+B+C]/2) + sin(A/2)・sin(B/2)・cos(C/2) = 0.
 ∴A/2=B/2=C/2=θ, これを上式に入れて, cos(3θ) + cosθ・(sinθ)^2 =(3x^2 -2)x=0,
  x=cosθ=√(2/3), θ=90゚-54゚44', a=b=c=1/3, ∴ S≦2/(√3).
ぬるぽ
641132人目の素数さん:04/11/13 05:30:27
>639-640
キタ━(゚∀゚)━ ! ! !
さらりと解いてしまう、そこに しびれる あこがれる〜!
642132人目の素数さん:04/11/13 08:18:53
>640
その置き換えに痺れる憧れるぅ〜。 (;´Д`) ハァハァ しました。
しかし、(2)の最大値は 9/8 ではないでしょうか?

(1)より得られた上限値 1+sin(A/2)・sin(B/2)・sin(C/2) に、Jensenの不等式を用いる。
[>>557補題] より、0≦x≦π において f(x) = log sin(x/2) は上に凸だから、
 f(A)+f(B)+f(C) ≦ 3f((A+B+C)/3) = 3f(π/3)
 ∴ 1+sin(A/2)・sin(B/2)・sin(C/2) ≦ 1+{sin(π/6)}^3 = 9/8
等号成立条件は、A=B=C=π/3 すなわち a=b=c=1/4
    ___
  ./  ≧ \  
  |::::  \ ./ |    これでいいでしょうか?
  |::::: (● (● |
  ヽ::::... . ワ ....ノ /  チン ☆    たのも〜
 _(  ⊃  ⊃  チン ☆        たのも〜
 |\ ̄ ̄ ̄ ̄旦 ̄\
 | | ̄ ̄ ̄ ̄ ̄ ̄ ̄|
 \|  愛媛みかん |
    ̄ ̄ ̄ ̄ ̄ ̄ ̄
643642:04/11/13 08:40:50
あぁそうか、勘違いしてました。
(2)はSの最大値で、1-√(abc) の最大値じゃないんですね。

      ∧_∧
     ( ´Д` )    まことに
     /     ヽ
     し、__X__,ノJ

      /´⌒⌒ヽ
    l⌒    ⌒l   すいませんでした
   ⊂ (   ) ⊃
      V ̄V
644132人目の素数さん:04/11/13 08:55:02
あーでも変だなぁ…
どこがおかしいんだろ?
645642:04/11/13 10:47:27
自己解決。
A+B+C=π という束縛の下で解いたから最大値が小さくなったんですね。
sin((A+B+C)/2) が1にならないときに最大となる可能性があるから…。
普通に解いてみました。

>638の最大値を求める。 >640の置き換えより、
 (与式) = sin((A+B+C)/2) + sin(A/2)・sin(B/2)・sin(C/2)

0≦x≦π において g(x) = sin(x/2) は、g(x)≧0 かつ上に凸だから、
相加相乗平均とJensenの不等式より
 g(A)・g(B)・g(C) ≦ [g(A)+g(B)+g(C)/3]^3 ≦ [g((A+B+C)/3)]^3
等号成立条件は A=B=C

これと3倍角の公式から、(与式) ≦ [g(3A)+g(A)]^3 = 3g(A)-3[g(A)]^3

0≦g(A)≦1 における最大値を考えて、g(A)=1/√3 で極大かつ最大値 2/√3 をとるぽよ。
646645の訂正:04/11/13 10:49:10
下から2行目の書き間違い

(×) これと3倍角の公式から、(与式) ≦ [g(3A)+g(A)]^3 = 3g(A)-3[g(A)]^3
(○) これと3倍角の公式から、(与式) ≦ g(3A)+[g(A)]^3 = 3g(A)-3[g(A)]^3
647問題:04/11/13 12:52:57
ネタが尽きかけたので、補充しまつ。

【問題】 文字は全て正の数とする。

(1) 1/[1/(1+a) + 1/(1+b) + 1/(1+c)] - 1/(1/a + 1/b + 1/c) ≧ 1/3

(2) [1997 Belarus] a/b + b/c + c/a ≧ (b+c)/(a+b) + (c+a)/(b+c) + (a+b)/(c+a)

(3) [1995 Russia] 1/(ab) ≧ a/(a^4+b^2) + b/(a^2+b^4)

(4) [1997 Belarus] (a+y)x/(a+x) + (a+z)y/(a+x) + (a+x)z/(a+y) ≧ x+y+z ≧ (a+z)x/(a+z) + (a+x)y/(a+y) + (a+y)z/(a+z)

(5) [1997 Romania] a^2/(a^2+2bc) + b^2/(b^2+2ca) + c^2/(c^2+2ab) ≧ 1 ≧ bc/(a^2+2bc) + ca/(b^2+2ca) + ab/(c^2+2ab)

(6) [1993 Poland] (a+c)(b+d)/(a+b+c+d) ≧ ab/(a+b) + cd/(c+d)

(7) [1999 Moldova] ab/[c(c+a)] + bc/[a(a+b)] + ca/[b(b+c)] ≧ a/(c+a) + b/(b+a) + c/(c+b)


発掘先 (;´Д`) ハァハァ /lァ/lァ /lア/lア!!
http://myhome.personaldb.net/ideahitme/psine1.pdf
http://myhome.personaldb.net/ideahitme/psine2.pdf
648132人目の素数さん:04/11/13 13:17:37
下らん不等式並べるなよ馬鹿
649132人目の素数さん:04/11/13 19:15:06
>>564(6)
> 実数 a,b,c≧0 に対して、常に次式が成り立つような定数 k の最小値。
> k(a^3 +b^3 +c^3) +(9-3k)abc ≧ (a+b+c)(a^2 +b^2 +c^2)

 a+b+c=s, ab+bc+ca=t, abc=u とおく。
 k(s^3 -3st +3u) +(9-3k)u ≧ s(s^2-2t).
 (k-1)s(s^2 -3t) -(st-9u) ≧ 0.

 k=2 のときは、↓により成立。
  s^3 -4st +9u = a(a-b)^2 +c(b-c)^2 +(a-b)(b-c)(a-b+c) ≧0 ・・・・・(1) [>>611]
 k>2 のときは、これと (k-2)(a^3 +b^3 +c^3 -3abc)≧0 とを加えて成立。
k<2 のときは、a=b, c=0 の場合、左辺=2(k-2)a^3 < 0 で不成立。

----------------------------------------------------------------------

一方、ts -9u = [ c(a-b)^2 +a(b-c)^2 +b(c-a)^2] ≧0 ・・・・・(3) [>>611]

>>601-602 [1999.3 BMO] 左側:(3)から, 右側:[>>608] または {(1)×2+(3)}/7
>>603(1) [1984.A1 IMO] 左側:(3)から, 右側:{(1)×7+(3)}/27
ぬるぽ
650132人目の素数さん:04/11/14 04:38:37
お〜なるほどッ!
651132人目の素数さん:04/11/16 20:19:06
>>562
(1) |a|,|b|≧|c| とすると, |a^2 +b^2 +c^2| ≦ |a|^2 +|b|^2 +|c|^2 ≦ |a|^2 +|b|^2 +2|ab| = (|a|+|b|)^2.

(2) [1999 Poland] 50th, 1st round (1998 Sep.-Dec.), No.2
  a+b=s とおくと、右辺−左辺 = 3(s^2 +c^2 +d^2) -(s+c+d)^2 = (s-c)^2 +(c-d)^2 +(d-s)^2 ≧0.

(3) 相加・相乗不等式で瞬殺。[>>567]

(4) 相加・相乗平均により (10ab^4+11bc^4+7cd^4+23da^4)/51 ≧ a(abcd). これを循環的に加える。

(5) [2003 Poland] 54th, 1st round (2002 Sep.-Dec.) No.12
  4文字の基本対称式を a+b+c+d=S, ab+bc+ca+ad+bd+cd=T, abc+bcd+cda+dab=U, abcd=V とおく。
  A = a^3+b^3+c^3+d^3 = S^3 -3ST +3U, B=U.
  右辺−左辺−9U = 4A +15U - S^3 = 3(S^3 -4ST +9U).

【補題】S^3 -4ST +9U ≧0 は4文字の場合も成り立つ。
 (略証) S^3 -4ST +8U = (a+b-c-d)(a-b+c-d)(a-b-c+d) ・・・・・・ (*)
  対称式だから、a≧b≧c≧d>0 としてよい。 a-b=f, b-c=g, c-d=h とおくと、U≧abc.
  S^3 -4ST +9U ≧ (f+2g+h)(f+h)(f-h) + abc = (f+2g+h)(f^2-h^2) + (f+g+h+d)(g+h+d)(h+d)
  ≧ (f+2g+h)(f^2 -h^2) + (f+g+h)(g+h)h = f^2(f+2g+h) +gh(f+g) ≧ 0. (終)
  なお、3文字のときは [>>611]

※ S^2 -(8/3)T = (1/3)Σ[i<j] (a_i-a_j)^2, 
  ST-6U = (1/2)Σ[i<j] (S-a_i-a_j)(a_i-a_j)^2,
  S^3 -4ST +8U = S{S^2 -(8/3)T} -(4/3)(ST-6U) = (1/3)Σ[i<j] (2a_i+2a_j-S)(a_i-a_j)^2 より.
652132人目の素数さん:04/11/16 20:22:17
>>562
(6) [2002 Poland] 53rd, 2nd round, 2nd day (2002.2.23) No.6
  n=2 のときはコーシーの不等式、n>2 のときは帰納法で。 n=1は不成立。[>590]
 
(7) a_1 = b_2 ≠ a_2 = b_1 の場合を考えると・・・

 [562]の解答のレス番(主なもの)  
  (1) [651] (2) [651] (3) [567] (4)[651] (5) [651] (6) [590] (7) ?
ぬるぽ
653132人目の素数さん:04/11/17 01:13:22
>651-652
神キタ━(゚∀゚)━ !!!

651(4)、うますぎる!とても作り出せない…。
abcd=1 としてゴチャゴチャやってましたができませんでした。
_| ̄|○
654132人目の素数さん:04/11/17 01:16:05
連立方程式をたてて解けばいいんだよ・・・
655132人目の素数さん:04/11/17 01:18:27
実数 x に対して、|sin x + cos x + tan x + cot x + sec x + cosec x| ≧ 2√2 - 1

[AMM_Oct._2004 P.682] より発掘。解答は P.685

(;´Д`) ハァハァ /lァ/lァ /lア/lア!!
656132人目の素数さん:04/11/17 04:07:03
s=sinx,c=cosx,s^2+c^2=1
|s+1/s+c+1/c+s/c+c/s|≧2√2-1
657132人目の素数さん:04/11/17 04:29:10
>656
ドラクエでもやれば?
( ゚∀゚) アハハ八八ノヽノヽノヽノ \ / \/ \
658132人目の素数さん:04/11/17 13:04:33
>655
(i) 0<x<π/2 のとき 各項>0 ゆえ
 与式 = {sin(x) +1/sin(x)} + {cos(x) +1/cos(x)} + {tan(x)+cot(x)} > 2+2+2 = 6.
(ii) π/2 < x < 2π のとき
 z=cot(x/2) とおくと, 1-z>0.
 sin(x) + cos(x)= 1 -2(1-z)/(1+z^2),
 tan(x) + 1/cos(x) = [sin(x)+1]/cos(x) = -(1+z^2)/(1-z) -z,
 cot(x) + 1/sin(x) = [cos(x)+1]/sin(x) = z.
 ∴ 与式 = 1 -2(1-z)/(1+z^2) -(1+z^2)/(1-z) ≦ 1 -2√2 (←相加・相乗平均).
 等号成立は、z =−(1/2)√2 ± √(√2 -1/2) のとき.
659658:04/11/17 20:57:29
>658(ii) (補足)
[sin(x) +1]/cos(x) + [cos(x)+1]/sin(x) = [1+sin(x)+cos(x)]/[sin(x)cos(x)]
= -2/[1-sin(x)-cos(x)].
660659:04/11/18 08:57:13
(続き)
 1-sin(x)-cos(x) =y とおくと 与式 = 1-y-2/y. ∴ |与式−1| =|y +2/y|≧ 2√2.
661660:04/11/18 18:08:39
(続き) 以上により↓が示された。
【命題】 F= sin(x) + cos(x) + tan(x) + cot(x) + sec(x) + cosec(x) とおくとき
 F≧ 3√2 +2 または F≦-(2√2 -1).
ぬるぽ
662132人目の素数さん:04/11/18 21:23:57
>>563
(1) a+b+c=s、ab+bc+ca=t、abc=u とおき、次の同次形を示す。 s^2 -2t +2√(3su) ≦ s^2. [>>576]

(2) 綺麗な形をしているけど分からんちん。
  同次形は (a+b+c)^2・(pa+qb+rc) -8abc(p+q+r) ≧0.
  左辺−右辺 = {(a+b+c)^2 -8bc}pa + {・・・}qb + {・・・}rc としても {・・・}≧0 とは言えない...
  題意により (p+q+r)/2 ≧ p,q,r ≧0 だから、 p,q,rは三角不等式を満たす。そこで
  左辺−右辺 = f(a,b;c)(p+q-r) + f(b,c;a)(-p+q+r) + f(c,a;b)(p-q+r).
  f(a,b;c)≡{(a+b)+c}^2・(a+b)/2 -8abc≧ {4(a+b)c}・(a+b)/2 -8abc≧ 0.

(3) 相加・相乗平均でハァハァと [>>578]

(4) できそうで できない。 a,b≧cの使い方。[>>639]

(5) 上に凸 [>624]
  類題は  (ab+bc+ca+1) - (a+b+c) = (1-a)(1-b)(1-c) + abc ≧0.

(6) f(x) = 1/x -1 とおくと {log|f(x)|} '' = {log(1-x) -log(x)} '' = 1/(x^2) -1/(1-x)^2.
  log|f(x)| は 0<x<1/2 で下に凸 [>>620]

(7) 0≦y≦1, n≧1 のとき 1-y^n≧1-y を使う。

[563]の解答のレス番号(主なもの)
(1) [576] (2) これ (3) [578] (4) [639] (5) [624] (6) [620] (7) これ
ぬるぽ
663132人目の素数さん:04/11/19 09:44:50
>>662 ぬるぽ神 キタ─wwヘ√レvv〜(゚∀゚)─wwヘ√レvv〜─ !!!
ありがとうございます。

(2) ウホッ! どうやって同次系の不等式を思いつくのですか? コツがあるのでしょうか?
a+b+c も p+q+r もどちらも1なので、組合せは何通りも考えられますよね。
その後の進め方もまた (;´ρ`) ハァハァ /lァ/lァ /lア/lア!!

(5)類題、こんなにアッサリ片付くとは…。文字を固定してゴリゴリ証明してました。

(7) うう、分かりません。 「0≦y≦1, n≧1 のとき 1-y^n≧1-y を使う」 と、
 左辺第1項 ≧ (1-x)^m
 左辺第2項 ≧ x^n
となるから、左辺 ≧ (1-x)^m - x^n となって、その後どうするのでしょうか?

>654 なるほど、連立方程式の立て方が分かりました。 (  ̄ー ̄) ニヤソ
664132人目の素数さん:04/11/19 12:11:56
>>564
 基本対称式を a+b+c=s, ab+bc+ca=t, abc=u とおく。

(1) 相加・相乗でハァハァと [>>619]
  a^3 +b^3 +c^3 = s^3 -3(a+b)(b+c)(c+a)
  s^3 = {(3/2)[(a+b)+(b+c)+(c+a)]/3}^3 ≧ (3/2)^3・(a+b)(b+c)(c+a).
  ∴ 与式 = (s^3)/[(a+b)(b+c)(c+a)} -3 ≧ (3/2)^3-3 = 3/8.

(2) 分母 = (2a^2+b^2)(b^2+2c^2) = t^2 +(b^2 -ac)^2 +a^2(b-c)^2 +c^2(a-b)^2 ≧ t^2.
  ∴ 与式 = us/(t^2) ≦ 1/3.

(3) 最大値:√{3(p^2+q^2+r^2)}, 最小値:(p+q+r)/(√3). [>>613]

(4) 条件式が汚いからサッパリ。
 b-c=x, c-a=y とおくと a-b=(-x-y), 条件式は xy≧1.
 F(x,y) = {(x+y)^(2n+1)-x^(2n+1)-y^(2n+1)}/{xy(-x-y)}
 = {(x+y)^(2n) - x^(2n) +x^(2n-1)・y-・・・・+x・y^(2n-1)-y^(2n)}/xy
 = Σ[k=0,2n-2] {C[2n,k+1]+(-1)^k} x^k・y^(2n-2-k)
 = Σ[k=0,2n-2] {C[2n,k+1]+(-1)^k} {x^k・y^(2n-2-k) + x^(2n-2-k)・y^k}/2
 ≧ |xy|^(n-1)・Σ[k=0,2n-2] {C[2n,k+1]-(-1)^k}  (←相加・相乗平均)
 = |xy|^(n-1)・F(1,1) ≧ F(1,1) = 2^(2n) -1.
665132人目の素数さん:04/11/19 12:13:32
>>564
(5) できるのか、これ? [>>617,>>627-631]
 0<a,b,c<1, a+b+c=s のとき, g(x)=1+x^r, f(x)=log|g(x)| とおくと
 f '(x) = g '(x)/g(x) = r・x^(r-1)/(1+x^r),
 f ''(x) = {g(x)g ''(x)-g '(x)^2}/{g(x)}^2 = r・(r-1-x^r)・x^(r-2)/[(1+x^r)^2].

 (i) r≧2 のとき f(x) は 0<x<1 で下に凸だから
 log(与式) = f(a)+f(b)+f(c) ≧ 3f(s/3) = 3log|g(s/3)|.
 f(x) ≦ f(0)(1-x) + f(1)x.
 ∴ g(s/3)^3 ≦ 与式 ≦ g(0)^(3-s)・g(1)^s.

 (ii) 0≦r≦1 のとき g(x)=1+x^r はx>0 で上に凸
 ∴ f(x)=log|g(x)| も上に凸: f(x) ≧ f(0)・(1-x) + f(1)・x
 ∴ log(与式) = f(a)+f(b)+f(c) ≧ f(0)・(3-a-b-c) + f(1)・(a+b+c) = f(0)・(3-s) + f(1)・s.
 ∴ g(0)^(3-s)・g(1)^s ≦ 与式 ≦ g(s/3)^3.

(6) k≧2 成立、 k<2 不成立。 [>>649]

[564]の解答のレス番号(主なもの)
 (1) [619] (2) [664] (3) [613] (4) [664] (5) [617][627] (6) [649]
ぬるぽ
666132人目の素数さん:04/11/20 04:19:07
>651(5)
遅レスですが、勉強になりました。ハァハァ…
667132人目の素数さん:04/11/20 09:05:59
>664(4)
なるへそ。条件式を変形した式から b-c=x, c-a=y とおくことに気づくわけか…
その後の nCr の計算に ハァハァ…
条件式から xy≧1 だし、x^k・y^(2n-2-k) も x^(2n-2-k)・y^k も正だから相加相乗か…
計算の後を追いかけるのがやっとです。とても自力で出来ません。

だめぽ > (:D)| ̄|_
668580:04/11/22 11:39:24
>587
【補題】A+B+C=π, 0<A,B,C<π のとき、
 0 < sin(A)sin(B)sin(C) ≦ {(3√3)/2π}^3 ABC ≦ (3√3)/8.
 -1 < cos(A)cos(B)cos(C) ≦ [1-cos(A)][1-cos(B)][1-cos(C)] ≦ 1/8.

 フランダースの不等式 とか言うらしい...
 http://mathworld.wolfram.com/FlandersInequality.html
ぬるぽ
669132人目の素数さん:04/11/22 13:26:17
キタ━(゚∀゚)━( ゚∀)━( ゚)━( )━(゚ )━(∀゚ )━(゚∀゚)━!!!!
670132人目の素数さん:04/11/22 13:52:11
>>388の証明(>>391)がよくわかんない。詳しく教えてくれ。
671132人目の素数さん:04/11/22 15:43:58
>668
パトラッシュ、疲れたろう。
右側は相加相乗だけど、左側はどうするんだろうね
   ,.-─-、
   / /_wゝ-∠l
   ヾ___ノ,. - >
   /|/(ヽY__ノミ
  .{   rイ  ノ

僕もう疲れたよ…
何だかとても眠いんだ…パトラ…
672668:04/11/22 19:23:28
>671
 f(x) = log|sin(x)| - log|x| とおいて f '(x) = 1/tan(x) -1/x,
 f ''(x)= -1/[sin(x)^2] +1/(x^2) <0(上に凸)を使うんだろうな・・・ネロ・・・

AAらしい
http://www.geocities.co.jp/Playtown-Darts/3851/fra.html
壁紙らしい
http://www.accessup.org/anime_j/Frandarce%2520dog.html
公式サイトらしい
http://www.bandaivisual.co.jp/flanders/fd_flmf.html
http://www.nippon-animation.co.jp/new/title/flanders/
http://www.nippon-animation.co.jp/na/flanders/
673132人目の素数さん:04/11/22 21:33:28
>670
[>>391]にしたがって、まず極座標(r,θ)に変換する。
 ∫∫{f(x)g(y)/(x+y)}dxdy = ∫{∫f(r・cosθ)g(r・sinθ)dr}/(cosθ+sinθ) dθ
次にrについてヘルダーの不等式を使う。
 ∫_[0,R]f(r・cosθ)g(r・sinθ)dr ≦ {∫_[0,R]f(r・cosθ)^p dr}^(1/p)・{∫_[0,R]g(r・sinθ)^q dr}^(1/q)
 = {∫_[0,R・cosθ]f(r')^p dr'}^(1/p)・{∫_[0,R・sinθ]g(r")^q dr"}^(1/q)・(1/cosθ)^(1/p)・(1/sinθ)^(1/q)
 → (1/cosθ)^(1/p)・(1/sinθ)^(1/q)・F^(1/p)・G^(1/q). (R→∞)
一方、θについての積分は、
 I = ∫_[0,π/2] 1/{(cosθ)^(1/p)・(sinθ)^(1/q)・(cosθ+sinθ)} dθ
となるが、ここで tanθ = t とおくと、cosθ=1/√(1+t^2), sinθ=t/√(1+t^2), dθ=dt/(1+t^2) を使えば
 I = ∫[0,∞) 1/{(1+t)t^(1/q)} dt = π/sin(π/p) となる。  (←[391][396])
∴ ∫∫{f(x)g(y)/(x+y)}dxdy ≦ {π/sin(π/p)}F^(1/p)・G^(1/q).
ぬるぽ
674132人目の素数さん:04/11/22 23:33:45
>>673
なるほど。
>>388に書かれてるように、真の不等号が成り立つっていうのは証明難しい?
675673:04/11/23 01:12:56
>674
 ヘルダーの不等式を使う所で、等号成立する場合がないことを示すのかなぁ??

 >>137-139, >>158 (総和形)
 http://www007.upp.so-net.ne.jp/masema/Lebesgue.html
676132人目の素数さん:04/11/23 01:47:53
>671
 フランダースの場合は あまりにも お犬さん
 フランダースの場合は あまりにも 優しい
 ベルギー アントワープの 大聖堂 雪の 朝に
 ネロと そっと ひとつ
 フランダース

 (98/11/09)
 http://www2.gol.com/users/maroon/kaenor-1.htm
677132人目の素数さん:04/11/23 04:13:26
>>672
なるへそ! 凸な関数に目をつけるんだね、パトラッシュ。
678132人目の素数さん:04/11/23 06:08:19
A+B+C=π, 0<A,B,C<π のとき、
 0 < sin(A)sin(B)sin(C) ≦ {(3√3)/2π}^3 ABC ≦ (3√3)/8

一方、相加相乗平均とJensenの不等式から
 0 < sin(A)sin(B)sin(C) ≦ ( [sin(A)+sin(B)+sin(C)]/3 )^3 ≦ (3√3)/8

気になるのが ( [sin(A)+sin(B)+sin(C)]/3 )^3 と {(3√3)/2π}^3 ABC の大小。
 sin(A)+sin(B)+sin(C) = 4cos(A/2)cos(B/2)cos(B/2)
だから、f(x) = log cos(x) - (1/3)log(x) とおくと f(A/2)+f(B/2)+f(C/2) と log((3√3)/4π) の大小。
f(x) の 0<x<π/2 における凹凸が一定でないので、わからんちん。

   ,.-─-、
   / /_wゝ-∠l
   ヾ___ノ,. - > 僕もう疲れたよ…
   /|/(ヽY__ノミ 何だかとても眠いんだ…パトラ…
  .{   rイ  ノ
679132人目の素数さん:04/11/23 23:02:55
>>647(5) の類題
[2003 Baltic Way] http://www.liis.lv/NMS/bw03/uzd/bw03eng.pdf

正の数 a, b, c に対して、2a/(a^2+bc) + 2b/(b^2+ca) + 2c/(c^2+ab) ≦ a/(bc) + b/(ca) + c/(ac)
       ___ 


彡     /  ≧ \    彡 ビュゥ……
  彡   |:::  \ ./ |  彡
      |:::: (● (●|    
      ヽ::::......ワ...ノ    ネタを仕入れるために
        人つゝ 人,,
      Yノ人 ノ ノノゞ⌒〜ゞ      旅立とう・・・
    .  ノ /ミ|\、    ノノ ( 彡
     `⌒  .U~U`ヾ    丿
             ⌒〜⌒
680132人目の素数さん:04/11/24 01:46:36
>>647  a+b+c=s, ab+bc+ca=t, abc=u とおくと, s^2 -3t≧0, st-9u≧0, t^2 -3su≧0. これでゴリゴリ

(1) 左辺 = (1+s+t+u)/(3+2s+t)- u/t = 1/3 + [(st-9u) +2(t^2 -3su)]/[3t(3+2st+u)] ≧ 1/3.

(2) 相加・相乗平均により (ca^2/b +2ab^2/c +4bc^2/a)/7 ≧ bc. これを循環的に加えて,
  (ca^2 +ab^2 +bc^2)(1/a +1/b +1/c) = (a^2 +b^2 +c^2) +(ca+ab+bc) +(ca^2/b + ab^2/c +bc^2/a)
  ≧ (a^2 +b^2 +c^2) +2(ab+bc+ca) = s^2. 
  左辺 = (ca^2 +ab^2 +bc^2)/u ≧ (s^2)/t,
  右辺 = {(s^3 -st +3u) +(ca^2 +ab^2 +bc^2)}/(st-u).
  [左辺−右辺]・(st-u) = (st-2u)(左辺) -(s^3 -st +3u) ≧ (st-2u)(s^2)/t -(s^3 -st +3u)
  = -2(s^2)u/t +st -3u ≧ -2st/3 +st -3u = (st-9u)/3 ≧0.

(3) 1/(2ab) = a/(2a^2b) ≧ a/(a^4+b^2).
  1/(2ab) = b/(2ab^2) ≧ b/(a^2+b^4).
  辺々たす。

(4) 左辺 - 中辺 = (y-z)x/(a+z) +(z-x)y/(a+x) +(x-y)z/(a+y)
  = [a(xz^2 +yx^2 +zy^2 -3xyz) +(t^2 -3su)]/[(a+x)(a+y)(a+z)] ≧0.
  右側: x/(a+x), y/(a+y), z/(a+z) の大小と a+x, a+y, a+z の大小は一致する。
  ∴ チェビシェフの不等式により成立。

(5) (a^2)/(a^2 +2bc) ≧ (a^2)/(a^2 +b^2 +c^2) などより 左辺≧1.
   bc/(a^2+2bc) ≦ (1/2)(b^2 +c^2)/(a^2 +b^2 +c^2) などより 右辺≦1.
  なお、(左辺) + (右辺)×2 = 3.

(5)の類題[>679]: 相加・相乗平均より 2a/(a^2 +bc) ≦ a/√(au) = √(au)/u ≦ (2a^2 +b^2 +c^2)/4u.
  循環的に加える。
681132人目の素数さん:04/11/24 01:51:26
>>647 (続き)
(6) [1993 Poland] 44th, 2nd roud, 1st day No.1 (1993.2)
  S=a+b+c+d, U=bcd+cda+dab+abc とおく。
  左辺 - 右辺 = (a+c)(b+d)/S -ab/(a+b) -cd/(c+d)
  = [(a+c)(b+d)(a+b)(c+d)-SU]/[S(a+b)(c+d)] = (ad-bc)^2 /[S(a+b)(c+d)] ≧0.

(7) b/c=x, c/a=y, a/b=z とおくと u=xyz=1.
  左辺 = x/(y+1) +y/(z+1) +z/(x+1) = [3u +(s^2 -2t) +t +s]/[(x+1)(y+1)(z+1)].
  右辺 = 1/(y+1) +1/(z+1) +1/(x+1) = (t+2s+3)/[(x+1)(y+1)(z+1)].
  [左辺−右辺]・(x+1)(y+1)(z+1) = (s^2 -2t) -s
  = (1/2)[(x^2 +y^2 +z^2 -3) +(x-1)^2 +(y-1)^2 +(z-1)^2] ≧0.
ぬるぽ
682132人目の素数さん:04/11/24 11:27:48
>680-681 さすが不等式王!
                   ___,
  ┏┓  ┏━━┓      /  ≧ \.         .┏┓┏┓
┏┛┗┓┃┏┓┃      |:::: \ ./ |          ┃┃┃┃
┗┓┏┛┃┗┛┃┏━━━|::::: (● (● |━━━━━┓┃┃┃┃
┏┛┗┓┃┏┓┃┗━━━ヽ::::... .ワ.....ノ!━━━━━┛┗┛┗┛
┗┓┏┛┗┛┃┃      ( (つ  丿ノ         . ┏┓┏┓
  ┗┛      ┗┛        ( ヽノ             ┗┛┗┛
                   し(_)  
683132人目の素数さん:04/11/24 11:29:45
どうでもいいことだけど、>>392 + >>592 を合わせて…

非負実数 a, b, c, d に対して次式が成立。
4(a^3+b^3+c^3+d^3) + 24(abc+bcd+cda+dab) ≧ (a+b+c+d)^3 ≧ (a^3+b^3+c^3+d^3) + 15(abc+bcd+cda+dab)


(;´д`)ハァハァして書いた。
不等式なら何でもよかった。
今は反省している。
684132人目の素数さん:04/11/24 13:56:12
a,b,c≧0 のとき、 a^3+b^3+c^3+6abc ≧ (ab+bc+ac)(a+b+c)
(証明)
差をとれば、Schurの不等式のλ=1 の場合になる。(不等式への招待 P.28)
左辺-右辺 = a(a-b)(a-c)+b(b-c)(b-a)+c(c-a)(c-b) ≧ 0

一方、>>601 a,b,c≧0 のとき、 2(a+b+c)^3 +9abc ≧ 7(ab+bc+ac)(a+b+c) が成立。

そこで気になるのが、7(a^3+b^3+c^3+6abc) と 2(a+b+c)^3 +9abc の大小。
a+b+c=s, ab+bc+ca=t, abc=u とおくと、a^3+b^3+c^3 = s^3-3st+3u より

7(a^3+b^3+c^3+6abc)-[2(a+b+c)^3 +9abc]
= 5s^3-21st+54u
= 2(2s^3-7st+9u)+s(s^2-3t)-4(st-9u)

うまくいかんちん…
s, t, u に成り立つ不等式は、 s^3-27u ≧ 0 とか、
>>42より、2s^3-7st+9u = (a+b)(a-b)^2+(b+c)(b-c)^2+(c+a)(c-a)^2 ≧ 0
    ___
  ./  ≧ \  
  |::::  \ ./ |    よろしくおねがいします!
  |::::: (● (● |
  ヽ::::.... ワ ....ノ /  チン ☆    たのも〜
 _(  ⊃  ⊃  チン ☆        たのも〜
 |\ ̄ ̄ ̄ ̄旦 ̄\
 | | ̄ ̄ ̄ ̄ ̄ ̄ ̄|
 \|  愛媛みかん |
    ̄ ̄ ̄ ̄ ̄ ̄ ̄
685132人目の素数さん:04/11/24 14:23:47
非負実数 a, b, c に対して、(;´д`)ハァハァ できそうな不等式

(a+b)(a^4+b^4) ≧ (a^2+b^2)(a^3+b^3)
(a+b+c)(a^4+b^4+c^4) ≧ (a^2+b^2+c^2)(a^3+b^3+c^3)

これって、一般化できるのでせうか?
686132人目の素数さん:04/11/24 14:27:37
>>685
>>153とは違うのか?
687132人目の素数さん:04/11/24 15:59:55
>686 ( ̄ワ ̄;)! 見逃してた。ありがとうございます。

>153 x_k≧0 (k=1,2,・・・,m)のとき、 F(n)≡(1/m)Σ[1≦k≦m] (x_k)^n とおくと
> (補題) bc≧0 ⇒ F(a)・F(a+b+c) ≧ F(a+b)・F(a+c).
> (系1) F(n-1)・F(n+1) ≧ {F(n)}^2.
> (系2) {F(n)}^(1/n) はnに関して単調増加.

補題で、a=b=1, c=2, m=2, 3 の場合が >685ですね。
ちなみに153は、「モノグラフ4 不等式 P.57」より九州大の問題。
ところで、153の証明って…
688132人目の素数さん:04/11/24 22:17:04
>687
 項別に比べれ。
 >>153 の(補題)はさくらスレ145にありますた...

109 :PrinceMathematician◇ :04/05/27 12:02

(補題) bc≧0 ⇔ F(a)・F(a+b+c) ≧ F(a+b)・F(a+c).

  F(a)・F(a+b+c)−F(a+b)・F(a+c)
  = (1/m)^2 Σ[1≦i≦m][1≦j≦m] (x_i・x_j)^a・{x_j^(b+c)-(x_i)^b(x_j)^c}
  = (1/m)^2 Σ[1≦i<j≦m] (x_i・x_j)^a・{x_i^(b+c)+(x_i)^b・(x_j)^c+(x_j)^b・(x_i)^c+x_j^(b+c)}
  = (1/m)^2 Σ[1≦i<j≦m] (x_i・x_j)^a・{(x_i)^b-(x_j)^b}{(x_i)^c-(x_j)^c}
  〜 {(x_i-x_j)b}{(x_i-x_j)c} 〜 bc. 
 〜は同符号の意味。 等号成立は x_1=x_2=・・・=x_m のとき. (終)
689132人目の素数さん:04/11/24 22:20:55
>679 >>647(5) の類題
[2003 Baltic Way] http://www.liis.lv/NMS/bw03/uzd/bw03eng.pdf

正の数 a, b, c に対し、
 A = a/(bc) + b/(ca) + c/(ab)
 B = 1/a + 1/b + 1/c
 C = 1/√(ab) + 1/√(bc) + 1/√(ca)
 D = 2a/(a^2+bc) + 2b/(b^2+ca) + 2c/(c^2+ab)
 E = 3/\sqrt[3](abc)
 F = 2/(a+b) + 2/(b+c) + 2/(c+a)
 G = 6/\sqrt[3]{(a+b)(b+c)(c+a)}
 H = 9/(a+b+c)
とおくと
 A ≧ B ≧ C ≧ D
 C ≧ (E, F) ≧ G ≧ H
E, Fの大小は定まらない。(a, b, c) = (1, 1, 27), (1, 1, 1), (1, 1, 8) のとき、≧、=、≦。
    ___
  ./  ≧ \  
  |::::  \ ./ |    D と (E, F) ≧ G ≧ H は
  |::::: (● (● |        ドッキングできないですか?
  ヽ::::.... ワ ....ノ /  チン ☆    たのも〜
 _(  ⊃  ⊃  チン ☆        たのも〜
 |\ ̄ ̄ ̄ ̄旦 ̄\
 | | ̄ ̄ ̄ ̄ ̄ ̄ ̄|
 \|  愛媛みかん |
    ̄ ̄ ̄ ̄ ̄ ̄ ̄
690132人目の素数さん:04/11/24 22:22:05
>>688
おっ! 書き込んでいる間に。
ありがとうございます。
さっそく印刷して読みます。
691651:04/11/24 22:53:06
>683
 どうでもいいことだけど >>562 (5) を合わせて・・・ ↓が成り立たんかな?

 4(a^3+b^3+c^3+d^3) + 15(abc+bcd+cda+dab) ≧ (a+b+c+d)^3.
692683:04/11/24 22:58:41
>691
> 683 名前:132人目の素数さん[sage] 投稿日:04/11/24(水) 11:29:45
> どうでもいいことだけど、>>392 + >>592 を合わせて…

>>392 + >>562 を合わせて…」 の書き間違いでした。
693132人目の素数さん:04/11/25 00:39:29
>>691
成り立つね。
694683:04/11/25 02:25:58
>651の証明から成り立ってますね。dクス。691,693
(:D)| ̄|_
695132人目の素数さん:04/11/25 03:06:07
>>689
(1,1,2)を代入してみたら H≧D だけど、もしかして常に成り立っているのかな。
696132人目の素数さん:04/11/25 20:00:14
成り立ってないよ
697132人目の素数さん:04/11/26 09:48:46
うちのおじいちゃんの名前 成田多内
うちのおばあちゃんの名前 成田つね
698132人目の素数さん:04/11/26 10:01:07
>>696
反例キボンなり
699132人目の素数さん:04/11/26 10:51:09
a = 3, b = 1, c : きわめて 0 に近い数
700132人目の素数さん:04/11/26 11:01:32
>>699
ぉぉー、dクス
701132人目の素数さん:04/11/26 11:43:46
>>697
ageると荒らしの目にとまる一例。
702132人目の素数さん:04/11/26 12:44:30
703132人目の素数さん:04/11/26 17:21:41
うちのおばあちゃんの名前 成田たね
704132人目の素数さん:04/11/26 20:31:40
(1) [Komal. A358] http://www.komal.hu/verseny/2004-11/mat.e.shtml
正の数 a, b, c が abc=1 をみたすとき
 1/a + 1/b + 1/c - 3/(a+b+c) ≧ 2(1/a^2 + 1/b^2 + 1-c^2)/(a^2+b^2+c^2)

(2) [AMM 2003.10 Prob.10944]
正の数 a, b, c が abc≧2^9 をみたすとき
 1/√(1+a) + 1/√(1+b) + 1/√(1+c) ≧ 3/√(1+\sqrt[3]{abc})

以下の問題は [Bonus Problems 2] http://www.dpmms.cam.ac.uk/~par31/bmos/0304/ineqsOundle.pdf

(3) 正の数 a, b, c が abc=1 をみたすとき、a^5 + b^5 + c^5 ≧ 1/a + 1/b + 1/c

(4) 実数 a, b, c, d が ad-bc=1 をみたすとき、a^2 + b^2 + c^2 + d^2 + ac + bd ≧ √3

(5) 三角形の3辺の長さ a, b, c が a+b+c=1 をみたすとき、23/216 ≦ a^2b+b^2c+c^2a ≦ 1/8

(6) 正の数 a, b, c, d, e が abcde=1 をみたすとき
 (a+abc)/(1+ab+abcd) + (b+bcd)/(1+bc+bcde) + (c+cde)/(1+cd+cdea) + (d+dea)/(1+de+deab) + (e+eab)/(1+ea+eabc) +≧ 10/3

(7) 正の数 a, b, c に対して、a^4 + b^4 + c^4 + [(a+b+c)^4]/27 ≧ 2(a^2b^2 + b^2c^2 + c^2a^2)

(8) 正の数 a, b, c に対して、(a+b+c)(1/a + 1/b + 1/c) ≧ 9 + 3(1- a/b)^(2/3) + 3(1- b/c)^(2/3) + 3(1- c/a)^(2/3)
       ___ 
彡     /  ≧ \    彡 ビュゥ……
  彡   |:::  \ ./ |  彡
      |:::: (● (●|    
      ヽ::::......ワ...ノ    ネタを仕入れてきました
        人つゝ 人,,
      Yノ人 ノ ノノゞ⌒〜ゞ      
    .  ノ /ミ|\、    ノノ ( 彡
     `⌒  .U~U`ヾ    丿
             ⌒〜⌒
705132人目の素数さん:04/11/26 22:17:44
(問題)
 a,b>2 のとき、{1/(a+b)}^(1/a) + {1/(a+b)}^(1/b) >1.
 おながいします。

>703
 ねた足りな...
706662:04/11/26 23:06:45
>>663
 662の(7)を修正
 左辺の (1-x^n)^m =f(x) の下限を考える。
 a=1-[1/(m+n)]^(1/m), b=[1/(m+n)]^(1/n) とおくと, 0<a≦b<1. (←[705])
 f_1(x) = 1-m(x^n)  (0≦x≦b),
 f_2(x) = n(1-x)^m  (a≦x≦1) とおく。
f_1, f_2 とも単調減少で、 f_1(x) > f_1(b) = n/(m+n) = f_2(a) > f_2(x) (a<x<b)
 次に、これらが f(x) の下限となることを示す。
 (1) f(x) ≧ f_1(x) (0≦x≦b)
 (2) f(x) ≧ f_2(x) (a≦x≦1)
 (略証)
  (1) 相加・相乗平均を使う: X^m +(m-1) ≧ mX から。
  (2) f(a) > f_1(a) > f_1(b) = f_2(a).
   また f(x)^(1/m) = 1-x^n は上に凸、f_2(x)^(1/m)は直線だから
    f(x)^(1/m) ≧ [f(a)^(1/m)/(1-a)](1-x) ≧ [f_2(a)^(1/m)/(1-a)](1-x) =f_2(x)^(1/m).

 同様にして 1-f_1(x) (0≦x≦b), 1-f_2(x)(a≦x≦1)は [1-(1-x)^m]^n の下限となる。
 それぞれの区間でこれらを加えれば ≧1 となる。(終)
ぬるぽ  
707662:04/11/27 03:28:59
>706 の補足
 m=1 or n=1 のときは、等号成立。
 m=2, n>1 のときは、左辺 - 右辺 = (x^n){x^n +(2-x)^n -2} ≧ 0. (← y=x^n は下に凸)
 よって m,n>2 の場合を考えればよい。
ぬるぽ
708132人目の素数さん:04/11/30 21:41:42
>>681(7) ab/[c(c+a)] + bc/[a(a+b)] + ca/[b(b+c)] ≧ a/(c+a) + b/(b+a) + c/(c+b)
> b/c=x, c/a=y, a/b=z とおくと u=xyz=1.
> 左辺 = x/(y+1) +y/(z+1) +z/(x+1) = [3u +(s^2 -2t) +t +s]/[(x+1)(y+1)(z+1)].

左辺の分子は、xy^2+yz^2+zx^2+(s^2-2t)-s-3 となりますが…
709132人目の素数さん:04/11/30 22:52:14
>>708
> 左辺 = x/(y+1) +y/(z+1) +z/(x+1) = [3u +(s^2 -2t) +t +s]/[(x+1)(y+1)(z+1)].

2番目の等号は≧の間違えでしょ。
710132人目の素数さん:04/12/01 22:05:27
>>709
なるほど。dクスです。
711132人目の素数さん:04/12/01 23:11:56
マニアックな不等式教えてくれ
712132人目の素数さん:04/12/01 23:30:26
   ___  >711
 ./  ≧ \   -3≦r≦3、
 |::::  \ ./ |   a_1, …, a_n ≧ 1
 |::::: (● (● |   G を a_1, …, a_n の相乗平均とするとき
 ヽ::::... .ワ....ノ     
 ̄ ̄   \     Σ[k=1 to n](1+a_k)^r ≧ G^r*Σ[k=1 to n]{1+ 1/(a_k)}^r
フ     /ヽ ヽ
713132人目の素数さん:04/12/02 10:30:53
>>711
いちいち上げんな、ボケ!
714132人目の素数さん:04/12/02 21:38:22
>>704(3) 次式を回して加える。等号は a=b=c=1。
a^5+a^5+b^5+b^5+c^5 ≧ 5\sqrt[5]{a^(10)b^(10)c^5} = 5/c
715705:04/12/03 11:35:38
 まづ y = x^(2/a) 上の点(1,1)で接線を引きまつ: y = 1+(2/a)(x-1).
 a<2 のときは下に凸、a>2 のときは上に凸ゆえ、
 {x^(2/a)- 1 -(2/a)(x-1)}(2-a) = {x^(2/a) -(a+2x-2)/a}(a-2)≧0.
 ∴ {(1/x)^(2/a) -a/(a+2x-2)}(a-2) ≧0.
 これをベルヌーイの不等式とか言うらしい.....
a,b>2 のとき
 {1/(a+b)}^(1/a) = {1/√(a+b)}^(2/a) = {(1/2)^(2/a)}・{2/√(a+b)}^(2/a)
 = {(1/2)^(2/a)}・{2/√(a+b)}^(2/a) ≧ {(1/2)^(2/a)}・a/{a+√(a+b)-2}.
 {1/(a+b)}^(1/b) ≧ {(1/2)^(2/b)}・b/{b+√(a+b)-2}.
 左辺 ≧ {(1/2)^(2/a)}・a/{a+√(a+b)-2} + {(1/2)^(2/b)}・b/{b+√(a+b)-2}.
これが >1 であることを示したいんでつが...  増すますむずぽ.....
716132人目の素数さん:04/12/04 20:19:09
>>704(1)(6) を教えてたもれ。
さっぱりわからん。
717132人目の素数さん:04/12/05 05:17:19
早く教えれ
718132人目の素数さん:04/12/05 09:28:15
>>717
上げんなボケ!氏ね!
719132人目の素数さん:04/12/05 15:28:39
>704(1), 716-717
 a+b+c=s, ab+bc+ca=t, abc=u とおくと s^3 -4st+9u≧0, s^2-3t≧0. [>>611]
 左辺 = (t/u) - (3/s),
 右辺 = 2(t^2 -2su)/[(s^2 -2t)u],
 左辺 - 右辺 = {(s^3 -4st+9u)t +(s^2 -3t)}/[su(s^2 -2t)] ≧0.
ぬるぽ
720711:04/12/05 15:58:52
>712
 nに関する帰納法による。
 n=2のとき、a^(r/2)=A, b^(r/2)=B とおくと G^r = AB, B-A 〜 (b-a)r. (〜は同符号の意味)
 左辺 = (1+a)^r +(1+b)^r = A・(√a +1/√a)^r + B・(√b +1/√b)^r.
 右辺 = (G^r)・{(1/A)(√a +1/√a)^r + (1/B)(√b +1/√b)^r}.
    = B・(√a +1/√a)^r + A・(√b +1/√b)^r.
 f(x)=√x +1/√x は x≧1 では単調増加だから、f(b)-f(a) 〜 b-a.
 ∴ 左辺 - 右辺 ≡ d(a,b) = (A-B)・{(√a +1/√a)^r - (√b +1/√b)^r} ≧ 0.
 d(a,b)は ある意味での「距離」である。

 n>2 のとき、
 a_1=・・・・・・=a_n のとき、等号成立。
 そうでないとき、a_{n-1} < G < a_n (または逆) としても一般性を失わない。
 c_{n-1}=a_{n-1}・a_n/G, c_n=G とおくと、相乗平均Gは変わらない。
 a_{n-1} < c_{n-1}, G < a_n (または逆) だから d(a_{n-1},a_n) ≧ d(c_{n-1},G)
 ∴ F_n(a_1, ・・・・・・, a_n) ≧ F_n(a_1, ・・・・・・, a_{n-2}, c_{n-1}, G)
  = F_{n-1}(a_1, ・・・・・・, a_{n-2}, c_{n-1})
 となるが、帰納法の仮定により 右辺≧0 である。(終)
> -3≦r≦3、
 要りまつか?
むずぽ
721132人目の素数さん:04/12/05 18:47:31
>>720
神キタ━(゚∀゚)━!!!!
発掘元は 「不等式への招待 P.51 注意」 からです。
その本によれば、P.49の例8を用いて証明したそうなので
条件 -3≦r≦3 がついたのかな…
722132人目の素数さん:04/12/05 18:56:20
>>565(7) をたのも〜。なんかできません。

 s=(a+b+c)/2, x=(b+c-a)/2, y=(c+a-b)/2, z=(a+b-c)/2
とおくと、三角形の成立条件から x, y, z>0 で
 s=x+y+z, a=s-x, b=s-y, c=s-z
また t=xy+yz+zx, uxyz とおくと、
 (左辺)-(右辺) = (2s^3-5st-3u)/u - 18a = (2s^3-5st-3u-18su)/u
とここまでやって…
     ダメポ >(|ン:()| ̄|_
723132人目の素数さん:04/12/05 20:45:54
a=b=c=1で成り立ってないけど。
724凡例:04/12/06 00:25:20
>720
r<-3, 3<r には反例が...
 a_1=1, a_k=2 (k=2 to n), n=10, G=2^[(n-1)/n]=1.8660659830736, r=4 のとき,
 左辺 = 745 < 746.49040900637 = 右辺.
 a_1=2, a_k=1 (k=2 to n), n=10, G=2^(1/n)=1.0717734625363, r=-4 のとき,
 左辺 = 0.574845679012346 < 0.575995685961706 = 右辺.
したがって -3≦r≦3 は必要と思われ。
725720:04/12/06 01:35:12
>724
 dクス.
 2個の相乗平均√(a_{n-1}a_n)が全体の相乗平均Gと異なるため、そのままの比較は無理のようでつ。 
すまそ。
726132人目の素数さん:04/12/07 16:45:29
>>565(7)は、通分してゴリゴリするのは地獄を見そうだし…
なにかいい手があるのかな?
727132人目の素数さん:04/12/07 21:48:15
>722,726
 a,b,cに上限があるですか。相似拡大していったら成り立たな伊予柑...
ぬるぽ
728132人目の素数さん:04/12/07 22:12:12
>>704(6) について、(a+abc)/(1+ab+abcd) = a(1+bc+bcde)/(1+ab+abcd) - 1/(1+ab+abcd)
これと相加相乗を使えば、示すべき不等式は
 1/(1+ab+abcd) + 1/(1+bc+bcde) + 1/(1+cd+cdea)+ 1/(1+de+deab) + 1/(1+ea+aabc) ≦ 5/3
ここまでいけたけど…。まさか、あとは差をとるのですか?
(´д`;)ガクガクブルブル
729132人目の素数さん:04/12/07 22:30:19
>>728
>  1/(1+ab+abcd) + 1/(1+bc+bcde) + 1/(1+cd+cdea)+ 1/(1+de+deab) + 1/(1+ea+aabc) ≦ 5/3
この式は成り立たない。

最初から、a=q/p, b=r/q, c=s/r, d=t/s, e=p/t と置き換えると式が見やすくなる。
後は、相加相乗でもコーシーシュヴァルツでもお好きなように。
730132人目の素数さん:04/12/07 22:55:33
>>729 サンクス。
正の数 p, q, r, s, t を用いて a=q/p, b=r/q, c=s/r, d=t/s, e=p/t とおくと、示すべき不等式は
 (q+s)/(p+r+t) + (r+t)/(q+s+u) + (s+u)/(r+t+p) + (t+p)/(s+u+q) + (u+q)/(t+p+r) ≧ 10/3
ここから頑張るんですね。やってみまつ。
731132人目の素数さん:04/12/07 22:59:28
>>730
しまった、uはなかった…
732132人目の素数さん:04/12/07 23:14:14
示すべき不等式は
 (p+q+r+s+t){1/(p+r+t) + 1/(q+s+p) + 1/(r+t+q) + 1/(s+p+r) + 1/(t+q+s)} ≧ 25/3
これは Caushy-Schwarzの不等式から成立。
(;´д`)ハァハァ
733132人目の素数さん:04/12/08 00:16:34
>>723>>727
>>565(7)の発掘元をやっと再発見。下の2ページ目。
http://www.math.nwu.edu/~mlerma/problem_solving/problems/math_mag-75-5-dec02.pdf
734132人目の素数さん:04/12/08 02:30:20
>>704(7)
  a^4+b^4+c^4 = 2(a^2b^2 + b^2c^2 + c^2a^2)-(a+b+c)(a+b-c)(b+c-a)(c+a-b)
かつ a+b+c>0 だから、示すべき不等式は
  (a+b+c)^3 ≧ 27(a+b-c)(b+c-a)(c+a-b)
s=a+b+c, t=ab+bc+ca, u=abc とおくと、
 (左辺)-(右辺) = s^3-27(s-2a)(s-2b)(s-2c) = 4[(2s^3-7st+9u)+5(s^3-4st+9u) ≧ 0
等号成立条件は a=b=c。

〔蛇足〕
2s^3-7st+9u = (a+b)(a-b)^2+(b+c)(b-c)^2+(c+a)(c-a)^2 ≧ 0
s^3-4st+9u = a(a-b)^2 +c(b-c)^2 +(a-b)(b-c)(a-b+c) ≧0 [>>611]


>>729
>最初から、a=q/p, b=r/q, c=s/r, d=t/s, e=p/t と置き換えると式が見やすくなる。
この置き換えに (;´д`)ハァハァ
735132人目の素数さん:04/12/08 02:35:24
>>704(4)の方針が立ちません。たのも〜。
736132人目の素数さん:04/12/08 02:46:45
>>735
a = p+(q/√3), b = -p+(q/√3), c = r+(s/√3), d = -r+(s/√3) と置き換えると式が見やすくなる。
737132人目の素数さん:04/12/08 02:59:26
>>704 735
xy座標導入して三点O(0,0)、A(a,b)、B(c,d)を考える。これら三点がなす三角形の面積は1/2
また、問題は原点中心に三点を回転させても一般性を失わないので、(0,0)、(a,0)、(c,d)と置き直してもよい。
従って、題意を示すにはad=1を満たす時、a^2+c^2+d^2+ac≧√3を示せという問題に置き換えることができる。
d=1/aより、dを消去し
a^2+c^2+(1/a^2)+ac
≧(1/a^2) + (3a^2/4)
≧√3

ぬるぽ
738132人目の素数さん:04/12/08 03:43:30
>>736-737 ありがとうございます。やってみます


>>680
>>647(4)の解答の右側の証明についてですが、チェビシェフの不等式より
  3(x+y+z) ≧ {(a+x)+(a+y)+(a+z)}・{x/(a+x) + y/(a+y) + z/(a+z)}
展開して整理すると
  2(x+y+z) ≧ {(a+z)・x/(a+x) + (a+x)・y/(a+y) + (a+y)・z/(a+z)} + {(a+y)・x/(a+x) + (a+z)・y/(a+y) + (a+x)・z/(a+z)}
となるから、「右辺第2項 ≧ 右辺第1項」 を示せば
  2(x+y+z) ≧ 2(右辺第1項) … (★)
となって完成ですよね。(発掘元の問題が誤植ですが…)

そこで (★) を証明したいのですが、これはどうするのでしょうか? 
739132人目の素数さん:04/12/08 03:44:52
>>738の書き間違い

「右辺第2項 ≧ 右辺第1項」 …(★) をどうやって示すのですか?
740132人目の素数さん:04/12/08 04:32:02
>>737
なるへそ!
図形で考えるとは思いもしなかったです。
(;´д`)ハァハァ
741132人目の素数さん:04/12/08 04:52:53
>738
a_i, b_j が共に単調増加列(単調減少列)のとき、
 Σ[i=1,n] a_i・b_{n+1-i} ≦ Σ[j=1,n] a_j・b_Pj ≦ Σ[k=1,n] a_k・b_k
を使ったと思われ。P(j)は任意の順列。
742741:04/12/08 10:01:40
[741] の略証
 S= Σ[j=1,n] a_j・b_Pj とおく。Pは {1,2,・・・・,n} に作用する置換・・・・・スマソ.
 Pは互換(2元の交換)を何回か続けたものだが、互換については下が成り立つ。
 a<A, b<B のとき、(AB+ab)-(Ab+aB) = (A-a)(B-b) ≧0. a>A, b>B のときも同じ。
∴Pの転置数を減らすとSは増大し、転置数を増やすとSは減少する。
∴ P(j)=n+1-n のときSは最小で、P=I のときSは最大(終)。
743132人目の素数さん:04/12/08 14:58:52
>741-742
ありがとうございます。
数学オリンピック事典P.165にある 「並べ替え不等式」 とかいうやつですね。
 (同順序積の和) ≧ (乱順序積の和) ≧ (逆順序積の和)

a+x, a+y, a+z と x/(a+x), y/(a+y), z/(a+z) の大小は一致するから、>742の証明から、
同順序積からの転置数が小さいほど大きいことが分かるんですよね。

a+x, a+y, a+z を x, y, z で、x/(a+x), y/(a+y), z/(a+z) を X, Y, Z と略すと、>739の (★) は
 yX+zY+xZ ≦ zX+xY+yZ … (☆)
最も大きい (同順序積の和) = xX+yY+zZ から見ると、(☆) の両辺の転置数はともに2だから
  xX+yY+zZ ≧ zX+yY+xZ ≧ yX+zY+xZ
  xX+yY+zZ ≧ zX+yY+xZ ≧ zX+xY+yZ
であることは分かるけれども、(☆) が成り立つことは分からないような気がしますが…。
744132人目の素数さん:04/12/08 15:00:04
>>743 不等号の向きの下記間違い…
> yX+zY+xZ ≧ zX+xY+yZ … (☆)
745132人目の素数さん:04/12/08 15:11:41
>>743-744
なんでわざわざ遠回りする?

x=(a+x)*(x/(a+x))

  x+y+z = (a+x)*(x/(a+x)) + (a+y)*(y/(a+y)) + (a+z)*(z/(a+z))
≧ (a+z)*(x/(a+x)) + (a+x)*(y/(a+y)) + (a+y)*(z/(a+z))

741
746132人目の素数さん:04/12/08 15:16:05
たとえば x≧y≧z、X≧Y≧Z のとき、
 (同順序積) = xX+yY+zZ ≧ zX+yY+xZ = (逆順序積)
が成り立ちますが、転置数は1ですよね。

単純に転置数が多いほど小さくなるとすると、(逆順序積)よりも
転置数の大きい yX+zY+xZ や zX+xY+yZ のほうが小さくなって…。

何か混乱してしまいました。解説をよろしくお願いします。
  il||li _| ̄|○ il||li
747132人目の素数さん:04/12/08 15:20:49
>>745
( ̄□ ̄;)!!
ありがとうございます。こんな簡単なことだったのか…
748132人目の素数さん:04/12/08 15:22:48
>>746
> 単純に転置数が多いほど小さくなるとすると
ここが変だよん
749132人目の素数さん:04/12/08 15:28:43
>748
嗚呼、なるほど。
転置数の数でなくて、転置する2組の大小関係に注目しないといけないんですね。
転置する aX+bY と aY+bX について同順序積のほうが大きいんでしたね。
勘違いしてました…



晒し上げられてしまったけど、少し賢くなったので良し…
  il||li _| ̄|○ il||li
750742:04/12/08 17:59:00
>748-749
 たしかに私の説明が変だな。スマソ。
751132人目の素数さん:04/12/08 19:59:43
x>1, y>1, z>1, 1/x + 1/y + 1/z = 2 のとき、
√(x+y+z)≧√(x-1)+√(y-1)+√(z-1)
を示せ。
752132人目の素数さん:04/12/08 20:21:47
趣旨違うかも知れんが
任意の実数xに対して、
x^4 - x^3 + x^2 - x + 21/64 > 0
を示せ。
753132人目の素数さん:04/12/09 01:59:50
>>737
ふと疑問に思ったんですが、O(0,0)、A(a,b)、B(c,d)の△OABを原点の周りに回転させると
O(0,0)、A(\sqrt{a^2+b^2}, 0)、B(*,*)←略 だから、(0,0)、(a,0)、(c,d)と置き直すのはマズイかなと…

その証明では b=0 の特別な場合を証明したに過ぎないような気がするのですが…
754132人目の素数さん:04/12/09 02:02:11
>>753
そーかな?
文字を新しく置き直したと見ればいいんじゃない?
755132人目の素数さん:04/12/09 02:05:07
>>753
a^2+b^2 = OA^2
c^2+d^2 = OB^2
ac+bd = OA*OB*cos∠AOB
ad-bd = △OABの面積( ただし、符号つきで考える )

で△OABを原点中心に回転する。
756132人目の素数さん:04/12/09 02:20:13
>>754-755
アホな質問に付きあってもらって、ありがとうございます。
回転させて、頂点を改めて(0,0)、(a,0)、(c,d) と置き直すと、
考察する不等式も、変わってしまうと思うんです。
757132人目の素数さん:04/12/09 02:38:27
う〜〜ん、実は俺もよく分からないのだが
△ABCの面積が2の時
OA^2+OB^2+OA・OB≧√3
を示せという問題だと解釈すれば、いくら回転しても題意には影響を与えず、
回転した後、Aのy座標が0になるようにすればいいのではないか?

という事なんじゃない?
758132人目の素数さん:04/12/09 03:15:32
>>757
・ 文字を置き換える → 変数変換 → 考察する不等式も変数変換すべきでは?
・ 文字だけ置き換えて考察する不等式は変えない → b=0 という特別な場合を考察
だと思うんですよね。
で、>755のヒントにしたがってゴリゴリ証明してみました。

O(0,0)、A(a,b)、B(c,d)の△OABの面積をS、∠AOB=θとおくと、条件式は
 1 = |ad-bc| = 2S = √(|OA|^2|OB|^2-(OA・OB・cosθ)^2) … (★)
より、|OA|・|OB| = 1/sinθだから、
 a^2+b^2+c^2+d^2+ac+bd = |OA|^2+|OB|^2+OA・OBcosθ = (|OA|-|OB|)^2+(2+cosθ)/sinθ
ここで、f(θ) = (2+cosθ)/sinθ の最小値を 0<θ<π で考えると、
 f'(θ) = -(1+2cosθ)/(sinθ)^2
より、θ=2π/3 で極小かつ最小値をとる。したがって
 (★) ≧ f(θ) ≧ f(2π/3) = √3
759132人目の素数さん:04/12/09 03:29:54
三点(0,0)、(a,b)、(c,d)に対し回転行列
a/√(a^2+b^2)   ,   b/√(a^2+b^2)
-b/√(a^2+b^2)   ,   a/√(a^2+b^2)
を作用させる。
行列を
X  Y
-Y  X
とかけば

(a,b) → ( Xa+Yb , 0)
(c,d) → ( Xc+Yd , -Yc+Xd )

a^2+b^2+c^2+d^2+ac+bd≧√3が成立するならば
(Xa+Yb)^2 + 0^2 + (Xc+Yd)^2 + (-Yc+Xd)^2 + (Xa+Yd)(Xc+Yd)≧√3
も成立するはず
新しく
aをXa+Yb
bを0
cをXc+Yd
dを-Yc+Xd
で置き直せば>>737になると思う。
760132人目の素数さん:04/12/09 12:07:50
>752
 趣旨違うかも知れんが
 x-(1/4) =X とおくと、任意の実数Xに対して、
 左辺 = X^4 +(5/8)X^2 -(5/8)X + (33/256) = (X^2 -1/8)^2 +(7/8)(X-5/14)^2 + (3/1792) >0.
ぬるぽ
761132人目の素数さん:04/12/09 14:36:45
>>759
ありがとうございます。
762132人目の素数さん:04/12/09 14:52:51
>>751
 (x-1)/x + (y-1)/y + (z-1)/z = 3-(1/x + 1/y + 1/z) = 1
だから、Caushy-Schwarzの不等式より
 x+y+z = (x+y+z){(x-1)/x + (y-1)/y + (z-1)/z} > {√(x-1)+√(y-1)+√(z-1)}^2
両辺の平方根を取れば、示すべき不等式を得る。

もし等号が成立するとしたら、2つのベクトルが平行のときだから、
 k(√a, √b, √c,) = (√(x-1), √(y-1), √(z-1))
これを条件式下で解くと、a=b=c=2/3 となるので、根号内条件に反する。
763132人目の素数さん:04/12/09 15:45:22
>>762
最後の部分が変ですよ。
764132人目の素数さん:04/12/09 15:50:52
>>763
dクス。

もし等号が成立するとしたら、2つのベクトルが平行のときだから、
 k(√x, √y, √z,) = (√{(x-1)/x}, √{(y-1)/y}, √{(z-1)/z})
これを条件式下で解くと、a=b=c=2/3 となるので、根号内条件に反する。
765132人目の素数さん:04/12/09 15:55:44
>>764
x=y=z=3/2で等号成り立つでしょ。
766132人目の素数さん:04/12/09 16:11:30
>>765
あ、ほんとだ…
767132人目の素数さん:04/12/09 16:15:00
>>762
訂正
等号成立条件は
 k(√x, √y, √z,) = (√{(x-1)/x}, √{(y-1)/y}, √{(z-1)/z})
を条件式下で解いて x=y=z=3/2

  il||li _| ̄|○ il||li
768132人目の素数さん:04/12/09 21:22:52
x>0, x≠1 のとき
log(x)/(x^3-1) < (x+1)/{3(x^3+x)}
を示せ。
769132人目の素数さん:04/12/09 21:35:43
>>768
高校生の宿題は、質問スレに書け!
770727:04/12/09 21:38:36
>>565(7)
 [733] により修正された問題。 まづ余弦定理を使って a,b,c で表わす。
 左辺 = (b+c)cosA)/(a^2) + (c+a)cosB/(b^2) + (a+b)cosC/(c^2)
 = {(b+c)(b^2 +c^2 -a^2)/a +(c+a)(c^2 +a^2 -b^2)/b +(a+b)(a^2 +b^2 -c^2)/c}/(2abc).
 次に基本対称式 s=a+b+c, t=ab+bc+ca, u=abc とおくと s^3 -4st+9u≧0, t^2 -3su≧0, s^2 -3t≧0 [>>611]
 左辺 = {t(s^3 -4st+9u) +2s(t^2 -3su)}/(2u^2) + 3(s^2 -3t)/(2u) + t/u
 ≧ t/u = 1/a +1/b +1/c ≧ 右辺.
 3角形ぢゃなくても成り立ちそうな...
ぬるぽ
771768:04/12/09 21:59:58
出典はGreen Bookなんだが...
まあ、そう見えるか。
772132人目の素数さん:04/12/09 22:18:57
LCM(a,b)はa,bの最小公倍数を表すとする。

m>nなる自然数m,nに対し
LCM(m,n) + LCM(m+1,n+1)>2mn/√(m-n)
を示せ


若干趣旨がずれるかも知れないがネタ補給
773132人目の素数さん:04/12/09 22:28:45
>771
Green Book って何ですか?
検索したら830件もヒットしましたが…
http://www.amazon.co.jp/exec/obidos/search-handle-form/249-1225598-3680328
774132人目の素数さん:04/12/09 22:38:04
Green Book = 青チャート
775132人目の素数さん:04/12/09 23:06:11
>>773 Doverから出てる問題集があるんだよ。
776132人目の素数さん:04/12/09 23:10:48
>>775
ウホッ!不等式だけの問題集?
777132人目の素数さん:04/12/09 23:11:37
オラオラ!題名キボン!!
778132人目の素数さん:04/12/09 23:23:37
これですか?
不等式はどのくらい載ってるんですか?
問題のレベルはどれくらいですか?
http://www.amazon.co.jp/exec/obidos/ASIN/0486695735/qid=1102602093/sr=1-3/ref=sr_1_10_3/249-1225598-3680328
779132人目の素数さん:04/12/09 23:35:32
この数ヲタ! ___ オラッ !      
    ドッカン |   | でてこい、>>771
    ∩∩  |   |   |  ∩∩    
   | | | |  |   |   |  | | | |    
  ..(  ,,) .|   |   | (・x・ )  
  /  .つ━━ロ|ロ ドカン l   |U 
〜(   /   |   |   |⊂_ |〜
  し'∪   |   |   |   ∪ 
780771:04/12/10 00:27:12
>>778 そう、それ。普通の問題集だから、不等式の問題は殆ど載ってません。ごめんね。
"preparation for the Putnum" と書いてあるので、教養レベルくらいだと思うが、
高校生でも解けそうな問題もあります。
kalvaのページで紹介されていたので、けっこう有名な本なのかと思ったのだが。
781132人目の素数さん:04/12/10 01:04:48
>>752,>>760
ピタッと最小値を出してやろうとして、x-(1/4) =X とおいて
x^4-x^3+x^2-x = X^4 +(5/8)X^2 -(5/8)X - (51/256) = (t^2-a^2)^2+b(t-a)^2+c
を計算したら、aが3乗根を含む形になったので止めた…
782132人目の素数さん:04/12/10 02:14:30
【問題】 実数 x, y に対して 4 + x^2y^4 + x^4y^2 - 3x^2y^2 の最小値を求めよ。

ASU 1981 問22
http://www.kalva.demon.co.uk/soviet/sov81.html
783132人目の素数さん:04/12/10 06:24:16
【問題】 整数 a, b, c, d が a>b>c>d>0、ad=bc をみたすとき、(a-d)^2 ≧ 4d+8 を示せ。
784 ◆BhMath2chk :04/12/10 07:00:02
>>772
m−nはgcd(m,n)とgcd(m+1,n+1)の倍数。
gcd(m,n)とgcd(m+1,n+1)はそれぞれmとm+1の約数なので
互いに素だからm−nはgcd(m,n)gcd(m+1,n+1)の倍数。
gcd(m,n)gcd(m+1,n+1)≦m−n。
 lcm(m,n)+lcm(m+1,n+1)
=mn/gcd(m,n)+(m+1)(n+1)/gcd(m+1,n+1)
>mn/gcd(m,n)+mn/gcd(m+1,n+1)
≧2mn/√(gcd(m,n)gcd(m+1,n+1))
≧2mn/√(m−n)。
785132人目の素数さん:04/12/10 08:37:09
>782
 相加・相乗平均により、1 +x^2y^4 +x^4y^2 ≧ 3x^2y^2, 与式 ≧ 3, 等号成立は xy(x+y)(x-y)=0 のとき.
 高校生でも解けそうな問題....ぬるぽ
786785:04/12/10 08:40:45
 ...と思ったら間違えた。
 等号成立は x =±1, y =±1 のとき。ぬるぽ
787132人目の素数さん:04/12/10 10:53:51
>768
Ln のマクローリン展開 Ln(1-t) = -Σ[k=1,∞) (1/k)t^k より
 Ln{(1+t)/(1-t)} = Ln(1-t^2) -2・Ln(1-t) = Σ[k=0,∞) {2/(2k+1)}・t^(2k+1).
∴ (1/2t)Ln{(1+t)/(1-t)} = Σ[k=0,∞) {1/(2k+1)}・t^(2k)
 < (2/3) + (1/3)Σ[k=0,∞) t^(2k) = (2/3) + 1/{3(1-t^2)} < {1+(t^2)/3}/(1-t^4).
ここで (1+t)/(1-t)=x とおくと t=(x-1)/(x+1).
 {(x+1)/2(x-1)}Ln(x) < 2/3 + {(x+1)^2}/(12x) < {(x^2 +x+1)(x+1)^2}/{6(x^3 +x)}.
∴ Ln(x)/(x^3 -1) < (x+1)/{3(x^3 +x)}.

大関:「不等式への招待」p.46 例題6 (1987)
788132人目の素数さん:04/12/10 14:22:46
f(x)=(x+1)(x^3-1)/(x^3+x)-3log(x)
f'(x)=(x-1)^4(x^2+x+1)/(x^3+x)^2
f'(x)>0 & f(1)=0
789132人目の素数さん:04/12/10 19:03:36
>>785
等号は (x, y) = (1, 1), (1, -1), (-1, 1), (-1, -1) のとき成立ですな。
790785:04/12/10 20:08:06
>789
 仰せのとおり。dクス.

>788
 それなら高校の宿題に出てもおかしくない...

>781
 趣旨違うかも知れんが
 最小値 = 0.0016782234764・・・ > 0.00167410714285714 = 3/1792. [>>760 の下限]
 X = (1/2)[(5/18)^(1/3)]{(√6 +9/4)^(1/3) - (√6 -9/4)^(1/3)} = 0.35582958618827・・・
 x = X + 1/4 = 0.60582958618827・・・

ぬるぽ
791132人目の素数さん:04/12/10 20:18:22
ネタ補給

各辺の長さが整数値の三角形ABCがあり、∠A=2∠B ∠C>π/2を満たす。
この時、AB+BC+CA≧77を示せ。
792132人目の素数さん:04/12/10 21:14:13
m,nを正の整数とする。
Σ[ k^(m/(k^2)) ] < n+m( 2^(m/4)-1 )

[x] はガウス記号
793132人目の素数さん:04/12/11 17:10:30
age
794132人目の素数さん:04/12/11 20:08:12
>>793 上げんな、落ちこぼれ!
                            __________
                             .|<) <) <) <) <) |
  ―─=≡ ∧_∧     ガッ.          |<) <) <) <) <) |
 ──=≡ (,, ・∀・)      、_人.     .  |<)∧__∧ ) <) <) |
  ─=≡ ○_  と )__ _  _,) ,,;    =≡ ((    ), <) <) | ザクッ
   ──=≡ >       (´__) _) ∴; =≡≡ `)## . つ <) <) |
    ─=≡ ( / ̄ ̄ ̄    ) ''    =≡ ⊂ 、   ノ、) <) <) |
         ( ノ         ⌒Y´     .|<) `(_ノ<) <) <) |
                             .|<) <) <) <) <) |
795132人目の素数さん:04/12/11 21:30:03
>791
 等号成立は AB=33, BC=28, CA=16 のとき [cos(A)=17/32, cos(B)=7/8, cos(C)=-7/128<0]
 これだけぢゃ仕方ないが...

>792
 f(x)=Ln(x)/(x^2) とおくと f '(x)={1-2Ln(x)}/(x^3) ∴ x≧√e ≒1.6487212707 でf(x)は単調減少.
 k>2 ⇒ k^{m/(k^2)} < 2^(m/4)
 左辺は Σ[k=1,n] ・・・ と書いた方がいいYo.
ぬるぽ
796795:04/12/12 18:51:02
>792 (続き)
a_k = k^{m/(k^2)} とおく。  [795] より 1 = a_1 < a_k ≦ a_2 = 2^(m/4).
(i) m≦5 のときは計算実行する。 
 m≦3 のときは a_k ≦ a_2 < 2 ∴ 左辺 = n < 右辺 で成立。
 m=4,5 のときも a_2 <3, a_k ≦ a_3 < 2 (k>2). ∴ 左辺 = n+1 < 右辺 で成立。
(ii) m≧6 のとき、m > 5.77078016355585 = 4/Ln(2).
 a_1 = 1 < a_2 > a_k より Ln(a_k) < Ln(a_2) = m・Ln(k)/(k^2) = b.
 y=exp(x) は下に凸だから、0<x<b ⇒ exp(x) < 1 + {exp(b)-1}(x/b).
 ∴ a_k < 1 + (a_2 -1){Ln(a_k)/b}.
 下の補題を使って Σ[k=1,n] a_k < n + {4/Ln(2)}(a_2 -1)S_n < n + m(a_2 -1).
797795:04/12/12 18:52:26
>792 (続き)
【補題】S_n = Σ[k=2,n] Ln(k)/(k^2) <1.
(略証)S_n = S_5 + Σ[k=6,n) {Ln(k)-1}/(k^2) + Σ[k=6,n] 1/(k^2).
    < 0.44637574128158 + Σ[k=6,n) {Ln(k)/[(k-1)k] -1/(k^2)} + Σ[k=6,n] 1/(k^2)..
 ところで、y=Ln(x) は 上に凸だから、Ln(k) +1/k > Ln(k+1).
 (第2項) < Σ[k=6,n) {Ln(k)/[(k-1)k] -1/(k^2)} = Σ[k=6,n) {Ln(k)/(k-1) - [Ln(k)+1/k]/k}
 < Σ[k=6,n) {Ln(k)/(k-1) - Ln(k+1)/k} = Ln(6)/5 - Ln(n+1)/n ≒ 0.35835189384561 - Ln(n+1)/n.
 (第3項) < Σ[k=6,n] 1/(k^2) < (π^2)/6 -Σ[k=1,5] 1/(k^2) ≒ 0.18132295573712.
 ∴ S_n < 0.98605059086431 - Ln(n+1)/n <1. (終) ぬるぽ
798795:04/12/12 20:33:48
>792 (続き)
 S_n の上限値をピタッと出してやると S = Σ(k=2→∞) Ln(k)/(k^2) ≒ 0.91685416570・・・ に成増た。
ぬるぽ
799798:04/12/13 20:09:19
また間違えた。 S ≒ 0.9375482・・・ に訂正でつ。

      ∧_∧
     ( ´Д` )    まことに
     /     ヽ
     し、__X__,ノJ

      /´⌒⌒ヽ
    l⌒    ⌒l   すいませんでした。
   ⊂ (   ) ⊃
      V ̄V
800132人目の素数さん:04/12/14 01:52:03
>>368
おまいのようなやつがいるから、>>349 >>350のような話になるんだろうが。
                            __________
                            __________
                             .|<) <) <) <) <) |
  ―─=≡ ∧_∧     ガッ.          |<) <) <) <) <) |
 ──=≡ (,, ・∀・)      、_人.     .  |<)∧__∧ ) <) <) |
  ─=≡ ○_  と )__ _  _,) ,,;    =≡ ((    ), <) <) | ザクッ
   ──=≡ >       (´__) _) ∴; =≡≡ `)## . つ <) <) |
    ─=≡ ( / ̄ ̄ ̄    ) ''    =≡ ⊂ 、   ノ、) <) <) |
         ( ノ         ⌒Y´     .|<) `(_ノ<) <) <) |
                             .|<) <) <) <) <) |
801132人目の素数さん:04/12/14 01:52:26
ごめん、誤爆
802132人目の素数さん:04/12/14 03:40:27
雑魚の巡回ルートに入っているのかな?
定期的に意味不明なage荒らしがあるが…
氏ぬぇぇぇ〜
803132人目の素数さん:04/12/14 12:00:30
>797 の別法
 [795] より f(x) ≡ Ln(x)/(x^2), 原始函数はF(x)=-{1+Ln(x)}/x.
 次に S_n ≡Σ[k=1,n] f(k) ≦ ∫_[1,n+1/2] f(x)dx < ∫_[1,∞) f(x)dx
 = F(∞) - F(1) =1. を示す。

 f(2) ≒ 0.1732868 < 0.2334837 = F(5/2) - F(1) = ∫_[1,5/2] f(x)dx.

 k≧3 のときは f '(x) = [1-2Ln(x)]/(x^3), f "(x) = [6Ln(x)-5]/(x^4)
 ∴ x > exp(5/6) ≒ 2.301 ⇒ f(x)は下に凸。
 ∴ f(k)< ∫_[k-1/2, k+1/2] f(x)dx = F(k+1/2) - F(k-1/2).
804796:04/12/14 22:20:51
 [796]の下から4行目の右半分 Ln(a_k) < Ln(a_2) = m・Ln(2)/4 = b.
 死んでお詫びを...(AA省略)

 [799] S≒0.937548254325
805132人目の素数さん:04/12/15 15:24:38
>>704(2) の類題があるYo.
>>149(解答 >>157, >>162
806132人目の素数さん:04/12/15 15:27:16
>>805
Σ(゚Д゚ オーッ!! グッジョブ!
807132人目の素数さん:04/12/16 08:57:51
>>704(5)
 -a+b+c=x, a-b+c=y, a+b-c=z とおくと三角不等式が外れて、
 x,y,z≧0, x+y+z=a+b+c, a=(y+z)/2, b=(z+x)/2, c=(x+y)/2.
 ∴ (a^2)b +(b^2)c +(c^2)a = [(x+y+z)^3 -{(x^2)y +(y^2)z +(z^2)x}]/8 ≦ (1/8)(x+y+z)^3 = (1/8)(a+b+c)^3.
 等号成立は (x,y,z)=(1,0,0) & rotations, (a,b,c)=(0,1/2,1/2) & rotations のとき.

次に (4/27)(x+y+z)^3 -{(x^2)y +(y^2)z +(z^2)x} ≧0 を示す。
 x,y≧z≧0 としても一般性を失わない。x-z=X, y-z=Y とおくと X,Y≧0.
 (x+y+z)^3 = (X+Y+3z)^3 = (X+Y)^3 +27z{(1/3)(X+Y)^2 +(X+Y)z +z^2}.
 (x^2)y +(y^2)z +(z^2)x = (X^2)Y + 3z{(1/3)(X+Y)^2 +(X+Y)z +z^2}.
∴ (4/27)(x+y+z)^3 -{(x^2)y +(y^2)z +(z^2)x} ≧ (4/27)(X+Y)^3 -(X^2)Y = (1/27)(X-2Y)^2・(4X+Y) ≧0.
 等号成立は X-2Y=z=0, (x,y,z)=(2/3,1/3,0) & rotations, (a,b,c)=(1/6,1/3,1/2) & rotations のとき.

[704]の解答のレス番(主なもの)
  (1) 719 (2) 805 [>>149,>>157] (3) 714 (4) 736-737 (5) これ (6) 729 (7) 734
ぬるぽ
808807:04/12/16 13:02:11
>>704(8)
 左辺 = 3 +(b/a +a/b) +(c/b +b/c) +(a/c +c/a) = 9 +{(b-a)^2}/ab +{(c-b)^2}/bc +{(a-c)^2}/ca
 ≧ 9 + 3[{(b-a)^2 /ab}{(c-b)^2 /bc}{(a-c)^2 /ca}]^(1/3) = 9 + 3[(1- a/b)(1- b/c)(1- c/a)]^(2/3) ≡右辺.
ぬるぽ
809132人目の素数さん:04/12/17 20:59:17
△ABCの内心をO、内接円の半径をrとすると、
OA+OB+OC≧6r を示せ。
810132人目の素数さん:04/12/17 21:48:41
>807
乙でございます。なるほど、その置き換えだったんですね。

>808
なんだ、相加相乗だったのか…   il||li _| ̄|○ il||li

>809
簡単なのに難すぃ…
811132人目の素数さん:04/12/17 22:09:43
ネタ補充
(1) 1対1の上への関数 f : [0,1]→[0,1] は狭義単調増加であるとし、逆関数をgとおく。
  0<t<1のとき、∫[0,1] (f(x)+g(x))^t dx≧(2^t)/(1+t) を示せ。
(2) 平面上に5点A,B,C,D,Eがあるとき、AB+CD+DE+EC≦AC+AD+AE+BC+BD+BEを示せ。
(3) 非負実数係数のn次多項式f(x)のn次の係数と定数項は1であり、
  f(x)=0の解は全て実数とする。このとき、f(2)≧3^n を示せ。
(4) (2*4*6*...*100)/(1*3*5*...*99)>12 を示せ。
812132人目の素数さん:04/12/17 22:34:41
私もネタ補充。いつもと違う出題形式ですが…

【問題】 調和平均について、どのくらいの不等式を知ってますか?
このスレの不等式コレクションには2つしかありませんが…
a と b の調和平均を H(a, b) で表すことにすると、

[>>647(1)]  H(a+1, b+1, c+1) - H(a, b, c) ≧ 1

[>>647(6)]  H(a+b, c+d) ≧ H(a, b) + H(c, d)

下側の不等式は、繰り返し用いたら いくらでも長く出来そうですね。
他にもあったら教えてたも。
813132人目の素数さん:04/12/18 00:15:20
>>809
一般的に三角形ABCの内部に点Pを取り、Pから各頂点への距離の和をA
各辺への距離の和をBとおけば、A≧2Bが成立する。これはそれの特殊な場合かと。

2chでは
http://science3.2ch.net/test/read.cgi/math/1093676103/267
に解答が載ってる。

 ぬるぽ
814132人目の素数さん:04/12/18 19:57:31
>809,813
それぢゃあこっちには三角函数を使わない方法を載せておこう。

Pから3辺 BC, CA, AB に下した垂線を PD, PE, PF とする。
∠QAB=∠CAP であるような半直線AQを引き、B,CからAQに下ろした垂線をBM,CNとする。
2角相等により △ABM∽△APE ∴ PA・BM=c・PE
2角相等により △ACN∽△APF ∴ PA・CN=b・PF
∴ PA = (c・PE+b・PF)/(BM+CN).
ところが、半直線AMNと辺BCとの交点をQとすると, BM + CN ≦ BQ + QC = BC = a.
∴ PA ≧ (c/a)PE + (b/a)PF.
循環的に加えれば PA+PB+PC ≧ (c/b +b/c)PD +(a/c +c/a)PE +(b/a +a/b)PF ≧ 2(PD+PE+PF).
等号の成立は △ABCが正三角形で, Pがその中心 という場合だけ。 [東山和生氏による]

ほかにパッポスの定理や(外接円に関する)トレミーの定理を使う方法もあるらしい。

〔参考文献〕
「数学の問題 第(1)集」 No.53 日本評論社(1977) [数セミ,40(2) (2001,Feb.)で再出題]
N.D.Kazarinoff: "Geometric Inequalities", RandomHouse(1961) → 数理科学,1(4),サイエンス社(1963.10)
P.Erdosの問題: Amer.Math.Monthly (1935) → 大関:「不等式への招待」 p.14 近代科学社(1987.12)
815132人目の素数さん:04/12/19 02:03:39
a,b,cを正の有理数、(a^2)+(b^2)=(c^2)で(ab)/2は整数とする。
ab≧10を示せ
816132人目の素数さん:04/12/20 12:01:45
>811(4)
(4)
【補題】 [2*4*6*・・・*(2n)]/[1*3*5*・・・*(2n-1)] > √(3n+1).
(略証)
 (左辺)^2 = (2n+1)Π[k=1,n] {(2k)^2}/{(2k-1)(2k+1)}
  = (2n+1)Π[k=1,n] {1 +1/((2k-1)(2k+1))} > (2n+1){1 +Σ[k=1,n] 1/((2k-1)(2k+1)) }
  = (2n+1){1+(1/2)Σ[k=1,n] (1/(2k-1) -1/(2k+1)) } = (2n+1){1+(1/2)[1-1/(2n+1)]} = 3n+1.
 ∴ 与式 > √151 =12.288206

上記Σの初めの項を別扱いすれば少し増加して、
 左辺 > √{3n+1 +(n-1)/9} ≒ √156.4444 = 12.507775
 左辺 > √{3n+1 +(n-1)/9 +4(n-2)/225} = √157.29778 = 12.541841

なお、Stirling の公式 n!≒ n^(n+1/2)・exp(-n +1/(12n))・√(2π) から、
 (左辺) = {(2^n)(n!)}^2 /(2n)! = (4^n)/C[2n,n] ≒ √(πn)・exp(1/8n) = 12.564513
817816:04/12/20 17:55:23
>811(4)
 Stirlingの不等式 n^(n+1/2)・exp(-n)・√(2π) < n! < n^(n+1/2)・exp(-n +1/(12n))・√(2π) から、
 (左辺) = {(2^n)(n!)}^2 /(2n)! > √(πn)・exp(-1/24n) = 12.522701
ぬるぽ
818770:04/12/21 13:14:46
>>565(7), >>733
 r>0 とする。
 左辺 = (b+c)cosA/(a^r) + (c+a)cosB/(b^r) + (a+b)cosC/(c^r)
 = a{cosB/(b^r) +cosC/(c^r)} +b{cosC/(c^r) +cosA/(a^r)} +c{cosA/(a^r) +cosB/(b^r)}
 a,b,cの大小とA,B,Cの大小は一致するから
左辺 ≧ a{cosB/(c^r) +cosC/(b^r)} +b{cosC/(a^r) +cosA/(c^r)} +c{cosA/(b^r) +cosB/(a^r)}
 = (b・cosC+c・cosA)/(a^r) + (c・cosA+acosB)/(c^r) + (a・cosB+b・cosA)/(c^r)
 第一余弦定理より
 左辺 ≧ 1/a^(r-1) + 1/b^(r-1) + 1/c^(r-1).
819 ◆BhMath2chk :04/12/22 22:00:01
>>812
H(H(a,b),H(c,d))=H(a,b,c,d)。
H(a,b,c,H(a,b,c))=H(a,b,c)。
H(ab,ac,ad)=aH(b,c,d)。
H(1,1,1)=1。
a≦c,b≦dのときH(a,b)≦H(c,d)。
(1/4)H(a,b,c,d)≦(1/3)H(a,b,c)。
lim_{d−>+∞}H(a,b,c,d)=(4/3)H(a,b,c)。
H(a,b)+H(c,d)≦H(a+c,b+d)。
H(a,b,c)+H(d,e,f)≦H(a+d,b+e,c+f)。
H(a,b,c,d)+H(e,f,g,h)≦H(a+e,b+f,c+g,d+h)。
820132人目の素数さん:04/12/23 06:42:41
>819 調和平均 (;´д`)ハァハァ
   ___
 ./  ≧ \
 |::::  \ ./ |
 |::::: (● (● | みんな グッジョブ!
 ヽ::::... .ワ....ノ    n  
 ̄ ̄   \    ( E)
フ     /ヽ ヽ_//
821132人目の素数さん:04/12/24 03:13:38
Green Book = 青チャート

822132人目の素数さん:04/12/24 03:14:29
真似したり、関係の無い事言ったり、無茶苦茶書くな 荒らしは

 〜〜〜終了〜〜〜
 
ageるな馬鹿タレ
823言い切れる???:04/12/24 03:43:11
−∞/∞は負って言えますか???
教えてください!!!
824132人目の素数さん:04/12/24 03:44:04
真似したり、関係の無い事言ったり、適当な事書いたり、無茶苦茶書くな 

荒らしは
 〜〜〜終了〜〜〜
 
ageるな馬鹿タレ

お前が数学出来ないのはわかるが八つ当たりするな
825132人目の素数さん:04/12/24 07:55:18
>811(3)
 非負実係数だから x≧0 ⇒ f(x)>0. ∴ 実根はすべて負である。
 これらを -a_1, ・・・ -a_n とおくと、 f(x) = Π[i=1,n] (x+a_i) = Σ[k=1,n] S_{n-k} x^k.
 x^k の係数 S_{n-k} は{a_1,・・・a_n}の n-k次 の基本対称式。
 相加・相乗平均により S_k ≧ C[n,k]・g^k, ここに g=(S_n)^(1/n)=(定数項)^(1/n).
 f(x) ≧ Σ[k=1,n] C[n,n-k] g^(n-k) x^k = (x+g)^n.
826825:04/12/24 18:02:26
↑最下行を修正、すまそ。 x≧0 ⇒ f(x) ≧ ・・・・・ = (x+g)^n.

(1)(2) はむずぽ.....5点が平面上にないと無理でつか?
827132人目の素数さん:04/12/24 22:54:10
811(2)は、Kurschak1981ですね。
ttp://www.kalva.demon.co.uk/eotvos/kur1981.html
828132人目の素数さん:04/12/24 22:56:57
普通に考えると凸包で場合わけかなぁ、3通りあるけど
果たしてこれでいいものか。図を描いてみないと分からんな……
829132人目の素数さん:04/12/24 23:01:02
すまん、828は忘れてくれ。ムリポ……
830 ◆BhMath2chk :04/12/25 08:00:00
831132人目の素数さん:04/12/27 07:06:14
【問題】 正の数 a, b, c に対して a/b + b/c + c/a ≧ (a+b+c)/\sqrt[3]{abc}
発掘元 B3780 http://www.komal.hu/verseny/2004-12/mat.e.shtml

>>647(2)、解答>>680
 [1997 Belarus] a/b + b/c + c/a ≧ (b+c)/(a+b) + (c+a)/(b+c) + (a+b)/(c+a)

ところで、Cauchy-schwarz などから
 a/b + b/c + c/a ≧ (a+b+c)^2/(ab+bc+ca) ≧ (b+c)/(a+b) + (c+a)/(b+c) + (a+b)/(c+a)
が成り立つのが分かりますが、この不等式のどこかに
 (a+b+c)/\sqrt[3]{abc}
が収まるでしょうか?
832132人目の素数さん:04/12/27 11:44:30
>831
 相加・相乗平均: (1/3)(a/b + a/b + b/c) ≧ {(a^2)/bc}^(1/3) = a/[(abc)^(1/3)] を循環的にたす。
 どこにも収まらな伊予柑。
833132人目の素数さん:04/12/28 13:26:07
>>699
ageるとすぐ回答が出てくる一例
834132人目の素数さん:04/12/28 13:35:20
>>833
ageることに理由をつけたがる一例
835132人目の素数さん:04/12/28 14:29:39
晒しあげとくか
836132人目の素数さん:04/12/29 05:35:30
>614 名前:132人目の素数さん[] 投稿日:04/12/28(火) 21:02:34
>
>0より大きい相異なる3実数x、y、zに対し
>(x^3+y^3+z^3-3xyz)/|(x-y)(y-z)(z-x)|
>の最小値を求めよ。
>
>
>618 名前:132人目の素数さん[sage] 投稿日:04/12/28(火) 22:30:23
>>>614
>数学セミナーエレガントな問題を求むのコーナーに寄せられた
>問題だな。かなーーり昔だったと思う。
>
>解法は与式を二乗して、基本対称式使って書き換えて
>解と係数の関係を使って愚直にやっていくというもの。
>
>答えはきれいな形にならず、三乗根が入ってきたはずだ。


東大入試問題スレより。詳細キボン
837132人目の素数さん:04/12/29 05:51:14
検索では掛かりませんねぇ
838 ◆BhMath2chk :04/12/29 09:00:00
 x^3+y^3+z^3−3xyz
=(x+y+z)((x−y)^2+(y−z)^2+(z−x)^2)/2。
x,y,zの差を変えずにx+y+zを小さくできるので最小値はない。
839132人目の素数さん:04/12/29 09:29:56
840132人目の素数さん:04/12/29 11:31:03
>>839
dクス。
年明けが待ち遠しいぜ!
今すぐ図書館の窓割って探したいくらいだ!
841132人目の素数さん:04/12/29 11:42:54
1: 【社会】「解法が知りたくて…」 図書館の窓を割り潜入した数ヲタ逮捕 … 埼玉 (132)

「図書館が開くまで待てなかった」 ・・・ 大学図書館の窓を割って進入し、
書庫の雑誌を読み漁っていた数ヲタ(22)を、29日夕方、公共物破損および
不法侵入の罪で逮捕した。

調べでは、棒掲示板に書込みされていた数学の問題の解法を調べるために
忍び込んだという。逮捕の際には、「邪魔をするな。読み終わるまで待ってくれ」
と自分の行いに全く反省をしていなかったと言う。
842132人目の素数さん:04/12/29 12:10:05
実数 x, y が 0≦x≦2、0≦x-y≦2 をみたすとき、xy(y+1) の最大最小値。
843132人目の素数さん:04/12/29 15:28:07
むしゃくしゃしてやった。
悪気はなかった。
いまは反省している。
844132人目の素数さん:04/12/29 16:43:08
>>842
学コン
845132人目の素数さん:04/12/29 17:43:30
>836
 z≒0 の場合を考えるんだろうな。
 与式 = (x^3 +y^3)/|xy(x-y)| ≧ c, 但し c≡(1+√3)・√{(3/2)√3} = 4.4036694750416・・・・・
 (略証) (x^3 +y^3)^2 - [cxy(x-y)]^2 = [x^2 -(1+√3)xy +y^2]^2 [x^2 +2(1+√3)xy +y^2] ≧0.
   等号成立は [x^2 -(1+√3)xy +y^2]=0 すなわち x/y ={(1+√3)±√(2√3)}/2 のとき.
 となるはずの所だが、本題は z>0 なので等号は成田たね.

>842
 スレ違いかも知れんが、
 内部には極値は無さそうだから、平行四辺形の周を調べるんだろうな。
 (x,y)=(1+√(1/3), -1+√(1/3)) で最小値 -2/(3√3) = -0.38490017945975・・・・
 (x,y)=(2,2) で最大値 12.
ぬるぽ
846132人目の素数さん:04/12/29 17:53:14
z≒0の時っていうのが分からんorz

それに上の結果と一致しないし、三乗根入ってくると
解答者は言っているのだが……それが正しいかさえ分からん。




救済キボンヌ
847845:04/12/29 18:38:04
>846
 x,y>z>0 としても一般性を失わない。[838]から F(x,y,z) > F(x-z,y-z,0).
 となるから、F(X,Y,0) の下限を調べるんだろうな....
848132人目の素数さん:04/12/29 20:55:53
このスレの獣人ならもちろんシュプリンクラーの不当式勝ったよな。
849132人目の素数さん:04/12/29 21:08:36
>>847
thx 当たり前のことっすね。。。orz
850132人目の素数さん:04/12/29 21:46:16
>>848
当然だ。何をいまさら!
チンコ洗って出直してこいッ!
851132人目の素数さん:04/12/29 22:56:45
>>850
女子はどうすればよいでつか?
852132人目の素数さん:04/12/30 10:59:07
図書館潜入、模範解答報告マダァ?
853132人目の素数さん:04/12/30 18:09:27
>851
メチンコ洗って出直してこいッ!
なんなら洗ってやろうか?
854132人目の素数さん:04/12/31 12:07:52
おらおら、まだか?
855132人目の素数さん:05/01/04 13:25:45
0<1
856132人目の素数さん:05/01/04 14:36:20
    |┃三 ./  ≧ \  | あけおめ、ことよろ!
    |┃  |::::  \ ./   | 今年も たのも〜!
    |┃ ≡|::::: (● (●  <_________
______.|.ミ\ ヽ::::... .ワ......ノ_ /ミ !____
    |┃=__    __=┃|
    |┃ ≡ )   (  ≡ ┃|
    |┃  / /\ \  ┃| ガラッ
857132人目の素数さん:05/01/04 22:46:12
>>836
与式を F(x,y,z) とおいてまとめると・・・
[838]
F(x,y,z) = (x+y+z)・g(x-y,y-z,z-x), g>0.
[847]
 x,y>z>0 としても一般性を失わない。[838]から F(x,y,z) > F(x-z,y-z,0).
 となるから、下限 F(X,Y,0) の下限を調べるんだろう。
[845]
 F(x,y,0) = (x^3 +y^3)/|xy(x-y)| ≧ c, 但し c≡(1+√3)・√{(3/2)√3} = 4.4036694750416・・・・・
 (略証) (x^3 +y^3)^2 - [cxy(x-y)]^2 = [x^2 -(1+√3)xy +y^2]^2 [x^2 +2(1+√3)xy +y^2] ≧0.
みたいになるかな?

注)例によって c の発見には微分法を使いますたが、代数的方法を示していまつ...
858132人目の素数さん:05/01/05 14:50:09
正の数 x, y, z に対して、次式を示せ。
 (xy+yz+zx){1/(x+y)^2 + 1/(y+z)^2 + 1/(z+x)^2} ≧ 9/4

(x+y)^2 +(y+z)^2 + (z+x)^2 ≧ 4(xy+yz+zx) なので
Cauchy-Schwarz は使えなかったので、対称式を持ち出した。

x+y+z=s, xy+yz+zx=t, xyz=u とおくと、示すべき不等式は
 4t(s^4-2s^2t+4su+t^2) ≧ 9(st-u)^2
差を取って証明しようとしたけど、うまくいかんちん。
もしかして計算間違っていますか? たのも〜
859132人目の素数さん:05/01/06 02:50:34
非負実数 a, b に対して、次を示せ。
 4(a^9+b^9) ≧ (a^2+b^2)(a^3+b^3)(a^4+b^4)
860132人目の素数さん:05/01/06 08:48:25
>859
 a≧0, b≧0, mn>0 のとき、
2[a^(m+n) + b^(m+n)] - (a^m +b^m)(a^n +b^n) = (a^m -b^m)(a^n -b^n) ≧0.
ぬるぽ
861132人目の素数さん:05/01/06 11:43:53
>>860
キター *・゜゚・*:.。..。.:*・゜(゚∀゚)゚・*:.。..。.:*・゜゚・* !!!!!
なるほどっ!
dクスでごじゃる。
問題は、要するにチェビシェフを2回使ったんですね、ナットク。

発掘元はココ
http://www.math.pomona.edu/ps4_04.pdf
862132人目の素数さん:05/01/06 17:15:13
>>807
一般に a+b+c=Lとおけば、(23/216)L^3 ≦ a^2b+b^2c+c^2a ≦ (1/8)L^3 でいいんですよね?
863132人目の素数さん:05/01/06 17:16:50
>>858
基本対称式を使わずに、x, y, z のままで考えたほうがええかもな
864807:05/01/06 20:55:53
>862 桶。
x+y+z = a+b+c = L を使えば [807] は
 与式 = [L^3 -{(x^2)y+(y^2)z+(z^2)x}]/8,
 (4/27)L^3 ≧ (x^2)y +(y^2)z +(z^2)x ≧0.
下の式を上の式に代入すると
 (1- 4/27)(1/8)L^3 ≦ 与式 ≦ (1/8)L^3.
ぬるぽ
865860:05/01/06 21:21:41
>861
>>153 (a=0,m=2の場合)や >>685-688 のあたりにも類題....
866132人目の素数さん:05/01/06 22:01:59
>836
[839] や 東大入試作問者スレ[618][621] が指摘したように、この問題は
 創刊30周年記念『エレガントな問題をもとむ』【優秀賞】受賞問題2
と同じようでつ。(作問者は西宮の人)
 問題: 数セミ,31(4), p.79 (1992.4)
 解答: 数セミ, 31(8), p.59-60 (1992.7) ・・・ 基本対称式を持ち出してもしかた無いんぢゃ・・・以下(ry


東大入試作問者になったつもりのスレ4
http://science3.2ch.net/test/read.cgi/math/1099493043/614,618-621,624-629,636,638-639

さくらスレ154
http://science3.2ch.net/test/read.cgi/math/1103204972/417

不等式スレッド
http://science3.2ch.net/test/read.cgi/math/1072510082/836-849,857
867132人目の素数さん:05/01/06 22:11:59
>865 dクスです。
868132人目の素数さん:05/01/06 22:12:21
>>836-839>>845-847>>857
相異なる正の数 x、y、z に対し、(x^3+y^3+z^3-3xyz)/|(x-y)(y-z)(z-x)| > √(9+6√3)

1992年の数セミを見てきました。編集部の模範解答は>>836の如し。
読者2名からのエレガントな解答を以下に書きます。

分子は x^3+y^3+z^3-3xyz = -(x+y+z){(x-y)(y-z)+(y-z)(z-x)+(z-x)(x-y)} だから
 L=x+y+z, p=x/L, q=y/L, r=z/L
とおくと、p, q, r>0 かつ p+q+r=1 をみたし、示すべき不等式は
 -{(p-q)(q-r)+(q-r)(r-p)+(r-p)(p-q)}/|(p-q)(q-r)(r-p)| > √(9+6√3)
式の対称性から、p>q>r>0 としても一般性を失わない。
 p-q=α, q-r=β, α+2β=s
とおくと s+3r=1 より 0<s<1 で、示すべき不等式の左辺は
 1/(p-q) + 1/(q-r) + 1/(r-p) = 1/α + 2/(s-α) - 2/(s+α)
さらに α/s = t とおくと、s>α>0 より 0<t<1 で、結局、次式の最小値を考えればよい。
 (1/s){1/t + 2/(1-t) + 2/(1+t)}   … (☆)
f(t) = 1/t + 2/(1-t) + 2/(1+t) とおくと f'(t) = (3t^4+6t^2-1)/{t^2(1-t)^2(1+t)^2} より
0<t<1 において t = √{(2√3-3)/3} で極小かつ最小値 √(9+6√3) をとる。
0<s<1 より、s→1 の極限をとれば、(☆) は √(9+6√3) に上から限りなく近づく。
(蛇足) 2重根号を外したら、√(9+6√3) = (1+√3)√{(3√3)/2}
   ___
 ./  ≧ \ 
 |::::  \ ./ | 勉強になりました。
 |::::: (● (● |  
 ヽ::::... .ワ....ノ    n   グッジョブ!
 ̄ ̄   \    ( E)
フ     /ヽ ヽ_//
869132人目の素数さん:05/01/06 22:13:36
>>866
あ、書き込みに気づかずに、先に解答書いてしまいました。スマソ。
870132人目の素数さん:05/01/06 22:37:07
>>811(2) ,827
 第10回 日本数オリ本選(2000)の第3問らしい...
(略解)
 case1: {AC,AD,AE} のいづれかと {BC,BD,BE} のいづれかが(端点以外で)交わる場合
  たとえば 線分ACとBDが交わるときは、 AB+CD ≦ AC+BD.
  これに自明な DE ≦ AD+AE, EC ≦ BE+BC を加える。

 case2: {AC,AD,AE,BC,BD,BE} のいづれも(端点以外は)交わらない場合
  A,Bの一方は△CDEの1つの頂角の対頂角の中にある(平面CDE内にあることから)。
  たとえば A が∠CDE の対頂角∠XDYの中にあるときは、△CAE ⊇ △CDE ∴ CD+DE ≦ AC+AE.
  これに自明な EC ≦ BE+BC, AB ≦ AD+BD を加える。
ぬるぽ
871132人目の素数さん:05/01/09 07:05:47
ネタ補充
(1) 実数x,y,z, 正の偶数nに対し
 (x+y+z){x^(n+1)+y^(n+1)+z^(n+1)}
 ≧ 2(xy+yz+zx)(x^n+y^n+z^n)-3xyz{x^(n-1)+y^(n-1)+z^(n-1)} を示せ。
(2) F(n)をFibonacci数列とする。
 Σ[n=1..∞] 1/F(n) > 803/240 を示せ。
(3) 2^(3^(3^(-1))) < e を示せ。
(4) 三角形の周長をL, 面積をSとすると
 S≦(L^2)/(12√3) を示せ。
(5) 0<t≦1 のとき
 tan(t)/t < 2-√(1-t^2) を示せ。
872132人目の素数さん:05/01/09 11:16:07
>>783
これもよろしくお願いします。
873132人目の素数さん:05/01/09 18:19:09
(2) Σ[n=1,6] 1/F_n = 1 +1 +1/2 +1/3 +1/5 +1/8 = 758/240.
 左辺 > Σ[n=1,12] 1/F_n = 758/240 +1/13 +1/21 +1/34 +1/55 +1/89 +1/144
= 758/240 + 207530077/N > 758/240 +204459255/N = 758/240 + 9/48 = 右辺.
 ここに N=16*9*5*7*11*13*17*89 = 1090449360.
ぬるぽ
874132人目の素数さん:05/01/10 15:05:49
>871 (1)
 n は整数とし、 左辺 - 右辺 = F_n とおく。 
 x^n の係数は (x+y+z)x - 2(xy+yz+zx) +3yz = (x-y)(x-z) なので >>611 の方法を使う。
y は x,z の間にあるとすると (x-y)(y-z)≧0, {x^n -y^n +z^n}≧0 ゆえ、
 F_n = (x-y)(x-z)x^n +(y-x)(y-z)y^n +(z-x)(z-y)z^n = (x-y)^2(x^n) +(y-z)^2(z^n) +(x-y)(y-z){x^n -y^n +z^n} ≧0.
F_{-1} = (t^2 -3su)/u, F_0=s^2 -3t, F_1=s^3 -4st +9u, F_2=s^4 -5ts^2 +4t^2 +6su, ・・・・・.

 (応用例) >>858
 4t(s^4 -2s^2t +4su +t^2) -9(st-u)^2 = 4ts^4 -17(st)^2 +34stu +4t^3 -9u^2
 = 4t・F_2 +{(3t^2 -su)/s}・F_1 +(s +9t/s)u・F_0 ≧0.
ぬるぽ

 [873] は >871(2) の解答でつ。スマソ
875132人目の素数さん:05/01/10 18:21:22
>>874
Scherの不等式と同じ証明方法になるとは…。
(;´д`)ハァハァ
結局、任意の整数で成り立つんですね。
F_n に n=1, 2, … を代入して得られる 対称式≧0 は 858 にも使えそうですね。
876132人目の素数さん:05/01/10 18:26:56
>>874
よくみれば、問題文は 実数 x, y, z だから偶数 n でないといけないですね。
非負実数 x, y, z なら、任意の整数 n でOKですね。
877876:05/01/10 18:44:12
>874の後半に、ちゃんと書いてるし…
  il||li _| ̄|○ il||li
878132人目の素数さん:05/01/10 19:03:28
このスレを見ていると、ときどき面白いパターンがあるよね。

ある不等式Aが解決されていない
→ 誰かが別の不等式Bを出す
→ 神が解く
→ その不等式Bが、Aを解決する

858 と 871(1)、 538(3) と 580 など…
879132人目の素数さん:05/01/10 22:53:27
複素数 z, w に対して、|(1+|z|^2)w-(1+|w|^2)z| ≧ |z\bar{w}-\bar{z}w| を示せ。
880132人目の素数さん:05/01/11 00:03:18
>>871
(4) 周長の長さがLの三角形の面積の最大値maxSを求めればよい。
まず、一辺の長さがすでに決まっている、という条件をつけて一辺を固定すれば、
残りの頂点の軌跡は楕円になるから一辺の長さがyであるときの、
Sの最大値M(y)が求まる。次にyの定義域を[0,L/2]としてmaxM(y)=maxSを求めればよい。
881132人目の素数さん:05/01/11 09:25:40
>871(4) (等周問題)
 [880] にしたがって BC=y を固定して考える。
 AB+AC = L-y (一定) の軌跡は楕円で、B,C は長軸上にある。
 S=(1/2)BC・AH が最大値をとすとすれば、Aの高さ(AH)が最大のとき。∴Aが短軸の端点のとき。・・・・ (1)
 ∴M(y)=(y/4)√{L(L-2y)}, M(L/3)^2 -M(y)^2 = (L/16)・(y -L/3)^2・(2y +L/3) ≧0, M(y)≦M(L/3)=(L^2)/(12√3).
 または (1)から AB=AC. 同様にして AC=BC, AB=BC なので、正3角形のとき最大.

 【代数的な方法】
 3辺をa,b,cとし、a+b+c=L, ab+bc+ca=t, abc=u とおくと、ヘロンの公式や相加・相乗平均から、
 S^2 = (L/2)(L/2 -a)(L/2 -b)(L/2 -c) = (L/16)(-L^3 +4Lt-8u) ≦ (L/16)u ≦ (L/16)(L/3)^3.
∴ S ≦ (L^2)/(12√3), 等号成立は a=b=c (正3角形)のとき.
 (系) S=rL/2 (rは内接円の半径)だから S≧(3√3)r^2.
ぬるぽ

>876 グッジョブ!
882132人目の素数さん:05/01/11 15:24:28
正の数 a, b に対して a^6 +b^6 +8a^3 +8b^3 +2a^3b^3 +16 ≧ 36ab を示せ。
883132人目の素数さん:05/01/11 15:49:14
>882 相加・相乗平均で
 左辺 = (a^3 +b^3 +4)^2 = (a^3 +b^3 +1 +1 +1 +1)^2 ≧ {6√(ab)}^2 = 36ab =右辺.
884132人目の素数さん:05/01/11 17:50:53
>>871 (5)
 まづ 1-t^2 < 1-t^2 +(1/8)(t^6){1+(1/8)t^2} = [1-(1/2)t^2 -(1/8)t^4]^2.
 ∴ 右辺 = 2 -√(1-t^2) > 2 - [1 -(1/2)t^2 -(1/8)t^4].
 ∴ tan(t)/t < 2 -[1 -(1/2)t^2 -(1/8)t^4] が示せればよい。
 F(t) = {2 -[1 -(1/2)t^2 -(1/8)t^4]}t - tan(t) とおくと F(0)=0. 次に F '(t)>0 を示す。  
 F '(t) = 1 + (3/2)t^2 + (5/8)t^4 - 1/[cos(t)^2] = {[1 +(3/2)t^2 +(5/8)t^4]cos(t)^2 -1}/[cos(t)^2]
 ここで t>0 ⇒ -cos(t) > -1, -sin(t) > -t, cos(t) > 1 -(1/2)t^2 を使って,
 F '(t) > {[1+(3/2)t^2 +(5/8)t^4][1-(1/2)t^2]-1}/[cos(t)^2] ={1-(1/8)t^2 -(5/16)t^4}(t^2)/[cos(t)^2].
 ここで、t^2 <1 だから F '(t)>0.
 ∴ tan(t)/t < 2 -[1 -(1/2)t^2 -(1/8)t^4] < 2-√(1-t^2).
ぬるぽ

>>871 への解答レス(主なもの)
 (1) [874][876] (2) [873] (3)参った. (4) [880][881] (5) これ.
885132人目の素数さん:05/01/11 21:34:18
(log(2))^3<1/3 なら、間にある(有理数)^3を見つければ
示せそうな気がするが...
886132人目の素数さん:05/01/12 00:46:12
>>885
あのな、(a^m)^n = a^(mn) だよな。
これが a^(m^n) だと〜、a^(m^n) だと〜 (以下略)
887132人目の素数さん:05/01/12 01:03:51
>>886
????????
888132人目の素数さん:05/01/12 01:05:56
>>886-887
おまいら、荒らすな上げるな!
889132人目の素数さん:05/01/12 01:37:55
>885
 有理数としてたとえば 0.69333333・・・・= 36/100 + 1/3 = 52/75 をとる。
 右側は (52/75)^3 = 140608/4215 < 140625/421875 = 1/3.
 左側 Ln(2) < 52/75 の方をどうするか……
890132人目の素数さん:05/01/12 03:01:14
[0,1/3]上で
1/(1-x^2) = 1 + x^2 + x^4/(1-x^2) ≦ 1 + x^2 + x^4/(1-(1/3)^2) = 1 + x^2 + 9x^4/8

よって、(log2)/2 = ∫_[0,1/3](1/(1-x^2))dx < ∫_[0,1/3](1 + x^2 + 9x^4/8)dx = 1123/3240

3乗の計算が面倒だけど、(log2)^3 < (1123/1620)^3 = 1416247867/4251528000 < 1/3
891132人目の素数さん:05/01/12 03:19:35
そういえば数セミ 2005 年 2 月号の『NOTE/講評と解説』に
巡回不等式とか正弦関数の比の単調減少性とか今後使えそうな面白い記事があったね。
892132人目の素数さん:05/01/12 03:34:04
>>891
2005年2月号って、まだ発売されてないだろ!

>>783>>879
解答マダー!
おらおら出てこい!不等式オタクども!
893132人目の素数さん:05/01/12 03:50:36
>>892
年間購読者ですから, もう届いているんですよ。
894132人目の素数さん:05/01/12 04:02:05
>>893
さっさとウプシロン!
895中川 幸一 ◆MC1Z7pcz5k :05/01/12 04:13:10
じゃ〜問題だけ紹介します。

a_1, \cdots a_n > 0 に対して不等式
\dfrac{a_1^2}{a_2 a_3} + \dfrac{a_2^2}{a_3 a_4} + \cdots + \dfrac{a_{n-1}^2}{a_n a_1} + \dfrac{a_n^2}{a_1 a_2}
\geq \dfrac{2 a_1}{a_2 + a_3} + \dfrac{2 a_2}{a_3 + a_4} + \cdots + \dfrac{2 a_{n-1}}{a_n + a_1} + \dfrac{2 a_n}{a_1 + a_2}
が成立する。ここで, 等号は a_1 = \cdots =a_n のときのみ成立する。

\pi \geq A > B > 0 のとき関数
\dfrac{\sin (Ax)}{\sin (Bx)}
は 0 < x < \dfrac{1}{2} において単調減少である。
896132人目の素数さん:05/01/12 04:15:14
>>895
せめて \frac と書け!
いちいち上げんな!
897中川 幸一 ◆MC1Z7pcz5k :05/01/12 04:18:54
以下は演習問題という形で紹介されています。
(表記法を多少簡略化しました。)

a_1, \cdots , a_n が 1, \cdots , n の並べかえならば
\sum_{i=1}^n \dfrac{a_i}{i} \geq n
898132人目の素数さん:05/01/12 04:20:40
>>896
悪い…。
ついつい癖で \dfrac って書いてしまった…。
あと上げてしまってスマン…。
899884:05/01/12 17:50:14
>885,890
 871(3) なるほど! そういうことだったのか。 ベリィdクス.

 871(5) F(t)>0 を出すところに間違い・・・・・ 死んでお詫びを・・・(AA省略)。
 マクローリン展開より、0<t≦1 で、
 tan(t) = t + (1/3)t^3 +(2/15)t^5 +(17/315)t^7 +・・・ ≦ t + (1/3)t^3 +(2/15 +17/315 +・・・)t^5 = t +(1/3)t^3 +c・t^5.
 ここに c = tan(1) -1 -1/3 = 0.22407439・・・
 F(t) = {2 -[1-(1/2)t^2 -(1/8)t^4]}t -tan(t) ≧ (1/6)t^3 +(1/8 -c)t^5 = (1/6 -0.09907439t^2)t^3 >0.
900伊丹公理 ◆EniJeTU7ko :05/01/12 18:10:07
>>897
容易 ( n > 1 の時)
一般に正とは限らぬ実数列 a_1 < a_2 < ....... < a_n,
b_1 < b_2 < ....... < b_n があり、c1, c_2, .... , c_n が
b_1, b_2, ....... , b_n の並べ替えであるとき、
a_1*c_1 + a_2*c_2 + .... + a_n*c_n が最大となるのは
c_i = b_i, i = 1, 2, .... , n の時。(又その時に限る)。

なる事実(well-known)と
x + (1/x) ≧ 2, 等号は x = 1 の時なることより出る。
901伊丹公理 ◆EniJeTU7ko :05/01/12 19:13:47
>>900
訂正 x > 0 の時、
x + (1/x) ≧ 2, 等号は x = 1 の時なることより出る。
902中川 幸一 ◆MC1Z7pcz5k :05/01/12 21:01:59
>>897 の解答 (Original)

相加平均と相乗平均の大小関係より
(1/n)Σ[i=1 to n](a_i / i) ≧ (Π[i=1 to n](a_i / i))^(1/n) = 1^(1/n) = 1
故に
Σ[i=1 to n](a_i / i) ≧ n
(等号成立は a_i = i)
903132人目の素数さん:05/01/12 23:24:58
(1) 正の実数a,b,x,y,zに対し、x/(ay+bz) + y/(az+bx) + z/(ax+by)≧3/(a+b)
(2) 実数a,b,c,x,y,zに対し、
 ax+by+cz+√{(a^2+b^2+c^2)(x^2+y^2+z^2)}≧(2/3)(a+b+c)(x+y+z)
(3) 0<x≦π/2 のとき、sin(sin(x))<tanh(x)
(4) 正の実数x,y,zがx+y+z=xyzを満たすとき、
 x/√(1+x^2)+y/√(1+y^2)+z/√(1+z^2)≦(3√3)/2
904132人目の素数さん:05/01/12 23:30:54
まだ解かれていない問題をまとめてください。
905132人目の素数さん:05/01/13 04:34:03
>>819
>H(a,b)+H(c,d)≦H(a+c,b+d)。
>H(a,b,c)+H(d,e,f)≦H(a+d,b+e,c+f)。
>H(a,b,c,d)+H(e,f,g,h)≦H(a+e,b+f,c+g,d+h)。

これって n文字についても言えそうな感じだけど、うまい証明が思いつきません。
いい方法ないですか?


>>812
>[>>647(1)]  H(a+1, b+1, c+1) - H(a, b, c) ≧ 1

こちらは n文字についても言えるのでしょうか?
たのも〜、たのも〜。
906132人目の素数さん:05/01/13 04:39:47
>>879
(左辺)^2-(右辺)^2 = |(z-w)(\bar{z}w-1)|^2 ≧ 0

          ___    
    |┃三 ./  ≧ \  >>892 呼んだ?
    |┃   |::::  \ ./ | 
    |┃ ≡|::::: (● (● |  不等式と聞ゐちゃぁ
____.|ミ\_ヽ::::... .ワ......ノ     黙っちゃゐられねゑ…
    |┃=__    \           ハァハァ
    |┃ ≡ )  人 \ ガラッ
907132人目の素数さん:05/01/13 11:59:33
>903
(1) x/(ay+bz) = {(a^2)(ax+by)/(ay+bz) +(b^2)(az+bx)/(ay+bz) -ab}/(a^3 +b^3).
 循環的に加えて 相加・相乗平均を使うと
 左辺 ≧ 3(a^2 +b^2 -ab)/(a^3 +b^3) = 3/(a+b), 等号成立はx=y=zのとき.

(2) (a,b,c)=OA↑, (x,y,z)=OX↑, (1/√3, 1/√3, 1/√3)=OE↑ とおくと ∠AOX ≦ ∠AOE +∠EOX.
 左辺 = (OA↑・OX↑) + |OA|・|OX| = |OA|・|OX|{cos(∠AOX) +1} ≧ |OA||OX|{cos(∠AOE+∠EOX) +cos(∠AOE-∠EOX)}
 = 2|OA|・|OX|cos(∠AOE)・cos(∠EOX) = 2(OA↑・OE↑)(OE↑・OX↑) = 右辺.

(4) x=tan(A), y=tan(B), z=tan(C), 0<A,B,C<π/2 とおく。条件式より A+B+C=π. sin()は上に凸だから,
 左辺 = sin(A) + sin(B) + sin(C) ≦ 3sin{(A+B+C)/3} = 3sin(π/3) =(3√3)/2.
 等号成立は A=B=C=π/3 (正3角形?) すなわち x=y=z=√3 のとき.
ぬるぽ

>906 グッジョブ!
908132人目の素数さん:05/01/13 19:25:36
(1) n>2を整数、F(n)をFibonacci数列とすると、
 Σ[k=1..n](1/F(k)) > (n^2)/(F(n+2)-1)
(2) ちょうど1つの辺の長さが1より大きい四面体の体積をVとおくと、V<1/8
(3) 正整数p,qに対し、|p/q-√2|≧(6-4√2)/(q^2)
(4) nを正整数、s>1を実数とすると、
 Σ[k=1..n](1/k^s) < 1+1/{2^(s-1)-1}
909132人目の素数さん:05/01/13 20:10:24

>>647 (1), 680
  s/9 ≧ t/3s ≧ u/t (単調減少)により、
  左辺 = (1+s+t+u)/[3(3+2s+t)]- u/t = 1/3 + (s+2t+3u)/[3(3+2s+t)] - u/t ≧ 1/3.

>>812,905
 n文字のとき、k次の基本対称式を S_k とおく。 S_0=1, ・・・・・, S_n=abc・・・・
 H(a+1,b+1,c+1) = nΣ[k=0,n] S_k / Σ[k=0,n-1] (n-k)S_k = 1 + Σ[k=1,n] k・S_k / Σ[k=0,n-1] (n-k)S_k
 = 1 + Σ[k=0,n-1] (k+1)S_{k+1} / Σ[k=0,n-1] (n-k)S_k.
 ところで、 (k+1)S_{k+1}/[(n-k)S_k] ≧ nS_n / S_{n-1} = H(a,b,・・・・).(←kについて単調減少)
 ∴ H(a+1,b+1,・・・・) - H(a,b,・・・・) ≧1.

>903 (3) 逆転の発想?
 f(y)=arcsin(arcsin(y)), g(y)=arctanh(y)=(1/2)Ln{(1+y)/(1-y)} とおく。arcsin(y)>y より
 f '(y) = 1/√{1-arcsin(y)^2}・1/√(1-y^2) > 1/(1-y^2) = g '(y).
 これと f(0)=0=g(0) から 0<y≦sin(1) ⇒ f(y)>g(y).
ぬるぽ
910132人目の素数さん:05/01/13 21:51:02
>895
(NOTEの問題)
 π ≧ A > B > 0 のとき関数 sin(Ax)/sin(Bx) は 0 < x < 1/2 において単調減少である。
(略証)
 f(x) = Ln{sin(Ax)/sin(Bx)} とおくと、 f '(x) = A/tan(Ax) - B/tan(Bx).
 tan(y)/y は|y| とともに増加するから A>B>0 より tan(Ax)/(Ax) > tan(Bx)/(Bx), f '(x)<0.
911905:05/01/13 22:06:19
>>909
dクス。
さすが ぬるぽ神。そこに痺れる憧れるぅ〜。


【問題】 非負実数 a, b に対し、次を示せ。
\sqrt[3]{a+\sqrt[3]{a}} + \sqrt[3]{b+\sqrt[3]{b}} ≧ \sqrt[3]{a+\sqrt[3]{b}} + \sqrt[3]{b+\sqrt[3]{a}}
912 ◆BhMath2chk :05/01/14 07:00:00
>>905
 H(a,b,c,d,e,f)+H(g,h,i,j,k,l)
=H(H(a,b,c),H(d,e,f))+H(H(g,h,i),H(j,k,l))
≦H(H(a,b,c)+H(g,h,i),H(d,e,f)+H(j,k,l))
≦H(H(a+g,b+h,c+i),H(d+j,e+k,f+l))
=H(a+g,b+h,c+i,d+j,e+k,f+l)。

f−>+∞,l−>+∞として
 (6/5)H(a,b,c,d,e)+(6/5)H(g,h,i,j,k)
≦(6/5)H(a+g,b+h,c+i,d+j,e+k)。

 H(a+1,b+1,c+1,d+1)
≧H(a,b,c,d)+H(1,1,1,1)
=H(a,b,c,d)+1。
913132人目の素数さん:05/01/14 11:34:52
[912] の一般化
【補題】 H(a_1, a_2, ・・・・,a_n)+ H(b_1, b_2, ・・・・,b_n)≦ H(a_1+b_1, a_2+b_2, ・・・・, a_n+b_n))
(略証) nについての帰納法による。
n=1 のとき 明らか。
n=2 のとき [647](6) より成立。
nが偶数のとき、n=2m とおく。
 H(a_1,a_2,・・・・・, a_{2m}) = H(H(a_1,a_2,・・・・,a_m), H(a_{m+1}, ・・・・, a_{2m})) を使うと
 左辺 = H(H(a_1, a_2, ・・・・,a_m), H(a_{m+1}, ・・・・, a_n))+ H(H(b_1, b_2, ・・・・,b_m), H(b_{m+1}, ・・・・, b_n))
 ≦ H(H(a_1, a_2, ・・・・,a_m) + H(b_1, b_2, ・・・・,b_m), H(a_{m+1}, ・・・・, a_n}) + H(b_{m+1}, ・・・・, b_n}))
 帰納法の仮定により、
 左辺 ≦ H(H(a_1+b_1, a_2+b_2, ・・・・,a_m+b_m), H(a_{m+1}+b_{m+1}, ・・・・, a_n+b_n))
 =H(a_1+b_1, a_2+b_2, ・・・・, a_n+b_n)。
nが奇数のとき
 上記により n+1 に対しては成立つ。そこで a_{n+1}→∞, b_{n+1}→∞ として n/(n+1) を掛ければ出る。

(系) H(a_1 +1,a_2 +1,・・・・,a_n +1)≧ H(a_1, a_2, ・・・・, a_n)+ H(1,1,・・・,1)= H(a_1, a_2, ・・・・, a_n)+1.

>812  [>>647(6)] は H(a+c,b+d) ≧ H(a,b) + H(c,d).
914132人目の素数さん:05/01/14 11:45:07
>911
 f(x)=x^(1/3), f(a)=A, f(b)=B, a-b=d, A-B=D とおく。 fは単調増加だから d・D>0.
 g は上に凸とすると、
[d・g(a+A)+D・g(b+B)]/(d+D) ≧ g(a+B)
[D・g(a+A)+d・g(b+B)]/(d+D) ≧ g(b+A)
辺々たすと
 g(a+A) + g(b+B) ≧ g(a+B) + g(b+A).
ぬるぽ

>909
> ところで、 (k+1)S_{k+1}/[(n-k)S_k] ≧ nS_n / S_{n-1} = H(a,b,・・・・).(←kについて単調減少)
 の元レスは >>263, >>269, >>480 .
915132人目の素数さん:05/01/14 16:46:56
>>912-913
実は昨日書き込んだ後、自己解決しました。
n=2の場合は直接に証明。n=2mの場合は >>912-913 と同じ方法。
n=2m+1のときの証明は、n=2m+2のときに帰着されますよね?
H(a, b, c) = H(a, b, c, H(a, b, c)) が一般のn文字でも成り立つから…。

>>913
H(a+1, b+1, c+1) - H(a, b, c) ≧ 1 が H(a+d,b+e,c+f) ≧ H(a, b, c) + H(d, e, f)
の特別な場合だったことに、言われるまで気づきませんでした。 il||li _| ̄|○ il||li
   ___
 ./  ≧ \
 |::::  \ ./ | アリガトウ ゴザイマス
 |::::: (● (● |   ベリィ グッジョ-ブ!
 ヽ::::... .ワ....ノ    n  コンゴトモ ヨロシク オネガイシマス…
 ̄ ̄   \    ( E)
フ     /ヽ ヽ_//
916132人目の素数さん:05/01/14 17:03:40
>>914
[d・g(a+A)+D・g(b+B)]/(d+D) ≧ g([d(a+A)+D(b+B)]/(d+D)) = g(a+B)
この変形に気づけそうにない…。すごすぎる。
917132人目の素数さん:05/01/14 20:30:14
>908(2)
 四面体ABCDで、AB以外の5辺≦1 とする。
 A,BからCD(の延長)に下ろした垂線を AH,BL とする。
 このとき V= (1/6)AH・BL・CD・sinθ, θは平面ACDと平面BCDがなす面角.
 AC,AD≦1 より AH ≦ √{1-(CD/2)^2},  BC,BD≦1 より BL ≦ √{1-(CD/2)^2}
 ∴ V ≦ (1/24)・CD・(4 -CD^2)・sinθ.
0 < x ≦1 ならば 3 -x(4-x^2) = (1-x){3-x(1+x)} ≧0, x(4-x^2) ≦ 3 @ x=1.
 V ≦ (1/8)sinθ ≦ 1/8.  (等号はACD,BCDが正3角形で、面角が90°のとき)
ぬるぽ
918132人目の素数さん:05/01/14 22:50:36
>908 (4)
 {1- 2/(2^s)}・(左辺) = Σ[k=1..n] {1/(k^s) -2/[(2k)^s]} < Σ[k=1..n] (-1)^(k-1) /(k^s)
 = 1 - {1/(2^s) -1/(3^s)} -{1/(4^s) -1/(5^s)} - ・・・ < 1.
 ∴ 左辺 < 1/{1-2/(2^s)} = 1 + 1/{2^(s-1)-1} = 右辺.
ぬるぽ

〜〜〜 寒いでつね。水道チョロチョロ流しておやすみ・・・(凍結防止) 〜〜〜.
919911:05/01/15 02:15:39
不等号の向きが反対でした。すみません。

非負実数 a, b に対し、
\sqrt[3]{a+\sqrt[3]{a}} + \sqrt[3]{b+\sqrt[3]{b}} ≦ \sqrt[3]{a+\sqrt[3]{b}} + \sqrt[3]{b+\sqrt[3]{a}}


>914の証明は正しいように思えるのですが、どうなんでしょう?
920132人目の素数さん:05/01/15 03:15:25
…で、ずっと検索してて見つけたんだけど、
Majorization Inequarity って何ですか?

 ↓ 最後のページ
http://math.berkeley.edu/~bobbypwh/pdfs/InequaSol2.pdf
921132人目の素数さん:05/01/15 03:28:25
>914
上に凸だから、不等号の向きが逆ですね。
922132人目の素数さん:05/01/15 12:29:22
わかすれ198より

338 名前:132人目の素数さん[] 投稿日:05/01/15(土) 12:17:06
x,y,zが実数で
1≦x^2+y^2+z^2≦2
をみたす。このとき
(3/5)(x-2y)^2+(1/3)(2x+y-z)^2+(4/15)(2x+y+5z)^2
の最大値、最小値
923132人目の素数さん:05/01/15 13:33:29
>>915
H(a, b, c) = H(a, b, c, H(a, b, c)) が一般のn文字でも成り立つから…。
多分無理

>>913
H(a+1, b+1, c+1) - H(a, b, c) ≧ 1 が H(a+d,b+e,c+f) ≧ H(a, b, c) + H(d, e, f)
の特別な場合だったことに、言われるまで気づきませんでした。 il||li _| ̄|○ il||li
>>912じゃないの
924132人目の素数さん:05/01/15 13:47:25
>922
 主軸系で考える....
 http://science3.2ch.net/test/read.cgi/math/1105415383/16-17
925132人目の素数さん:05/01/15 19:57:33
>>923
>多分無理

反例あげてみ。
926132人目の素数さん:05/01/15 20:14:06
>>925
意味不明
927風あざみ:05/01/15 20:32:25
>>908
(1)この式は
 Σ[k=1..n]({F(n+2)-1}/F(k)) > (n^2)と同値である。
F(n+2)-1=Σ[h=1..n]F(h)である。
(∵F(1)=1=F(3)-1
F(n+3)-1=F(n+2)-1+F(n+1)=Σ[h=1..n]F(h)+F(n+1)=Σ[h=1..n+1]F(h)
より帰納的に示せる。)
よって 相加・相乗平均の不等式より
Σ[k=1..n]({F(n+2)-1}/F(k))=Σ[k,h=1..n]{F(h)/F(k)}>(n^2){(n^2)^√Π[i=1,n]({F(i)}/{F(i)})}=n^2



928132人目の素数さん:05/01/16 05:48:55
>>903(1) は n文字には一般化できませんか?
929132人目の素数さん:05/01/16 05:49:54
x(1), x(2), x(3),... は正の数とする。
A(n)=(x(1)+...+x(n))/n 、
G(n)=(x(1)...x(n))^(1/n)
とおく。

全ての n (=2,3,4,...) に対して
n(A(n)-G(n))≧(n-1)(A(n-1)-G(n-1))
が成り立つ。

【証明】
nA(n)-(n-1)A(n-1)
= x(n)
= G(n)^n/{G(n-1)^(n-1)}
= G(n-1){G(n)/G(n-1)}^n
≧ G(n-1){n(G(n)/G(n-1))-(n-1)}・・・★
= nG(n)-(n-1)G(n-1)

だから問題の不等式が成り立つ。【証明終】

(★で不等式「t>0 ⇒ t^n≧nt-(n-1)」を使った。
t≧1のとき
t^(n-1) + t^(n-2) + ... + t^2 + t + 1 ≧ n
の両辺に t-1 をかければ t^n - 1 ≧ nt - n が得られる。
0<t<1のとき
1 + t + t^2 + ... + t^(n-2) + t^(n-1) ≦ n
の両辺に 1-t をかければ 1 - t^n ≦ n - nt が得られる。
いずれにせよ t^n≧nt-(n-1) が成り立つ。)
930132人目の素数さん:05/01/16 15:28:43
>908(1)
 左辺 ≧ 5/2 なので 16/7 ≧ 右辺 を示せば十分。
【補題】 F(n+2) ≧ 1+c・n^2, c=7/16.
(略証) nについての帰納法による。
 n=0 のとき F(2) = 1.
 n=1 のとき F(3) = 2 > 23/16 = 1+c.
 n=2 のとき F(4) = 3 > 11/4 = 1+4c.
 n=3 のとき F(5) = 5 > 79/16 = 1+9c.
 n=4 のとき F(6) = 8 = 1+16c.
 n≧5 のとき F(n+2) = F(n+1) + F(n) ≧ {1+c(n-1)^2} + {1+c(n-2)^2} = 2 + c[n^2 +(n-1)(n-5)] > 2 + cn^2.(終)
 ∴ 左辺 ≧ 5/2 ≧ 1/c ≧ (n^2)/[F(n+2)-1] = 右辺.
ぬるぽ

>928
 a=b なら Shapiro型だが。 >>497-501
931132人目の素数さん:05/01/16 15:49:05
>929
 ★の不等式 「t>0 ⇒ t^n ≧ nt-(n-1)」 は次にもあるよ。
 微積分と一つの不等式「数学100の定理」日本評論社, p.89 (1983)

(問題)
 A = {x(1)+x(2)+・・・+x(n)}/n,
 G = {x(1)x(2)・・・・x(n)}^(1/n),
 H = n/{1/x(1) +1/x(2) + .... +1/x(n)} とおくとき
 A^(n-1)・H ≧ G^n ≧ A・H^(n-1).
932914:05/01/16 16:26:37
>919,921
 仰せのとおり、スマソ.

>925
 [923]は、[915]の方法では無理だろう、の意味。他の方法も考えられる[>>909].

>917 (補足)
 △ACD=(1/2)AH・CD, △BCD=(1/2)BL・CD より
 V= (1/3)AH・△BCD・sinθ = (1/3)BL・△ACD・sinθ = (1/6)AH・BL・CD・sinθ
933915:05/01/16 18:53:13
>H(a, b, c) = H(a, b, c, H(a, b, c)) が一般のn文字でも成り立つから…。

H(a_1, …, a_n, H(a_1, …, a_n))
= (n+1)/{1/a_1) + … + (1/a_n) + (1/H(a_1, …, a_n))}
= (n+1)/{n/H(a_1, …, a_n) + (1/H(a_1, …, a_n))}
= H(a_1, …, a_n)

となるので、n文字でも成り立つと書いたのですが、
もしかして、俺なにか勘違いしてる? ('A`)
934132人目の素数さん:05/01/16 19:03:59
【問題】
(1) 1 ≦ a, b, c, d ≦ 2 に対して、(a+b)/(b+c) + (c+d)/(d+a) ≦ 4(a+c)/(b+d) を示せ。

(2) 整数 a, b が(a+b)(a-b)≠0 をみたすとき、 |(a+b)/(a-b)|^(ab) ≧ 1 を示せ。
935132人目の素数さん:05/01/16 21:02:23
>934
(1) b,d ≦ 2 ≦ 2a,2c だから,
 (a+b)/(b+c) = 2(a+c)/(b+d) -{(a+b)(2c-d) +b(2a-b)/2 +(c +b/2)(2c-b)}/[(b+c)(b+d)] ≦ 2(a+c)/(b+d).
(c+d)/(d+a) = 2(a+c)/(b+d) -{(c+d)(2a-b) +d(2c-d)/2 +(a +d/2)(2a-d)}/[(d+a)(b+d)] ≦ 2(c+a)/(d+b).
辺々たす。

(2) |a+b|^2 - |a-b|^2 = 4ab より、
 ab≧0 のとき |a+b|≧|a-b|, 左辺 ≧1.
 ab≦0 のとき |a+b|≦|a-b|, 左辺 ≧1.
ぬるぽ

>933
それを使って
> n=2m+1のときの証明は、n=2m+2のときに帰着されますよね?
を示すのは 多分無理、という意味と思われ....
936132人目の素数さん:05/01/17 01:18:58
>>935
なるほど
937132人目の素数さん:05/01/17 02:36:53
>934(2) は、a, b が整数である必要はないですよね?
(a+b)(a-b)≠0 さえ満たしていれば…。
938132人目の素数さん:05/01/17 02:38:10
> n=2m+1のときの証明は、n=2m+2のときに帰着されますよね?

たしかに、ダメですね。切腹します。
939132人目の素数さん:05/01/17 02:56:16
>>903(4) 正の実数x,y,zがx+y+z=xyzを満たすとき、
 F_n = (x^n)/√(1+x^2) + (y^n)/√(1+y^2) + (z^n)/√(1+z^2)
について、
 n = 0, 1 のとき、F_n ≦ 3(√3)^n/2、 n = 2, 3, … のとき、F_n ≧ 3(√3)^n/2

でOKですよね? あっているかどうか確認たのも〜。

>>907(4) の方法で、x=tan(A), y=tan(B), z=tan(C), 0<A,B,C<π/2 とおくと、条件式より A+B+C=π
n=0 のときは、cosA+cosB+cosC ≦ 3cos[(A+B+C)/3]
n=1 のときは、>>903(2)
n≧2 のときは、0≦θ<π/2 において、左辺の関数が下に凸だから。
 f(θ) = (tanθ)^n・cosθ
 f'(θ) = (tanθ)^(n-1)・[n-(sinθ)^2]/cosθ
 f''(θ) = (tanθ)^(n-2)・[(n-1)n-(sinθcosθ)^2]/(cosθ)^3 > 0
940939:05/01/17 05:54:55
まとめると、こんな感じでせうか… (;´д`)ハァハァ
 1 ≦ F_0 ≦ 3/2
 2 ≦ F_0 ≦ 3(√3)/2
n = 2, 3, … のとき、F_n ≧ 3(√3)^n/2
941132人目の素数さん:05/01/17 06:07:48
もひとつおまけに
 x/(1+x^2) + y/(1+y^2) + z/(1+z^2) ≦ 3√3/4
942132人目の素数さん:05/01/17 06:14:02
>>940 3行目は F_1 の書き間違い
>>941 は、正の実数x,y,zがx+y+z=xyzを満たす条件下で。
943132人目の素数さん:05/01/17 06:19:05
たびたびスミマセン。
>>940の最小値には、イコールがつかないですね…
944132人目の素数さん:05/01/17 06:47:51
 G_n = (x^n)/(1+x^2) + (y^n)/(1+y^2) + (z^n)/(1+z^2)
こちらのほうも、作れそうですね。

n=0 のときは、(cosα)^2+(cosβ)^2+(cosγ)^2 となるけど
凹凸が一定でないので無理そうですが…
945132人目の素数さん:05/01/17 06:49:15
次スレ立ててみました。

不等式への招待 第2章
http://science3.2ch.net/test/read.cgi/math/1105911616/l50
946132人目の素数さん:05/01/17 10:39:25
>939-940
0 < sinθ・cosθ = (1/2)sin(2θ) ≦ 1/2 なので
まとめると、こんな幹事でせうか…
 n≦(1-√2)/2 のとき F_n ≧ 3(√3)^n /2,
 0≦n≦1 のとき 0 < F_n ≦ 3(√3)^n /2,
 (1+√2)/2≦n のとき F_n ≧ 3(√3)^n /2.
947132人目の素数さん:05/01/17 21:58:51
>941
 左辺 = (1/2)sin(2A) + (1/2)sin(2B) + (1/2)sin(2C) ≦ (3/2)sin(2π/3) = (3/4)√3.

>944
 f(θ) = (tanθ)^n (cosθ)^2.
 f '(θ) = n(tanθ)^(n-1) -2(tanθ)^(n+1)・(cosθ)^2.
 f "(θ) = n(n-1)(tanθ)^(n-2)/(cosθ)^2 -2(n+1)(tanθ)^n + 4{(tanθ)^(n+2)}(cosθ)^2
= {n(n-1)[1+(tanθ)^2]^2 -2(n+1)(tanθ)^2[1+(tanθ)^2] +4(tanθ)^4}{(tanθ)^(n-2)}(cosθ)^2
= {n(n-1) +2(n^2 -2n-1)(tanθ)^2 +(n-1)(n-2)(tanθ)^4}{(tanθ)^(n-2)}(cosθ)^2
  = {-D_n +[(n-1)^2 -2 +(n-1)(n-2)(tanθ)^2]^2}{(tanθ)^(n-2)}(cosθ)^2 /[(n-1)(n-2)].
 ここに、判別式 D_n = [(n-1)^2 -2]^2 -n(n-2)(n-1)^2 = 4 -3(n-1)^2.

 n≦1-(2/3)√3 = -0.154700538379252… または n≧1+(2/3)√3 = 2.154700538379252… のとき
 D_n ≦0, f "≧0 (下に凸).
 ∴ (x^n)/(1+x^2) + (y^n)/(1+y^2) + (z^n)/(1+z^2) ≧ (3/4)(√3)^n.
ぬるぽ
948未解答なもの (たぶん):05/01/17 22:00:55
>>360 (D.D.Adamovic)
m>2, Σ[k=1,m] x_k↑ = 0↑ のとき、
(1). (m-2) Σ[k=1,m] |x_k|^2 = Σ[1≦i<j≦m] |x_i +x_j|^2.
(2). (m-2) Σ[k=1,m] |x_k| ≧ Σ[1≦i<j≦m] |x_i +x_j|.

(1)>>364、(2) 未解答


>>407 正の数 a,b,c に対して、1/{a(1+b)} + 1/{b(1+c)} + 1/{c(1+a)} ≧ 3/(1+abc)


>>783 整数 a, b, c, d が a>b>c>d>0、ad=bc をみたすとき、(a-d)^2 ≧ 4d+8


>>815 a,b,cを正の有理数、(a^2)+(b^2)=(c^2)で(ab)/2は整数とする。 ab≧10 を示せ
949132人目の素数さん:05/01/17 22:26:39
>948
[407] は [477] [503] [>>504-505] の辺りにも...
950132人目の素数さん:05/01/17 22:34:01
>>949
あぁほんとだ。
どおりで解いたことあるような気がしてた。
951問題:05/01/17 22:52:33
三角形の辺の長さ a, b, c について
(1) (b+c-a)a^2 + (c+a-b)b^2 + (a+b-c)c^2 ≦ 3abc
(2) (a+b-c)^a・(b+c-a)^b・(c+a-b)^c ≦ (a^a)(b^b)(c^c)

正の数 a, b, c, p, q, r に対して、
(3) p^4/a^3 + q^4/b^3 + r^4/c^3 ≧ (p+q+r)^4/(a+b+c)^3
(4) p+q+r=1 のとき、a+b+c ≧ (a^p)(b^q)(c^r) + (a^q)(b^r)(c^p) + (a^r)(b^p)(c^q)

(5) 非負実数 a, b, c に対して、(bc+ca+ab)(a+b+c)^4 ≦ 27(a^3+b^3+c^3)^2

(6) 自然数 n と a>b>0 に対して、a^n-b^n > n(a-b)(ab)^[(n-1)/2]

(7) 正の数 a_1, … a_k に対して、相乗平均を G とおくと、(1+a_1)…(1+a_n) ≧ (1+G)^n
       ___ 
彡     /  ≧ \    彡 ビュゥ……
  彡   |:::  \ ./ |  彡
      |:::: (● (●|     (7) は既出のような気がしますが…
      ヽ::::......ワ...ノ     (3) は一般化できそうな予感
        人つゝ 人,,
      Yノ人 ノ ノノゞ⌒〜ゞ      よろしくお願いします。
    .  ノ /ミ|\、    ノノ ( 彡
     `⌒  .U~U`ヾ    丿
             ⌒〜⌒
952未解答なもの (その2):05/01/18 01:31:33
565 名前:三角形と三角関数の不等式[sage] 投稿日:04/11/01(月) 09:24:59

(1) [1963 Eotvos] 0<x<π/2 のとき、(1/(sin x)+1)(1/(cos x)+1) > 5

(2) [1978 Austria] tan k (k = 1度, …, 44度)の相加平均をA、相乗平均をGとおくとき、A > (√2)-1 > G

(3) [1959 IMO shortlist] 0≦x≦π/2、π/6<y<π/3 のとき、
tan{(π sin x)/(4 sin y)} + tan{(π cos x)/(4 cos y)} > 1

(4) 0<x<π に対して、{sin(x)/x}^3 < {(π^2-x^2)/(π^2+x^2)}^2  [類 : 不等式への招待 P.39 ]

(5) 0<x<1 に対して、(1-x^2){1+(x-x^2)^3}/(1+x^2) < (sin πx)/(πx)
953 ◆BhMath2chk :05/01/18 02:00:00
>>783
p=gcd(a,b),q=a/p,r=b/p,s=c/qとすると
a=pq,b=pr,c=qs,d=rs。
a>b>c>d>0からp/s>q/r>1。
a−d≧(r+1)(s+1)−rs=r+s+1≧2√(rs)+1=2√(d)+1。
等号が同時に成り立つことはないので
a−d>2√(d)+1。
(a−d)^2>4d+4√(d)+1≧4d+5。
(a−d)^2は4で割ると余りは0か1なので
(a−d)^2≧4d+8。
954132人目の素数さん:05/01/18 02:01:04
>951 (1)
b+c-a=x, c+a-b=y, a+b-c=z とおく。
(1) 左辺 = [x(y+z)^2 +y(z+x)^2 +z(x+y)^2]/4 = (y+z)(z+x)(x+y)/4 +xyz
= 3(y+z)(z+x)(x+y)/8 -(1/8){x(y-z)^2 +y(z-x)^2 +z(x-y)^2} ≦ 3(x+y)(y+z)(z+x)/8 = 右辺.

(7) 初めて見たが...
 f(x)=Ln(1+e^x) とおくと f '(x)=(e^x)/(1+e^x), f"(x) =(e^x)/(1+e^x)^2 >0 (下に凸) より。
ぬるぽ
955132人目の素数さん:05/01/18 03:04:14
>952 (>>565)
(1) {1+sin(x)}{1+cos(x)} -5sin(x)cos(x) = 1 + sin(x)+cos(x) -4sin(x)cos(x)
 = 1 + 1 -2sin(2x) ≧ 2{1-sin(2x)} >0.

(2) y=tan(x) は下に凸だから、[tan(0)+tan(π/4)]/2 ≧ [tanθ + tan(π/4 -θ)]/2 ≧ tan(π/8).
  また、tanθ・tan(π/4 -θ) = 1 - tanθ - tan(π/4 -θ) ≦ 1-2tan(π/8) = {tan(π/8)}^2.
  ∴ 1/2> A > tan(π/8) = √2 -1 > G >0.

(6) -a+b+c=x, a-b+c=y, a+b-c=z とおくと、相加・調和平均より
 1/a = 2/(y+z) ≦ (1/y +1/z)/2,
 1/b = 2/(z+x) ≦ (1/z +1/x)/2,
 1/c = 2/(x+y) ≦ (1/x +1/y)/2.
 辺々たす。
ぬるぽ
956132人目の素数さん:05/01/18 03:17:06
こんな遅くまで、乙です。
新スレのほうに、例の Majorizatoin Inequality と
外国の不等式ヲタのサイトを発見したので、見てみてください。(笑)
957132人目の素数さん:05/01/18 08:17:50
>951 (7)
 S_kは{a_i}のk次の基本対称式, G = (S_n)^(1/n) = (a_1・a_2・・・・a_n)^(1/n) とおく。
 相加・相乗平均により、
 左辺 = Σ[k=0,n] S_k ≧ Σ[k=0,n] C[n,k]・G^k = (1+G)^n.
ぬるぽ
958132人目の素数さん:05/01/18 11:55:12
>951 (2)
 b+c-a=x, c+a-b=y, a+b-c=z とおくと、x+y+z=a+b+c.
 yz = (c+a-b)(a+b-c)= a^2 - (b-c)^2 ≦ a^2 など.
 Ln(左辺) = (1/2){(y+z)Ln(z) + (z+x)Ln(x) + (x+y)Ln(y)} = (1/2){y・Ln(yz) + z・Ln(zx) + x・Ln(xy)}
 ≦ y・Ln(a) + z・Ln(b) + x・Ln(c) ≦ a・Ln(a) +b・Ln(b) +c・Ln(c) = Ln(右辺).

>928 (凡例)
 nが偶数, 0<a<b のとき x_{2k-1}=Xo, x_{2k}=Xe≠Xo に対して
 左辺 = (n/2){Xo/(aXe+bXo) +Xe/(aXo+bXe)} = (n/2)[a(Xo-Xe)^2 +2(a+b)XoXe]/{(aXe+bXo)(aXo+bXe)}
 = [n/(a+b)]{1 -(a/2)(b-a)(Xo-Xe)^2/[XoXe(a+b)^2 +ab(Xo-Xe)^2]} < n/(a+b) = 右辺.
となるので、nが偶数のときは a≧b が必要かと....
959132人目の素数さん:05/01/18 22:02:56
>951 (2)
 [958]の訂正 死んでお詫びを……(AA省略)
 Ln(1+x) ≦x より a・Ln(a+b-c) = a{Ln(a)+Ln[1+(b-c)/a]} ≦ a・Ln(a) +b-c.
 これを循環的にたす。

>951 (5)
 bc+ca+ab = (a^2 +b^2 +c^2) -(1/2)[(b-c)^2 +(c-a)^2 +(a-b)^2] ≦ (a^2 +b^2 +c^2).
 (a+b+c)^2 = 3(a^2 +b^2 +c^2) -(b-c)^2 -(c-a)^2 -(a-b)^2 ≦ 3(a^2 +b^2 +c^2).
 辺々かけて、 左辺 ≦ 9(a^2 +b^2 +c^2)^3.
これと [>>153]の(補題) [>>688] を使って、
 左辺 ≦ 9(a^2 +b^2 +c^2)^3 ≦ 27(a^3+b^3+c^3)^2 = 右辺.
ぬるぽ
960132人目の素数さん:05/01/18 22:17:02
   〜〜相加・相乗平均の練習問題〜〜
>951(4)
 p+q+r=1 ⇒ pa+qb+rc ≧ (a^p)(b^q)(c^r) これを循環的にたす。

>951(6)
 (a^n-b^n)/(a-b) = a^(n-1) +a(n-2)b +…… + ab^(n-2) +b^(n-1) > n(ab)^[(n-1)/2].
ぬるぽ
961132人目の素数さん:05/01/19 03:35:24
>>944>>947
0<A,B,C<π/2, A+B+C=π のとき、>>580より次が成り立つ。等号はA=B=C=π/3
 0 < cosA・cosB・cosC ≦ 1/8

G_n = (tanA)^n (cosA)^2 + (tanB)^n (cosB)^2 + (tanC)^n (cosC)^2 において、
 G_0 = (cosA)^2 + (cosB)^2 + (cosC)^2 = 1-2cosA・cosB・cosC
 G_2 = (sinA)^2 + (sinB)^2 + (sinC)^2 = 2+2cosA・cosB・cosC
だから、G_0 > 3/4、 0 < G_2 < 9/4

>>947より、0 < G_1 ≦ (3/4)√3、 n=3,4,…のとき G_n ≧ (3/4)(√3)^n

でよろしいでせうか? (;´д`)ハァハァ
>>360 (D.D.Adamovic) の(2)番。 (1)の解答は>>364
m>2, Σ[k=1,m] x_k↑ = 0↑ のとき、
(1). (m-2) Σ[k=1,m] |x_k|^2 = Σ[1≦i<j≦m] |x_i +x_j|^2.
(2). (m-2) Σ[k=1,m] |x_k| ≧ Σ[1≦i<j≦m] |x_i +x_j|.

>>565
(3) [1959 IMO shortlist] 0≦x≦π/2、π/6<y<π/3 のとき、
 tan{(π sin x)/(4 sin y)} + tan{(π cos x)/(4 cos y)} > 1

(4) 0<x<π に対して、{sin(x)/x}^3 < {(π^2-x^2)/(π^2+x^2)}^2  [類 : 不等式への招待 P.39 ]

>>815 a,b,cを正の有理数、(a^2)+(b^2)=(c^2)で(ab)/2は整数とする。 ab≧10 を示せ。

>>908 (3) 正整数p,qに対し、|p/q-√2|≧(6-4√2)/(q^2)

>>951 (3) 正の数 a, b, c, p, q, r に対して、 p^4/a^3 + q^4/b^3 + r^4/c^3 ≧ (p+q+r)^4/(a+b+c)^3
963132人目の素数さん:05/01/19 05:50:03
>>951(3) の類題をコレクションの中から発見!
このスレには書いてなかったよね? (いちおう検索したけど…)

【類題】 文字はすべて正の数とする。
 p^2/a + q^2/b + r^2/c ≧ (p+q+r)^2/(a+b+c)

(証)
 p^2/a + q^2/b - (p+q)^2/(a+b) = (aq-bp)^2/[ab(a+b)] ≧ 0
 ∴ p^2/a + q^2/b ≧ (p+q)^2/(a+b)
これを2回用いて、
 p^2/a + q^2/b + r^2/c ≧ (p+q)^2/(a+b) + r^2/c ≧ (p+q+r)^2/(a+b+c)
等号成立条件は
 aq=bp かつ (a+b)r=c(p+q) ⇔ p/a = q/b = r/c
964132人目の素数さん:05/01/19 05:57:32
>>955
(>>565)(1) [1963 Eotvos] 0<x<π/2 のとき、(1/(sin x)+1)(1/(cos x)+1) > 5

の解答について質問です。

{1+sin(x)}{1+cos(x)} -5sin(x)cos(x)
 = 1 + sin(x)+cos(x) -4sin(x)cos(x)
 = 1 + 1 -2sin(2x)             ← ここは なぜですか?
 ≧ 2{1-sin(2x)}
 > 0.
965964:05/01/19 05:59:22
あぁ、分かりました。
問題文に 0<x<π/2 とあるから、合成して sin(x)+cos(x) ≧ 1 なんですね。
スマソ。
966132人目の素数さん:05/01/19 09:22:16
>962
>>565(5)もまだだよ。
967132人目の素数さん:05/01/19 09:30:30
>>953
なるほど。ありがとうございまする。
こりゃ思いつかんわ…。
968132人目の素数さん:05/01/19 19:09:06
>951(3) >963
 a,b,c,n,p,q,r>0. p/a=P, q/b=Q, r/c=R とおく。 y=x^(n+1) は下に凸なのでJensenにより、
 左辺 = p^(n+1)/(a^n) + q^(n+1)/(b^n) +r^(n+1)/(c^r) = a・P^(n+1) +b・Q^(n+1) +c・Q^(n+1)
 ≧ (a+b+c){(aP+bQ+cR)/(a+b+c)}^(n+1) = (p+q+r)^(n+1) /(a+b+c)^n = 右辺.
 等号条件は P=Q=R すなわち p/a = q/b = r/c.
ぬるぽ
969968:05/01/19 19:59:11
[968] に写し間違い,スマソ。 r^(n+1)/(c^n)、 c・R^(n+1) 

[951]への解答レス(主なもの)
  (1)[954] (2)[959] (3)[968] (4)[960] (5)[959] (6)[960] (7)[954][957]

>961
 3/4 ≦ G_0 < 1、 2 < G_2 ≦ 9/4  でいいんぢゃない?
 (0<A,B,C<π なら 3/4 ≦ G_0 < 3、 0 < G_2 ≦ 9/4)
970132人目の素数さん:05/01/19 21:53:12
>>811(1)も未解答では?
971132人目の素数さん:05/01/19 23:49:36
実数a[k],k=1,2,...,nに対し
|sin(a[1])|+|sin(a[2])|+...+|sin(a[n])|+|cos(a[1]+a[2]+...+a[n])|≧1
972132人目の素数さん:05/01/20 00:59:14
>>969
dクス。
 ↑ の問題と、>>962のリストと、次の3問。

>>563(7) 自然数 m, n と実数 0≦x≦1 に対して、(1-x^n)^m+{1-(1-x)^m}^n ≧1

>>565(5) 0<x<1 に対して、(1-x^2){1+(x-x^2)^3}/(1+x^2) < (sin πx)/(πx)

>>811(1) 1対1の上への関数 f : [0,1]→[0,1] は狭義単調増加であるとし、逆関数をgとおく。
  0<t<1のとき、∫[0,1] (f(x)+g(x))^t dx≧(2^t)/(1+t) を示せ。
974132人目の素数さん:05/01/20 09:36:51
ごめん、563(7) は>>662にあった
975132人目の素数さん:05/01/20 10:08:21
>>973 さらに追加
>>791
各辺の長さが整数値の三角形ABCがあり、∠A=2∠B ∠C>π/2を満たす。
この時、AB+BC+CA≧77を示せ


等号成立条件しか書かれていないので未解決としてよいかと
出題は某国数学オリンピック。
976132人目の素数さん:05/01/20 12:23:46
>971
 a[k]/π に最も近い整数を q[k]、r[k] = a[k] - q[k]π とすると, |r[k]| ≦ π/2.
 左辺 = |sin(r[1])| + …… + |sin(a[n])| + |cos(S)|
 ≧ (2/π)(|r[1]|+|r[2]|+ …… +|r[n]|) + |cos(S)| = (2/π)Θ + |cos(S)|.
 ここに S = r[1] + r[2] + …… + r[n], Θ = |r[1]| + |r[2]| + …… + |r[n]| とおいた。
 Θ ≧ π/2 のときは成立。
 Θ < π/2 のとき 0 ≦ S ≦ Θ より, 左辺 ≧ (2/π)Θ + cosΘ ≧1.
ぬるぽ

>974
 ごめん、563(7) は >>706-707 で解決のように見えまつが、>>705 (未解決)を使ってますた。
977 ◆BhMath2chk :05/01/20 14:00:00
 |sin(x+y)|
≦|sin(x)||cos(y)|+|cos(x)||sin(y)|
≦|sin(x)|+|sin(y)|。

 1
≦|sin(Σ(a(k)))|+|cos(Σ(a(k)))|
≦Σ|sin(a(k))|+|cos(Σ(a(k)))|。
978132人目の素数さん:05/01/20 16:09:47
ウホッ いい不等式 ( ゚∀゚) テヘ
979132人目の素数さん:05/01/20 16:28:30
一年二十四日。
980132人目の素数さん:05/01/20 22:13:31
>>951(6) を改造。
自然数 n と a>b>0 に対して、(n/2)(a-b)[a^(n-1)-b^(n-1)] > a^n-b^n > n(a-b)(ab)^[(n-1)/2]
( ゚∀゚) テヘッ
981132人目の素数さん:05/01/20 22:28:19
∫[0,1]{x/cos(x)}dx < log(2)
982132人目の素数さん:05/01/21 06:05:38
>>976
最後の行
> Θ < π/2 のとき 0 ≦ S ≦ Θ より, 左辺 ≧ (2/π)Θ + cosΘ ≧1.
において、
  左辺 ≧ (2/π)Θ + cosΘ
ここまでは分かりましたが、それが ≧1 となるのは何故ですか?
983982:05/01/21 06:12:39
まさか、f(θ) = (2/π)θ + cosθ を微分して、
0<θ<π/2 における増減を調べて ≧1 を確認
なんて面倒なことをしないですよね?
984982
グラフから、0≦θ<π/2 において cosθ ≧ 1-(2/π)θ であることを使うのですか?